NCLEX 3

Pataasin ang iyong marka sa homework at exams ngayon gamit ang Quizwiz!

Air/Pulmonary Embolism: S&S and Care

S&S Include: chest pain, difficulty breathing, tachycardia, pale/cyanotic, sense of impending doom. Care: turn patient to the *LEFT SIDE* and *LOWER* the head of the bed

Woman in Labor w/ NON-Reassuring FHR: S&S and Care

S&S Include: late decelerations, decreased variability, fetal bradycardia, etc. Care: turn on *LEFT SIDE* (also give O2, stop pitocin and increase fluids)

What are the S/S of serotonin syndrome?

S-weating A-pprehension AKA=sense of doom D-izziness H-eadache

Cholinergic Agonist Effects: SLUDGE BAM

S: salivation, sweating, secretions L: lacrimation U: urination D: defecation G: gastrointestinal upset E: emesis B: bradycarida A: abdominal cramps M: miosis

IADLs: SCUM

S: shopping C: cooking and cleaning U: using telephone or transportation M: managing money and meds

Staph aureus causes: SOFT PAINS

S: skin infections O: osteomyelitis F: food poisoning T: toxic shock syndrome P: pneumonia A: acute endocarditis I: infective arthritis N: necrotizing fasciitis S: sepsis

Hormones that increase blood glucose: STENGG

S: somatotropin (growth hormone) T: thyroid hormones E: epinephrine N: norepinephrine G: glucogon G: glucocorticosteroids

Calcium Channel Blocker Side effects: SHED the GAPS

S: stevens johnson syndrome H: headache E: edema D: dizziness G: gingival hyperplasia A: angina P: palpitations S: sleepiness

Airborne precautions are for what 4 diseases?

SARS, TB, Measles and Varicella

Zoloft(Sertraline) is a?

SSRI

What is the #1 nursing diagnosis for a patient taking phenothiazines?

Safety

If you know what a particular drug does pick a SE in the ______ body system.

Same

HHNK S/S?

Same as dehydration

S/S of Hyperthyroidism?

Same as hypermetabolism... agitated, nervous, diarrhea, hot, ^HR, ^BP, thin, hyperactive etc.

Acidosis/alkalosis compensation: RUB MUB

Respiratory Uses Bicarb, Metabolic Uses Breathing (Respiratory acidosis retains bicarb and alkalosis excretes bicarb. Metabolic acidosis increases ventilation and alkalosis decreases ventilation)

If the potassium is over 6 you?

Stop what you're doing and assess If negative effects present you call rapid response. If no negative effects you do the same as "high but within the 5's)

A client in the allergy clinic develops all of these clinical manifestations after receiving an intradermal injection of an allergen. Which symptom requires the most immediate action by the nurse? A. Anxiousness B. Urticaria C. Pruritus D. Stridor

Stridor

What side do you hold the cane?

Strong side.

When changing the dose of Prozac for a young adult you must monitor for?

Suicide

15. Which of the following is the most important to consider when assisting the client in passive range-of-motion exercises? 1. Flex the joint to the point of discomfort. 2. Work from the proximal joints to the distal joints. 3. Quickly work through the range of motion. 4. Support the distal joints while performing range-of-motion exercises.

Support the distal joints while performing range-of-motion exercises.

Cephalohematoma is?

Swelling caused by bleeding between the osteum and periosteum of the skull. This swelling does not cross suture lines.

Do not give what type of pain medication to a women in labor if the medication is liekly to peak when the baby is born?

Systemic- AKA IV, IM and oral

When giving a Calcium Channel Blocker you hold and notify if?

Systolic is 100 or lower.

PVC's fall on what wave of the previous beat?

T wave

s/s hypoglycemia: TIRED

T: tachycarida I: irritability R: restlessness E: excessive hunger D: diaphoresis (also: cold and clammy, need some candy)

Traction care: TRACTION

T: temp of extremity R: ropes hang freely A: alignment C: circulation check T: type and location of fracture I: increase fluid intake O: overhead trapeze N: no wts on bed or floor

Sulfonylurea side effects: THAL

T: teratogenicity H: hypoglycemia A: abdominal upset L: lipid deposit

Teratogenic substances: TAP CAP

T: thalidomide A: alcohol P: progestins C: coritcosterroids A: aspirin P: phenytoin

Tricyclic antidepressant side effects: TCA'S

T: thrombocytopenia C: cardiac (arrythmia, MI, stroke) A: anticholinergic (tachycardia,urinary retention etc) S: seizures

Kalemias do the same as the prefix except for the what?

The HR and urine output.

Cushing Syndrome is the oversecretion of?

The adrenal cortex.

Crutches always move with what leg?

The bad leg.

Thoracic =

Upper

All psych medications cause what?

Weight changes and low BP.

Mast cell stabilizers

prevent mast cell membranes from opening when an allergen binds to IgE; prevents the symptoms of allergic rhinitis but are not useful in acute episodes

corticosteroids

prevents synthesis of mediators; avoided for rhinitis; used for other problems associated with type 1 reactions

"floxicin" - besifloxacin, ciprofloxacin, levofloxacin, moxifloxacin, ofloxacin

quinolone - antibiotic

Assess for phlebitis

reddness, pain, heat, swelling

"tretin" - acitretin, alitretinoin, isotretinoin, tretinoin

retinoid (dermatologic agent - form of vit A)

"setron" - alosetron, dolasetron, granisetron, ondansetron, palonsetron

serotonin receptor antagonist - antiemetic and antinauseant

"thiazide" - chlorothiazide, hydrochlorothiazide, methyclothiazide

thiazide diuretics

"pramine" - clomipramine, desipramine, imipramine, trimipramine "tyline" - amitriptyline, nortriptyline, protriptyline

tricyclic antidepressant

What is the biggest problem in abuse?

Denial

MAOI's treat?

Depression

What are the two tocolytics?

Terbutaline= Brethine Nifediopine= Procardia

Who can sterilize my bowel?

"Neo" "Kan"

90% of Calcium Channel Blockers end in?

"dipine" and "zem"

After week 12 you can subtract 9 to get appropriate weight gain.

...

Drugs DON't determine peak and trough times, the ROUTE does.

...

If asked a question and your patient is critical remember to always STAY with the patient!!

...

Treatment for low blood sugar in Type II is the same as Type I low blood sugar treatment.

...

What should you teach a patient taking Baclofen(Lioresil, Flexeril)?

Don't drive/ operate machinery Don't drink alcohol Don't care for children under 12

A client with chronic pain states, "I just want to be pain-free. Do something to make that happen." The most therapeutic response is: 1. "Together we will all work at making your pain tolerable." 2. "I will do everything I can to manage your pain; I promise." 3. "Are you feeling depressed or anxious because of your pain?" 4. "You sound anxious. Would you like something for your nerves?"

1. "Together we will all work at making your pain tolerable."

The nurse on a postoperative care unit is assessing the quality of the client's pain. In order to obtain this specific information about the pain experience from the client, the nurse should ask: 1. "What does your discomfort feel like?" 2. "What activities make the pain worse?" 3. "How much does it hurt on a scale of 0 to 10?" 4. "How much discomfort are you able to tolerate?"

1. "What does your discomfort feel like?"

1. The nurse recognizes that facilitating correct body alignment for a dependent client may well result in which of the following positive client outcomes? (Select all that apply.) 1. A comfortable night's sleep 2. Minimized activity intolerance 3. Muscle tone that promotes ambulation 4. Reduction of falls caused by general weakness 5. Minimal strain placed on the spinal column 6. Increased socialization, resulting in peace of mind

1. A comfortable night's sleep 2. Minimized activity intolerance 3. Muscle tone that promotes ambulation 4. Reduction of falls caused by general weakness 5. Minimal strain placed on the spinal column

Normal weight gain for pregnancy?

28 + or - 3 lb

The first stage of labor has how many phases?

3

ANS: 2140 DIF: Cognitive Level: Application REF: 190 OBJ: Special Questions: Alternate Item Format TOP: Nursing Process: Implementation MSC: NCLEX: Physiological Integrity

1. A patient's temperature has been 101° F (38.3° C) for several days. The patient's normal caloric intake to meet nutritional needs is 2000 calories per day. Knowing that the metabolic rate increases 7% for each Fahrenheit degree above 100° in body temperature, calculate the total calories the patient should receive each day. ____________________

Humalog duration?

3 hours

What are the main 6 organs in prioritized order?

1. Brain 2. Lung 3. Heart 4. Liver 5. Kidney 6. Pancreas

"eprazole" - esomeprazole, omeprazole, rabeprozole "oprazole" - dexlansoprazaole, lansoprazole, pantoprazole

Protein Pump Inhibitor

When given a prioritization question for lethal arrythmias if you are asked to prioritize and one say it happened 6 minutes ago and the other says 15 minutes ago which do you choose?

Always the one closest to the 8 minute mark. After 8 minutes the survival rate is LOW.

5 examples of psychosis of dementia?

Alzheimers, dementia, organic brain syndrome, wernickesand seniality.

Which one of the following nursing interventions for a client in pain is based on the gate-control theory? 1. Giving the client a back massage 2. Changing the client's position in bed 3. Giving the client a pain medication 4. Limiting the number of visitors

1. Giving the client a back massage

When a client's husband questions how a patient-controlled analgesia (PCA) pump works, the nurse explains that the client: 1. Has control over the frequency of the intravenous (IV) analgesia 2. Can choose the dosage of the drug received 3. May request the type of medication received 4. Controls the route for administering the medication

1. Has control over the frequency of the intravenous (IV) analgesia

CO2 norm?

35-45

PaCO2

35-45

2. The nurse chooses to use a mechanical lift to move an obese immobile client. The nurse recognizes that the positive outcomes for both the client and the staff resulting from this intervention will be: (Select all that apply.) 1. Less of the client's body will be dragged along the sheets during the transfer 2. There will be less chance of injuring the skin on the client's elbows and buttocks 3. The staff involved in the transfer will have less likelihood of self-injury 4. The staff will have a greater degree of control over the move 5. The client will feel physically safer during the transfer 6. The move will be accomplished more quickly

1. Less of the client's body will be dragged along the sheets during the transfer 2. There will be less chance of injuring the skin on the client's elbows and buttocks 3. The staff involved in the transfer will have less likelihood of self-injury 4. The staff will have a greater degree of control over the move

Discharge teaching for a laminectomy?

Don't sit for longer then 30 minutes for 6 weeks Ly flat and log roll for 6 weeks No driving for 6 weeks Do not lift more then 5 lb. for 6 weeks

Describe a 3 point gait?

1. Move two crutches and bad leg together. 2. Move good foot. Move all three together and then the good leg.

Overdose of a downer causes everything to go?

Down

Describe a 4 point gait?

1. One crutch 2. Opposite foot. 3. Other crutch. 4. Other foot. It moves one at a time so 1,2,3,4 and 1,2,3,4, and 1,2,3,4 so crutch, foot, other crutch, other foot, and crutch,foot, other foot, other crutch etc,

Withdrawal of an upper causes everything to go?

Down

If your are pressurizing for drawing up insulin you?

Draw what you are backwards. NR----> RN

What do you not delegate to a family member?

Safety responsibilities-" can you watch your dad really fast while I grab something."

ANS: B The incisional redness and warmth are indicators of the normal initial (inflammatory) stage of wound healing by primary intention; the nurse should document the wound appearance and continue to monitor the wound. Notification of the health care provider, assessment every 2 hours, and obtaining wound cultures are not indicated because the healing is progressing normally.

1. The nurse assesses a surgical patient the morning of the first postoperative day and notes redness and warmth around the incision. Which action by the nurse is most appropriate? a. Obtain wound cultures. b. Document the assessment. c. Notify the health care provider. d. Assess the wound every 2 hours.

Hct norm?

36-54

Regular insulin duration?

4 hours

Of which of the following symptoms might an older woman with diabetes mellitus complain? wps.prenhall.com 1) anorexia 2)pain intolerance 3) weight loss 4) perineal itching

4) perineal itchingRationale: Older women might complain of perineal itching due to vaginal candidiasis.

Match the breath sound with the appropriate description. 1) High-pitched sound heard on inspiration in infants 2) High-pitched, continuous musical sound 3) High-pitched popping or low-pitched bubbling sounds 4) Low-pitched continuous sounds that clear with coughing 5) Labored, snoring sound a. Crackles b. Rhonchi c. Stridor d. Wheezes e. Stertor

1. c. High-pitched sound heard on inspiration in infants: Stridor 2. d. High-pitched, continuous musical sound: Wheezes 3. a. High-pitched popping or low-pitched bubbling sounds: Crackles 4. b. Low-pitched continuous sounds that clear with coughing: Rhonchi 5. e. Labored, snoring sound: Stertor

Proper order for removing PPE?

1. gloves 2. goggles 3. gown 4. mask

Proper order for donning PPE?

1. gown 2. mask 3. goggles 4. gloves

RBC norm?

4-6

Describe a 2 point gait?

1. one crutch and opposite foot together 2. Other crutch and other foor together. 2 points 2gether and the same time.

Moderate lochia is?

4-6 inches on pad in one hour

triglycerides

100-200

High fetal heart rate?

160 This is ok

HCO3

19-25

Antabuse onset and duration is?

2 weeks

A nurse is preparing a plan of care for a client with diabetes mellitus who has hyerglycemia. The priority nursing diagnosis would be: 1. Deficient knowledge 2. Deficient fluid volume 3. Compromised family coping 4. Imbalanced nutrition less than body requirements

2) deficient fluid volumeAn increased blood glucose level will cause the kidneys to excrete the glucose in the urine. This glucose is accompanied by fluids and electrolytes, causing an osmotic diuresis leading to dehydration. This fluid loss must be replaced when it becomes severe.

Crutches should be how many fingerwidths below the armpit?

2-3

Which of the following statements made by a nurse requires follow-up with additional instruction regarding the personal nature of pain? 1. "I have experienced pain before, and so I have great compassion for anyone dealing with pain." 2. "My postsurgical clients get the prescribed pain medications on schedule with no diversion from that schedule." 3. "If I were experiencing severe pain, I certainly would want someone to devote their time to managing for me." 4. "Clients don't always request pain medication, and so I always ask them if they want it according to the schedule."

2. "My postsurgical clients get the prescribed pain medications on schedule with no diversion from the schedule."

Hypothyroidism S/S?

Same as hypometabolism ie, tired, sluggish etc

ANS: B, D, A, C The first action should be to administer the antibiotic because treating the infection that has caused the fever is the most important aspect of fever management. The next priority is to lower the high fever, so the nurse should administer acetaminophen to lower the temperature set point. A cool sponge bath should be done after the acetaminophen is given to lower the temperature further. The wet-to-dry dressing change will not have an immediate impact on the infection or fever and should be done last. DIF: Cognitive Level: Analysis REF: 190 | 199 OBJ: Special Questions: Alternate Item Format, Prioritization TOP: Nursing Process: Planning MSC: NCLEX: Physiological Integrity

2. A patient who has an infected abdominal wound develops a temperature of 104° F (40° C). All the following interventions are included in the patient's plan of care. In which order should the nurse perform the following actions? Put a comma and space between each answer choice (a, b, c, d, etc.) ____________________ a. Sponge patient with cool water. b. Administer intravenous antibiotics. c. Perform wet-to-dry dressing change. d. Administer acetaminophen (Tylenol).

ACE inhibitor side effects: CAPTOPRIL

C: cough A: angioedema/agranulocystosis P: potassium excess T: taste changes O: orthostatic hypotension P: pregnancy contraindicated R: renal artery stenosis contraindicated I: inflammation related pain L: lower GFR

Excessive lochia is?

Saturated pad in 15 minutes

"The client diagnosed with type 1 diabetse is receiving Humalog, a rapid-acting insulin, by sliding scale. The order reads blood glucose level: <150, zero (0) units; 151 to 200, three (3) units; 201 to 250, six (6 units); >251, contact health-care provider. The unlicensed assistive personnel (UAP) reports to thenurse the client's glucometer reading is 189. How much insulin should the nurse administer to the client?

3 unitsThe client's result is 189, which is between 151 and 200, so the nurse should administer 3 units of Humalog insulin subcutaneously.

A nonpharmacological approach that the nurse may implement for clients experiencing pain that focuses on promoting pleasurable and meaningful stimuli is: 1. Acupressure 2. Distraction 3. Biofeedback 4. Hypnosis

2. Distraction

The client is experiencing breakthrough pain while receiving opioids. An order is written for the client to receive a transmucosal fentanyl "unit." In teaching about this medication, the nurse should instruct the client to: 1. Swab the unit over the cheeks 2. Do not chew the unit after administration 3. Take no more than two units per episode of discomfort 4. Allow the unit to dissolve slowly in the mouth over 15 minutes or more

2. Do not chew the unit after administration

The nurse recognizes that the most likely reason a runner who has injured his ankle during a race is not aware of it until after he crosses the finish line is that: 1. The emotional exhilaration of running the race masked the pain of the injury 2. His endorphin levels were high as a result of the physical stressors of the race 3. He was mentally distracted by the need to concentrate on the ever-changing nature of the race 4. The physical effects of the injury slowly increased during the race and reached pain-producing capacity only after the race

2. His endorphin levels were high as a result of the physical stressors of the race

A client who ruptured his spleen in a motor vehicle accident rates his postoperative pain as a level 8 on a 0 to 10 pain scale. After administering pain medication, the nurse discusses the use of complementary therapies with the client to explore ways to reduce the pain. The client would like to try a massage. The nurse delegates this task to the assistive personnel (AP). Which of the following instructions is most important for the nurse to share with the AP? 1. "You need to warm the bottle of lotion before using it." 2. "Report any changes in the client's skin condition to me immediately." 3. "Do not massage the client's legs." 4. "Massage each body part at least 10 minutes."

3. "So not massage the client's legs."

While on Lithium you must monitor the level as well as what?

Sodium

A client with chronic back pain has an order for a transcutaneous electrical nerve stimulation (TENS) unit for pain control. The nurse should instruct the client to: 1. Keep the unit on high 2. Use the unit when pain is perceived 3. Remove the electrodes at bedtime 4. Use the therapy without medications

2. Use the unit when pain is perceived

What is Hegars sign?

Softening movesd from the cervix to the uterus.

"The nurse is discharging a client diagnosed with diabetes insipidus. Which statementmade by the client warrants further intervention? "1."I will keep a list of my medications in my wallet and wear a Medi bracelet." 2."I should take my medication in the morning and leave it refrigerated at home." 3."I should weigh myself every morning and record any weight gain." 4."If I develop a tightness in my chest, I will call my health-care provider."

2."I should take my medication in the morning and leave it refrigerated at home.""1.The client should keep a list of medication being taken and wear a Medic Alert bracelet. 2. Medication taken for DI is usually every 8-12 hours, depending on the client. Theclient should keep the medication close at hand. 3.The client is at risk for fluid shifts. Weighing every morning allows the client to follow thefluid shifts. Weight gain could indicate too much medication. 4.Tightness in the chest could be an indicator that the medication is not being tolerated; if this occurs the client should call the health-care provider"

The home care nurse notes that a 67-year-old female diabetic client's blood glucose level has been elevated since she strained her back the previous week. The client states that she cannot understand why her blood glucose level is elevated. The nurse suspects the most likely cause for the elevated blood sugar is: 1. The decreased activity level of the client since the injury 2. Parasympathetic stimulation from the body's normal response to pain 3. The client is consuming more food as a comfort measure 4. The client may not be taking her medication as ordered

2.Parasympathetic stimulation from the body's normal response to pain

The fundus is palpable at the naval at week?

20-22

Bicarb (HCO3) norm?

22-26

The nurse is caring for a client with long-term Type 2 diabetes and is assessing the feet. Which assessment data would warrant immediate intervention by the nurse? "1)The client has crumbling toenails 2)The client has athlete's feet 3)The client has a necrotic big toe 4)The client has thickened toenails."

3) Nectrotic big toe"1)Crumbling toenails indicate tinea unguium, which is a fungus infection of the toenail. 2)Athlete's foot is a fungal infection that is not life threatening. 3)A necrotic big toe indicates "dead" tissue. The client does not feel pain in the lower extremity and does not realize there has been an injury and therefore does not seek treatment. Increased blood glucose levels decrease oxygen supply that is needed to heal the wound and increase the risk for developing an infection. 4)Big, thick toenails are fungal infections and would not require immediate intervention by the nurse; 50% of the adult population has this."

"A nurse performs a physical assessment on a client with type 2 diabetes mellitus. Findings include a fasting blood glucose of 120 mg/dL, temp of 101 F, pulse of 88 bpm, respirations of 22, and blood pressure of 100/72. Which finding would be of most concern to the nurse? "1. Pulse 2. Respiration 3. Temperature 4. Blood pressure"

3) temp. An elevated temperature may indicate infection. Infection is a leading cause of hyperglycemic hyperosmolar nonketotic syndrome or diabetic ketoacidosis. The other findings noted in the question are within normal limits.

How long does it take for Coumadin to work?

3-5 days

What is the length for hand scrubbing?

3-7 minutes

ANS: D Common clinical manifestations of inflammation and infection are frequently not present when patients receive immunosuppressive medications. The earliest manifestation of an infection may be "just not feeling well."

7. Which nursing action is most likely to detect early signs of infection in a patient who is taking immunosuppressive medications? a. Monitor white blood cell count. b. Check the skin for areas of redness. c. Check the temperature every 2 hours. d. Ask about fatigue or feelings of malaise.

pH

7.35-7.45

pH norm?

7.35-7.45

Which of the following statements made by a client reporting severe pain expresses the most insight into how pain impacts a client's energy reserves? 1. "I can't sleep if I don't get something for this pain." 2. "If only I could get an hour when I was free of this pain." 3. "I'm exhausted physically and emotionally trying to live with this pain." 4. "I don't see how I can continue to cope with this pain; I need some relief."

3. "I'm exhausted physically and emotionally trying to live with this pain."

ANS: C The patient's complaints of feeling cold and shivering indicate that the hypothalamic set point for temperature has been increased and the temperature is increasing. Because associated peripheral vasoconstriction and sympathetic nervous system stimulation will occur, skin flushing and hypotension are not expected. Muscle cramps are not expected with chills and shivering or with rising temperatures.

3. A patient with a systemic bacterial infection has "goose pimples," feels cold, and has a shaking chill. At this stage of the febrile response, the nurse will plan to monitor for a. skin flushing. b. muscle cramps. c. rising body temperature. d. decreasing blood pressure.

An older client with mild musculoskeletal pain is being seen by the primary care provider. The nurse anticipates that treatment of this client's level of discomfort will include: 1. Fentanyl 2. Diazepam 3. Acetaminophen 4. Meperidine hydrochloride

3. Acetaminophen

A client who is scheduled for the second in a series of painful dressing changes asks for "my pain medication now so it's working when the dressing is changed" is most likely expressing: 1. A great fear of the expected pain 2. A need to be in control of his pain 3. An understanding that it is easier to prevent the pain than to stop the pain 4. An acceptance of the pain that the dressing change will obviously cause him

3. An understanding that it is easier to prevent the pain than to stop the pain

Before inserting a Foley catheter, the nurse explains that the client may feel some discomfort. This is an example of: 1. Distraction 2. Reducing pain perception 3. Anticipatory response 4. Self-care maintenance

3. Anticipatory response

The nurse should describe pain that is causing the client a "burning sensation in the epigastric region" as: 1. Referred 2. Radiating 3. Deep or visceral 4. Superficial or cutaneous

3. Deep or visceral

Which of the following symptoms would the nurse expect with a client who is experiencing acute pain? 1. Bradycardia 2. Bradypnea 3. Diaphoresis 4. Decreased muscle tension

3. Diaphoresis

A priority nursing intervention when caring for a client who is receiving an epidural infusion for pain relief is to: 1. Use aseptic technique 2. Label the port as an epidural catheter 3. Monitor vital signs every 15 minutes 4. Avoid supplemental doses of sedatives

3. Monitor vital signs every 15 minutes

Which of the following is most appropriate when the nurse assesses the intensity of the client's pain? 1. Ask about what precipitates the pain. 2. Question the client about the location of the pain. 3. Offer the client a pain scale to objectify the information. 4. Use open-ended questions to find out about the sensation.

3. Offer the client a pain scale to objectify the information

Detached Retina: Care

Care Includes: Area of detachment should be in the dependent position.

Shock: Care

Care Includes: Bed-rest w/ extremities elevated 20 degrees, knees straight, head slightly elevated (modified trendelenberg)

After Total Hip Replacement: Care

Care Includes: Don't sleep on the operated side; don't flex hip for more than 45 - 60 degrees, don't elevate HOB more than 45 degrees. Also maintain hip *ABDUCTION* by separating thighs with pillows.

After A Supratentorial Surgery (Incision Behind Hairline): Care

Care Includes: Elevate HOB 30-45 degrees

Above the Knee Amputation: Care

Care Includes: Elevate for the first 24 hours on a pillow, position prone daily to provide for hip extension.

Head Injury: Care

Care Includes: Elevate the HOB 30 degrees to decrease ICP.

Buck's Traction (Skin Traction): Care

Care Includes: Elevate the foot of the bed for counter-traction.

Below Knee Amputation: Care

Care Includes: Foot of the bed elevated for the first 24 hours, position prone daily to provide for hip extension.

Prolapsed Cord: Care

Care Includes: Knee to Chest position OR Trendelenburg

Heat Stroke: Care

Care Includes: Lie *FLAT* w/ legs *Elevated*

During Internal Radiation: Care

Care Includes: On bedrest while implant is in place.

A client with a history of chronic back pain is questioning the need to "keep asking for pain medication," fearing that he will be viewed as being weak by his family. The most therapeutic nursing response to this client would be: 1. "Chronic back pain is very difficult to deal with; utilize the pain medication because that's what it's there for." 2. "Your family won't think you're weak; they want you to be comfortable, and the medication will help." 3. "Taking the medication as prescribed will help you to be more active; your family will be happy you can do things with them again." 4. "It's important that you manage your pain as effectively as possible; it really doesn't matter what other people think about you."

3."Taking the medication as prescribed will help you to be more active; your family will be happy you can do things with them again."

potassium (k)

3.5-5

Potassium norm?

3.5-5.3

serum albumin

3.5-5.5

Humalog peak?

30 minutes

When do you draw a trough level?

30 minutes before the next scheduled dose.

When the handgrip of a crutch is properly in place the elbow felxion should be?

30*

Which of the following statements made by a nurse shows the greatest understanding of the personal nature of the pain experience? 1. "I have experienced pain before, and so I have great compassion for anyone dealing with pain." 2. "People handle pain differently, but everyone in pain is only interested in having the pain stop." 3. "Managing a client's pain is the single most important thing a nurse can do for a client experiencing pain." 4. "I can only accept what the client reports concerning the pain being felt and attempt to intervene successfully in its management."

4. " I can only accept what the client reports concerning the pain being felt and attempt to intervene successfully in its management."

Lumbar Puncture: Care After Procedure

Care Includes: Place the client in the supine position for 4 to 12 hours as prescribed.

ANS: D Mild to moderate temperature elevations (less than 103° F) do not harm the young adult patient and may benefit host defense mechanisms. The nurse should continue to monitor the temperature. Antipyretics are not indicated unless the patient is complaining of fever-related symptoms. There is no need to notify the patient's health care provider or to use a cooling blanket for a moderate temperature elevation.

4. A 24-year-old patient who is receiving antibiotics for an infected leg wound has a temperature of 101.8° F (38.7° C). Which action by the nurse is most appropriate? a. Apply a cooling blanket. b. Notify the health care provider. c. Give the prescribed PRN aspirin (Ascriptin) 650 mg. d. Check the patient's oral temperature again in 4 hours.

After an Myringotomy: Care

Care Includes: Position patient on the side of the *affected* ear after surgery. (This allows drainage of secretions)

Tube Feeding w/ DECREASED LOC: Care

Care Includes: Position pt on *right side* (this promotes emptying of the stomach) with the HOB *elevated* (to prevent aspiration)

After A Infratentorial Surgery (Incision at the nape of the Neck): Care

Care Includes: Position the patient *FLAT* and lateral on either side.

The nurse is attempting to ambulate a postoperative client who continues to rate his pain as a 7 on a scale of 0 to 10, with 10 being the most severe. The client is reluctant to walk and consents to move only to the chair, reporting that "it hurts too much to walk." The nurse's primary concern regarding the client's recovery related to his pain experience is that: 1. His pain medications are not effectively managing his pain 2. He does not fully understand the importance of ambulation 3. He is expending too much of his energy dealing with the pain 4. He is not ready to participate in the activities needed to recover quickly

4. He is not ready to participate in the activities needed to recover quickly

The nurse inquires of a postoperative client as to the need for pain medication. The client denies the need then but 30 minutes later reports, "I am really in a lot of pain. Can you bring me my pain pill now?" The nurse recognizes that the most immediate need for client education is related to explaining that: 1. His oral medication will take approximately 30 minutes to affect his pain 2. There may be a need to administer his pain medication via the intravenous route 3. Pain medication is more effective if blood levels are maintained at a constant level 4. His pain will be more effectively managed if he reports a need for pain medication while the pain is still tolerable

4. His pain will be more effectively managed if he reports a need for pain medication while the pain is still tolerable

After Cataract Surgery: Care

Care Includes: The patient will sleep on the *unaffected* side with a night shield for 1 - 4 weeks.

A client is having severe, continuous discomfort from kidney stones. Based on the client's experience, the nurse anticipates which of the following findings in the client's assessment? 1. Tachycardia 2. Diaphoresis 3. Pupil dilation 4. Nausea and vomiting

4. Nausea and vomiting

Peritoneal Dialysis w/ Inadequate outflow: Care

Care Includes: Turn patient from side to side *BEFORE* checking for kinks in the tubing (According to Kaplan)

After a Lumbar Puncture (also an Oil Based Myelogram): Care

Care Includes: patient lies *FLAT/SUPINE* (This is done to prevent headache and leaking of CSF)

During Epidural Puncture: Care

Care Includes: position the patient *SIDE LYING*

Infant w/ Spina Bifidia: Care

Care includes: Position *prone* (on abdomen) so the sac does not rupture.

Never choose _______ as a food for children.

Casserol

n creating the plan of care for a newly diagnosed breast cancer client, the nurse is concerned about pain control. The client has expressed an interest in relaxation therapy as a complementary pain therapy. The nurse knows that the best time to teach the client is: 1. Immediately following the client's mastectomy 2. Before giving pain medication to evaluate if the complementary therapy works 3. Immediately preceding surgery 4. When the client is comfortable

4. When the client is comfortable

If Prozac is BID give at what times?

6A and 12N

BUN

7-18

The nurse is caring for a cognitively impaired client who has experienced a painful procedure. The nurse is most effective in determining the client's pain medication needs when using which of the following assessment methods? 1. Medicating the client with the as-needed (prn) analgesic as often as ordered 2. Utilizing the pain face scale to assess the client's pain experience 3. Asking the client to rate his or her pain on a scale of 1 to 10, with 10 being the most severe pain 4. Observing the client's body movements and facial expressions for typical pain behaviors

4. observing the client's body movements and facial expressions for typical pain behavior

25. The nurse recognizes that a client who is inactive is at a risk for decreased muscle mass as a result of increased muscle atrophy and: 1. Decrease metabolic rate 2. Catabolic tissue breakdown 3. Inactivity-induced depression 4. Anorexia caused by decreased peristalsis

Catabolic tissue breakdown

a nurse is interviewing a client with type 2 diabetes mellitus. which statement by the client indicated an understanding of the treatment for this disorder? "1. ""i take oral insulin instead of shots"" 2. ""by taking these medications I am able to eat more"" 3. ""when I become ill, I need to increase the number of pills I take"" 4. ""the medications I'm taking help release the insulin I already make""

4.)Clients with type 2 diabetes mellitus have decreased or imparied insulin secretion. Oral hypoglycemic agents are given to these clients to facilitate glucose uptake. Insulin injections may be given during times of stress-induced hyperglycemia. Oral insulin is not available because of the breakdown of the insulin by digestion. Options 1, 2 and 3 are incorrect

calcium (Ca)

4.5-5

What is the average dose of Haldol?

5 mg

WBC ct

5,000-10,000

WBC norm?

5,000-11,000

When do you draw a sublingual peak level?

5-10 minutes after it is dissolved.

Active phase is?

5-7 cm dilated Contractions are 3-5 min apart Lasting 30-60 sec. They are moderate

alkaline phosphatase: liver and bone function

50-120

Absolute neutrophil count (ANC) norm?

500

Humulin 70/30 is what?

70% N insulin and 30% R insulin

Empathy question have?

A quote in the question and a quote in the answer.

Clozaril(Clozapine) is what?

A second generation tranquilizer

16. Which of the following clients is most at risk for losing his or her balance? 1. A woman who is 9 months pregnant walking down a flight of stairs 2. A 16-year-old skate boarding down a 15-degree slope 3. A 45-year-old taking hypertensive medication 4. A 4-year-old riding a tricycle

A woman who is 9 months pregnant walking down a flight of stairs

1. A client has been on prolonged bed rest, and the nurse is observing for signs associated with immobility. In assessment of the client, the nurse is alert to a(n): 1. Increased blood pressure 2. Decreased heart rate 3. Increased urinary output 4. Decreased peristalsis

Decreased peristalsis

The client diagnosed with Type I diabetes is found lying unconscious on the floor of the bathroom. Which interventions should the nurse implement first? A. Administer 50% dextrose IVP. B. Notify the health-care provider. C. Move the client to ICD. D. Check the serum glucose level.

A) admin 50% dextrose IVPThe nurse should assume the client is hypoglycemic and administer IVP dextrose, which will rouse the client immediately. If the collapse is the result of hyperglycemia, this additional dextrose will not further injure the client.

"When assessing the patient experiencing the onset of symptoms of type 1 diabetes, which question should the nurse ask? a. ""Have you lost any weight lately?"" b. ""Do you crave fluids containing sugar?"" c. ""How long have you felt anorexic?"" d. ""Is your urine unusually dark-colored?""

A) lost any weight?"a. Weight loss occurs because the body is no longer able to absorb glucose and starts to break down protein and fat for energy. b. The patient is thirsty but does not necessarily crave sugar- containing fluids. c. Increased appetite is a classic symptom of type 1 diabetes. d. With the classic symptom of polyuria, urine will be very dilute."

Beta Blocker Receptor Selectivity: think alphabetical A-N vs O-Z

A-N = beta 1 selective (acebutalol, atenolol, esmolol, metropolol) O-Z = B1/B2 nonselective (propranalol, sotalol, timolol) (note carvedilol and labetalol have nonselective beta and alpha activity)

The SE for phenothiazines are? (ABCDEFG)

A-nticholinergic SE B-lurred vision C-onstipation D-rowsiness E-xtra paramital syndrome F-otosensitivity aG-granulocytosis

Tricyclic SE? (Elavil starts with E so they go through E ABCDE)

A-nticholinergic effects B-lured vision C-onstipatioh D-rowsiness E-uphoria

Benzo SE? (ABCD)

A-nticholinergic effects B-lured vision C-onstipation D-rowsiness

MAOI SE?

A-nticholinergic effects B-lured vision C-onstipation Drowsiness

A client's vital signs at the beginning of the shift are as follows: oral temperature 99.3°F (37°C), heart rate 82 beats per minute, respiratory rate 14 breaths per minute, and blood pressure 118/76. Four hours later the client's oral temperature is 102.2°F (39°C). Based on the temperature change, the nurse should anticipate the client's heart rate would be how many beats per minute? Why?

ANS: 111 BPM. ∆ 1 °F = 10 BPM 102.2 °F - 99.3 °F = 2.9 °F * (10 BPM / 1 °F) = ∆ 29 BPM + 82 BPM = 111 BPM

An external insulin pump is prescribed for a client with DM. The client asks the nurse about the functioning of the pump. The nurse bases the response on the information that the pump: "a. Gives small continuous dose of regular insulin subcutaneously, and the client can self-administer a bolus with an additional dosage from the pump before each meal. b. Is timed to release programmed doses of regular or NPH insulin into the bloodstream at specific intervals. c. Is surgically attached to the pancreas and infuses regular insulin into the pancreas, which in turn releases the insulin into the bloodstream. d. Continuously infuses small amounts of NPH insulin into the bloodstream while regularly monitoring blood glucose levels"

ANSWER A. An insulin pump provides a small continuous dose of regular insulin subcutaneously throughout the day and night, and the client can self-administer a bolus with additional dosage from the pump before each meal as needed. Regular insulin is used in an insulin pump. An external pump is not attached surgically to the pancreas.

How quickly does Heparin work?

ASAP

Tranquilizers work?

ASAP

Supra means?

Above

If the pH is down it is?

Acidosis

What do you do if a patient with psychosis of dementia is having a hallucination or illusion?

Acknowledge and Redirect.

If a functional psychotic is having a delusion or illusion you?

Acknowledge the feeling ( that sound horrible), present reality (but we have no spiders in the room), set a limit ( were not going to talk about that lets talk about something else), enforce the limit ( I see you're to ill to talk about reality).

APGAR: A (2nd A)

Activity - flexed/flaccid/limp

S/S of dumping syndrome are?

Acute abdominal distress= gas, ^bs, cramping, bloat Dumping= s/s of being drunk Syndrome= s/s shock

What are the four rules of prioritization?

Acute beats chronic Fresh post-op(12hr.) beats medical or other surgical. Unstable beats stable The more vital the organ the higher the priority(use only as a tie breaker)

What makes a patient unstable? (7)

Acute illness Post-op less than 12 hr General anesthesia Changed assessment Phrase "newly admitted" or "newly diagnosed" Labs C & D Unexpected S/S

Cushings Syndrome treatment?

Adrenalectomy

To treat supraventricular arrythmias you give?

Adenocard, Beta-blocker (end in "lol'), Calcium Channel Blocker and Digitalis or Lonoxin.

A nurse is assisting in developing a plan of care for a client with acquired immunodeficiency syndrome (AIDS) who is experiencing night fever and night sweats. Which nursing intervention should the nurse suggest including in the plan of care to manage this symptom? Keep the call bell within reach for the client. Administer a sedative at bedtime. Administer an antipyretic at bedtime. Provide a back rub and comfort measures before bedtime.

Administer an antipyretic at bedtime.

Primary symptom of Wernickes?

Amnesia with confabulation (making up stories).

Which client is at the highest risk for systemic lupus erythematous (SLE)? An Asian male A white female An African-American male An African-American female

An African-American female

Prozac is?

An SSRI

Piagets formal operations stage can be taught like?

An adult

A client with pemphigus is being seen in the clinic regularly. The nurse plans care based on which of the following descriptions of this condition? The presence of tiny red vesicles An autoimmune disease that causes blistering in the epidermis The presence of skin vesicles found along the nerve caused by a virus The presence of red, raised papules and large plaques covered by silvery scales

An autoimmune disease that causes blistering in the epidermis

The nurse is assigned to care for a client with systemic lupus erythematosus (SLE). The nurse plans care knowing that this disorder is: A local rash that occurs as a result of allergy A disease caused by overexposure to sunlight An inflammatory disease of collagen contained in connective tissue A disease caused by the continuous release of histamine in the body

An inflammatory disease of collagen contained in connective tissue

"1. A patient with newly diagnosed type 2 diabetes mellitus asks the nurse what ""type 2"" means in relation to diabetes. The nurse explains to the patient that type 2 diabetes differs from type 1 diabetes primarily in that with type 2 diabetes a. the pt is totally dependent on an outside source of insulin b. there is a decreased insulin secretion and cellular resistance to insulin that is produced c. the immune system destroys the pancreatic insulin-producing cells d. the insulin precurosr that is secreted by the pancreas is not activated by the liver

Answer B - Rationale: In type 2 diabetes, the pancreas produces insulin, but the insulin is insufficient for the body's needs or the cells do not respond to the insulin appropriately. The other information describes the physiology of type 1 diabetes

"The nurse caring for a 54-year-old patient hospitalized with diabetes mellitus would look for which of the following laboratory test results to obtain information on the patient's past glucose control? a. prealbumin level b. urine ketone level c. fasting glucose level d. glycosylated hemoglobin level

Answer d: A glycosylated hemoglobin level detects the amount of glucose that is bound to red blood cells (RBCs). When circulating glucose levels are high, glucose attaches to the RBCs and remains there for the life of the blood cell, which is approximately 120 days. Thus the test can give an indication of glycemic control over approximately 2 to 3 months.

High blood sugar in type II diabetes aka hyperosmolar, hyperglycemic, non-ketotic coma (HHNK) is the same as?

Dehydration

If the BUN is elevated cheack for?

Dehydration

"A client is taking Humulin NPH insulin daily every morning. The nurse instructs the client that the mostlikely time for a hypoglycemic reaction to occur is: A) 2-4 hours after administration B) 4-12 hours after administration C) 16-18 hours after administration D) 18-24 hours after administration

B: Rationale: Humulin is an intermediate acting insulin. The onset of action is 1.5 hours, it peaks in 4-12 hours, and its duration is 24 hours. Hypoglycemic reactions to insulin are most likely to occur during the peak time.

Large doses of phenothiazines are ?

Anti-psychotic

Hgb under 8 you?

Assess bleeding Prepare blood Call doctor

"P" refers to?

Atrail

To treat AsystolE you give?

Atropine and Epinephrine but give it in reverse.

Five types of hallucinations?

Auditory, visual, tactile, olfactory and gustatory.

Interventions for Allergic Rhinitis

Avoidance therapy: Environmental changes Pet interventions

SSRI side effects: BAD SSRI

B: body wt increase A: anxiety D: dizziness S: serotonin syndrome S: stimulated CNS R: reproductive (sexual) dysfunction I: insomnia

Calcemias do the opposite of the prefix plus all what?

BP changes

A multigavid client in labor at 38 weeks' gestation has been diagnosed with Rh sensitization and probably fetal hydrops and anemia. When the nurse observes the fetal heart rate pattern on the monitor, which of the following patterns is most likely? a. early deceleration pattern b. sinusoidal pattern c. variable deceleration pattern d. late deceleration pattern

B - The fetal heart rate of a multipara diagnosed with Rh sensitization and probably fetal hydrops and anemia will most likely demonstrate a sinusoidal pattern that resembles a sine wave. It has been hypothesized that this pattern reflects an absence of autonomic nervous control over the fetal heart rate resulting from severe hypoxia. This client will most likely requires a cesarean delivery to improve the fetal outcome. Early decelerations are associated with cord compression; and late deceleration are associated with poor placental perfusion

Magnesemias do the opposite of the prefix plus all what?

BP changes

What is Trousseaus sign?

BP cuff inflated and causes a carpal spasm.

Late decelerations?

Bad (placenta) you LION

Low baseline variability? (aka heart rate not changing)

Bad you LION

If you have a lab thats high and you don't know why pick?

Dehydration

Why is manipulation easier to treat then dependency/codependency?

Because no one likes being manipulated.

When should you give Prozac?

Before 12 noon.

When do you wash your hands?

Before and after gloves, entering and exiting a room and after you soil your hands.

You can use alcohol based solution when?

Before and after gloves, entering and leaving a room

How do you assess frequency of contractions?

Beginning of one contraction to the beginning of another.

How do you assess duration of a contraction?

Beginning to end of contraction

Mongolian spots are?

Bluish/black macules appearing over the buttocks and or thighs of darker skinned neonates.

Type II: Cytotoxic Reactions

Body makes special autoantibodies directed against self cells that have some form of foreign protein attached to them Reaction of IgG with host cell membrane or antigen adsorbed by host cell membrane Ex. Autoimmune hemolytic anemia, Goodpasture's syndrome, Myasthenia gravis

Neomycin and Kanmycin are used for what?

Bowel sterilzation?

What is the biggest danger with low blood sugar?

Brain damage if its less then 15

What 5 words should you avoid when answering a question about toys for a child under 9 months old?

Build Make Construct Sort Stack

Heparin is what pregnancy class?

C

A 40-year-old executive who was unexpectedly laid off from work 2 days earlier complains of fatigue and an inability to cope. He admits drinking excessively over the previous 48 hours. This behavior is an example of: a. alcoholism b. manic episode c. situational crisis d. depression

C - A situational crisis results from a specific event in the life of a person who is overhwlemed by the situation and reacts emotional. Fatigue, insomnia, and inability to make decisions are common signs and symtpoms. The situations crisis may precipitate behavior that causes a criss (alcohol or drug abuse). There isn't enough information to label this client an alcoholic. A manic episodes is characterized by euphoria and labile effect. Symptoms of depression are usually present for 2 or more weeks.

"A diabetic patient has a serum glucose level of 824 mg/dL (45.7 mmol/L) and is unresponsive. Following assessment of the patient, the nurse suspects diabetic ketoacidosis rather than hyperosmolar hyperglycemic syndrome based on the finding of "a. polyuria b. severe dehydration c. rapid, deep respirations ) d. decreased serum potassium"

C is correct, Signs and symptoms of DKA include manifestations of dehydration such as poor skin turgor, dry mucous membranes, tachycardia, and orthostatic hypotension. Early symptoms may include lethargy and weakness. As the patient becomes severely dehydrated, the skin becomes dry and loose, and the eyeballs become soft and sunken. Abdominal pain is another symptom of DKA that may be accompanied by anorexia and vomiting. Kussmaul respirations (i.e., rapid, deep breathing associated with dyspnea) are the body's attempt to reverse metabolic acidosis through the exhalation of excess carbon dioxide. Acetone is identified on the breath as a sweet, fruity odor. Laboratory findings include a blood glucose level greater than 250 mg/dL, arterial blood pH less than 7.30, serum bicarbonate level less than 15 mEq/L, and moderate to large ketone levels in the urine or blood ketones.

After 72 hours of alochol withdrawal a small minority may get?

Delirium Tremens

So what do you do and in what order for the second stage?

Deliver the head Suction 1st the mouth then the nose Check for nuchle cord (cord around neck) Deliver shoulders and body ID band

In educating a client with diabetes, what response would reveal need for further education? "A. I should avoid tights B. I should take good care of my toe nails C. I should not go more than 3 days without washing my feet D. I should avoid going barefoot and should wear clean socks

C)I should not go more than 3 days w/o washing my feet"The recommended self-care routine is to wash feet on a daily basis without soaking and carefully cleaning."

A patient is admitted with diabetes mellitus, has a glucose level of 380 mg/dl, and a moderate level of ketones in the urine. As the nurse assesses for signs of ketoacidosis, which of the following respiratory patterns would the nurse expect to find?"A-Central apnea B-Hypoventilation C-Kussmaul respirations D- Cheyne-Stokes respirations"

C-Kussmaul respirationsIn diabetic ketoacidosis, the lungs try to compensate for the acidosis by blowing off volatile acids and carbon dioxide. This leads to a pattern of Kussmaul respirations, which are deep and nonlabored.

"A nurse is caring for a client admitted to the emergency department with diabetic ketoacidosis (DKA). In the acute phase, the priority nursing action is to prepare to: "A. Correct the acidosis B. Administer 5% dextrose intravenously C. Administer regular insulin inraVenously D. Apply a monitor for an electrocardiogram."

C. Administer regular insulin inraVenously Lack (absolute or relative) of insulin is the primary cause of DKA. Treatment consists of insulin administration (regular insulin), intravenous fluid administration (normal saline initially), and potassium replacement, followed by correcting acidosis. Applying an electrocardiogram monitor is not a priority action.

Limited Scleroderma: CREST syndrome

C: calcinosis R: raynaud's syndrome E: esophageal motility disorder S: sclerodactyly T: telangiectasia

Multiple Myeloma Symptoms: CRAB

C: calcium level elevated R: renal failure A: anemia B: bone breakdown and resorption

Canes and walkers: COAL WWAL

C: cane O: opposite A: affected L: leg W: walker W: with A: affected L: leg

Tertiary Syphilis: CLASS

C: cardiovascular disorder L: late benign syphilis (gumma) A: asymptomatic neurosyphilis S: symptomatic neruosyphilis S: seizures and apathy (meningeal involvement)

Side effects of corticosteroids: CUSHINGOID

C: cataracts U: ulcers S: skin thinning, bruising and straiae H: hyperglycemia, hypertension, hirsuitism I: infections N: necrosis of the femoral head G: glycosuria O: osteoporosis, obesity I: immunosuppression D: diabetes

Stages of Shock: CPR

C: compensatory stage P: progressive stage R: refractory stage

Secondary Syphilis: CAMP

C: condyloma lata A: acute infection symptoms (fever, sore throat, malaise) M: mucocutaneous lesion, mucous patches P: papules & pustules

Acute Lithium tox symptoms: CAN HAM SUCS

C: confusion A: an increase in urine and thirst N: nausea H: hand tremors A: ataxia (incoordination of arms and legs) M: muscle twitches S: seizures U: uncontrollable eye movement C: coma S: slurred speech

ANF and BNP=

CHF

BNP is a good indicator of what?

CHF

Lanoxin(digoxin) is for?

CHF, Atrial arrhythmia

Benzodiazepine - NC

CNS depressant also used as muscle relaxants, sedatives, hypnotics, anticonvulsants, seizures, alcohol withdrawal check renal and hepatic function don't discontinue abruptly potential for tolerance or abuse

O2 Dissociation Right shift: right shift is DATE w/ CO2

CO2 D: 2,3 Diphosphoglycerate DPG A: acidosis T: temp E: exercise

What are the five uppers?

Caffeine, Cocaine, Methamphetamines, PCP/LSD and ADHD Meds

What 2 newborn variations are you most likely to be tested on for the NCLEX?

Cephalohematoma and Caput Succedaneum

What is Chadwicks sign?

Cervical change to cyanosis (blue hue)

What is Goodells sign?

Cervical softening

Sodium is ok if abnormal unless what occurs?

Change in LOC

A-Fib is?

Chaotic QRS depolarizations

V-fib is?

Chaotic QRS depolarizations

N/I for restraints?

Check Q15min. and rotate sites Q2H

ACE of spades answer for OB?

Check fetal HR

If platelets are below 90,000 you?

Check for bleeding Place on bleeding precautions (thrombolytic precautions)

What makes a patient stable? (7)

Chronic illness Over 12 hr post op Local or regional anesthesia Unchanged assessment Phrase "to be discharged" Lab values A or B Typical S/S for the disease they have or what they are receiving treatment for.

Addisons Disease treatment?

Chronic steroids (glucocorticoids)

What 4 things do you do if the water seal breaks on a chest tube?

Clamp it 1st!! Cut broken device off of tube, put the tube in water (NS), unclamp.

Dry yours hands from...?

Cleanest to least clean

4 complications of hypertension

Coronary artery disease Coronary rheumatic fever Congestive heart failure cardiovascular accident

A nurse is caring for a client with type 1 diabetes mellitus. Which client complaint would alert the nurse to the presence of a possible hypoglycemic reaction ? A. Tremors B. Anorexia C. Hot, Dry skin D. Muscle cramps

Correct Answer A Decreased blood glucose levels produce autonomic nervous system symptoms, which are manifested classsically as nervousness, irritability, and tremors. Option C is more likely to occur with hyperglycemia. Options B and D are unrealted to the signs of hyperglycemia

A client with DKA is being treated in the ED. What would the nurse suspect? 1. Comatose state 2. Decreased Urine Output 3. Increased respirations and an increase in pH. 4. Elevated blood glucose level and low plasma bicarbonate level.

Correct Answer: 4 Rationale: In DKA the arteriole pH is lower than 7.35, plasma bicarbonate is lower than 15 mEq/L, the blood glucose is higher than 250, and ketones are present in the blood and urine. The client would be experiencing polyuria and Kussmauls respirations would be present. A comatose state may occur if DKA is not treated.

The nurse is assigned to care for a client admitted to the hospital with a diagnosis of systemic lupus erythematosus (SLE). The nurse reviews the health care provider's prescriptions. Which of the following medications would the nurse expect to be prescribed? Antibiotic Antidiarrheal Corticosteroid Opioid analgesic

Corticosteroid

While assessing the fundus of a multiparous client on the first postpartum day, the nurse performs hand washing and dons clean gloves. Which of the following should the nurse do next? a. place the non-dominant hand above the symphysis pubis and the dominant hand at the umbilicus b. ask the client to assume a side-lying position with the knees flexed c. perform massage vigorously at the level of the umbilicus if the fundus feels boggy d. place the client on a bedpan in case the uterine palpation stimulates the client to void

D - The nurse should place the non-dominant hand above the symphysis pubis and the dominant hand at the umbilicus to palpate the fundus. This prevents the uterine inversion and trauma, which can be very painful to the client. The nurse should ask the client to assume a supine, not side-lying, position with the knees flexed. The fundus can be palpated in this position and the perineal pads can be evaluated for lochia amounts. The fundus should be massaged gently if the fundus feels boggy. Vigorous massaging may fatigue the uterus and cause it to become firm and then boggy again. The nurse should ask the client to void before fundal evaluation. A full bladder can cause discomfort to the client, the uterus to be deviated to one side, and postpartum hemorrhage.

The second stage of labor is?

Delivery of the baby.

DKA S/S?

D-ehydration K-etones in the urine and blood, Kussmaul Resp and K+ A-cidosis, Acetone breath, Anorexia d/t nausea

When administering an enema, list the following steps in the order in which they should be performed. Label the steps from 1 to 6, with 1 being the first step to perform. A. Document the results of the procedure. B. Assess the patient for cramping. C. Insert the tubing about 3 to 4 inches into the rectum. D. Lubricate the tip of the enema tubing generously. E. Raise the container to the correct height and instill the solution at a slow rate. F. Encourage the patient to hold the solution for 3 to 15 minutes, depending on the type of enema.

D. Lubricate the tip of the enema tubing generously. C. Insert the tubing about 3 to 4 inches into the rectum. E. Raise the container to the correct height and instill the solution at a slow rate. B. Assess the patient for cramping. F. Encourage the patient to hold the solution for 3 to 15 minutes, depending on the type of enema. A. Document the results of the procedure. You must lubricate the tip before inserting the tubing. You would then begin instilling the solution before assessing for cramping that the instillation might produce. Only after the solution is instilled would you ask the patient to hold the solution. The last action is to document the results of the procedure, after the procedure is finished.

The third stage of labor is?

Delivery of the placenta.

"A frail elderly patient with a diagnosis of type 2 diabetes mellitus has been ill with pneumonia. The cliet's intake has been very poor, and she is admitted to the hospital for observation and management as needed. What is the most likely problem with this patient? "A. Insulin resistance has developed. B. Diabetic ketoacidosis is occuring. C. Hypoglycemia unawareness is developing. D. Hyperglycemic hyperosmolar non-ketotic coma.

D.Illness, especially with the frail elderly patient whose appetite is poor, can result in dehydration and HHNC. Insulin resisitance is inidcated by a daily insulin requirement of 200 units or more. Diabetic ketoacidosis, an acute metabolic condition, usually is caused by absent or markedly decreased amounts of insulin.

To decrease K+ you give?

D5W with Insulin R (insulin pulls the K+ from the blood and pulls it in the cell with glucose. this buys time but doesnt solve the problem)

18. The benefits of using an insulin pump include all of the following except: "a. By continuously providing insulin they eliminate the need for injections of insulin b. They simplify management of blood sugar and often improve A1C c. They enable exercise without compensatory carbohydrate consumption d. They help with weight loss

D: Using an insulin pump has many advantages, including fewer dramatic swings in blood glucose levels, increased flexibility about diet, and improved accuracy of insulin doses and delivery; however, the use of an insulin pump has been associated with weight gain.

Hypertension interventions: DIURETIC

D: daily wt I: intake U: urine output R: response of BP E: electrolytes T: take pulses I: ischemic episodes (TIA) C: complications (4Cs)

E. Coli causes: DUNG

D: diarrea U: UTI N: neonatal meningitis G: gram negative sepsis

Drug Therapy for Allergic Rhinitis

Decongestants Antihistamines Corticosteroids Mast cell stabilizers Leukotriene antagonists Complementary and alternative therapy Desensitization therapy

If a drug has the word _____ behind it, it means long IM form for noncompliance?

Deconoate

If the pH is down my patient will show signs and symtoms of?

Decrease... like decreased output, bradycardia and constipation

34. The nurse caring for a 78-year-old male client recovering from hip replacement surgery is assessing for signs of improvement of the client's activity tolerance. The nurse determined a baseline for ongoing assessments by: 1. Determining how much time it takes the client to recover from an activity 2. Assessing how much the client can do at one time 3. Determining the level of pain experienced by the client during the activity 4. Asking the client how much the client feels like doing

Determining how much time it takes the client to recover from an activity

A nurse is assisting in developing a plan of care for a pregnant client with acquired immunodeficiency syndrome (AIDS). The nurse determines that which of the following is the priority concern for this client? Inability to care for self at home Development of an infection Lack of available support services Isolation

Development of an infection

Study Mode Question 31 of 64 The nurse is assessing a client who has small groups of vesicles over his chest and upper abdominal area. They are located only on the right side of his body. The client states his pain level is 8/10, and describes the pain as burning in nature. Which question is most appropriate to include in the data collection? Did you have chicken pox as a child?" "How many sexual partners have you had?" "Did you use an electric blanket on your side?" "Why don't you try docosanol cream (Abreva) on your lesions

Did you have chicken pox as a child?"

How do you treat type II diabetes? (DOA)

Diet 1 Oral hypoglycemic 3 Activity 2

Therapeutic Dosages and Toxicity values "the magic 2s" Digitalis, Lithium, Theophylline, Dilantin, APAP

Digitalis (0.5-1.5) Tox = 2 Lithium (0.6-1.2) Tox = 2 Theophylline (10-20) Tox = >20 Dilantin (10-20) Tox = >30 APAP (1-20) Tox = 120

7. An immobilized client is suspected of having atelectasis. This is assessed by the nurse upon auscultation as: 1. Harsh crackles 2. Wheezing on inspiration 3. Diminished breath sounds 4. Bronchovesicular whooshing

Diminished breath sounds

Low pressure alarms are normally caused by?

Disconnection

"What insulin type can be given by IV? Select all that apply: "A. Glipizide (Glucotrol) B. Lispro (Humalog) C. NPH insulin D. Glargine (Lantus) E. Regular insulin

E) Regular insulinThe only insulin that can be given by IV is regular insulin.

For Peds when in doubt pick the?

Easier task.

What raises ammonia levels the most?

Ecoli in the gut

Caput Succedaneum is?

Edematous swelling on the scalp caused by pressure during birth. This swelling may cross suture line. It usually disappears in a few day.

SE of Cervidil?

Effacement which leads to contractions.

What are the four Tricyclic antidepressants you need to know?

Elavil Tofranil Aventyl Desyrel

What position are your hands in for scrubbing?

Elbows below hands.

Increased perfusion for VEINS and ARTERIES

Ele*V*ate *V*eins (*Think V,V*) and D*A*ngle *A*rteries. (*Think A,A*)

Tricyclic's do what to the mood?

Elevates it

On top of alcohol a patient taking Antabuse should also avoid what other 7 things?

Elixirs, Vanilla Extract, Aftershave/Perfumes, Alcohol based hand sanitizer, Insect repellant, Mouthwash and Vinagerette.

10. To promote respiratory function in the immobilized client, the nurse should: 1. Change the client's position every 4 to 8 hours 2. Encourage deep breathing and coughing every hour 3. Use oxygen and nebulizer treatments regularly 4. Suction the client's secretions every hour

Encourage deep breathing and coughing every hour

Interventions for Anaphylaxis

EpiPen or Twinject Assess respiratory function; establish airway CPR (if needed) Epinephrine Antihistamines (for angioedema and urticaria) Oxygen Treat bronchospasm IV fluids

Situation: The client with a history of asthma is admitted to the clinic for allergy testing. During skin testing, the client develops shortness of breath and stridor and becomes hypotensive. What is the most appropriate drug for the nurse to give in this situation? A. Epinephrine (Adrenalin) B. Fexofenadine (Allegra) C. Cromolyn sodium (Nasalcrom) D. Zileuton (Zyflo)

Epinephrine (Adrenalin)

3. Two nurses are standing on opposite sides of the bed to move the client up in bed with a drawsheet. Where should the nurses be standing in relation to the client's body as they prepare for the move? 1. Even with the thorax 2. Even with the shoulders 3. Even with the hips 4. Even with the knees

Even with the shoulders

Use the even numbered gaits when weakness is?

Evenly distributed (bilateral)

S/S of downer use?

Everything goes down...Bradycardia, decreased BP etc.

S/S of upper use?

Everything goes up...Tachycardia, increased BP etc.

Downers are?

Everything other then the five uppers.

Type III: Immune Complex Reactions

Excess antigens cause immune complexes to form in blood Circulating complexes lodge in small blood vessels Deposited complexes trigger inflammation, result in tissue or vessel damage Formation of immune complex of antigen and antibody, which deposits in walls of blood vessels and results in complement release and inflammation Ex. Serum sickness, Vasculitis, Systemic lupus erythematosus, Rheumatoid arthritis

Number 1 intervention for Lithium toxicity?

Keep them hydrated If sweating give sodium as well as fluids

Dumping syndrome is?

Gastric contents moving in the RIGHT DIRECTION and the INCORRECT RATE.

Hiatal hernia is?

Gastric contents moving in the WRONG DIRECTION and the CORRECT RATE.

Anaphylaxis

First feelings of uneasiness, apprehension, weakness, impending doom Pruritus and urticaria Erythema, sometimes angioedema of eyes, lips, tongue Histamine causes capillary leak, bronchoconstriction, mucosal edema, excess mucus secretion Causes congestion, rhinorrhea, dyspnea, increasing respiratory distress with audible wheezing Can be fatal Drugs and dyes most common causes

A client is prescribed prednisone for treatment of a type I reaction. The nurse plans to monitor the client for which adverse effects? Select all that apply. A. Fluid retention B. Gastric distress C. Hypotension D. Infection E. Osteoporosis

Fluid retention Gastric distress Infection Osteoporosis

What is a somatic delusion?

False fixed belief about a body part (X-ray vision).

What is a paranoid delusion?

False fixed belief that people are out to harm you (CIA, FBI).

What is a grandiose delusion?

False fixed belief that you are superior (God, the Pope).

What is a hallucination?

False, fixed sensory experience.

Piagets pre-operational stage is characterized by?

Fantasy oriented Illogical No rules

In psych choose the answer that reflects the _______ and not the _______ they said.

Feelings Words

Positive signs of pregnancy?

Fetal skeleton on X-ray Fetal presence on ultrasound Auscultation of FHR (heard at week 8, most likely at 10 and should be heard by 12) Examiner palpates fetal movement

Hyperprexia is?

Fever- really bad fever

When talking about arrythmias the word "chaotic" means?

Fibrillation

What two things should you work on with a preschooler (3-6 yr.) ?

Fine motor (fingers) Balance (dance, gymnastics)

For a toddler pick ______ food.

Finger

The uterus should be like what after delivery?

Firm not boggy Fundal height= days postpartum (3days= 3cm below naval) Midline- if not catheterize

If the WBC is low you should?

Follow strict handwashing Shower BID with antimicrobial soap Avoid crowds Private room No fresh flowers or potted plants Low bacteria diet- no raw fruits/veggies, no undercooked meat. No water drinking if its been sitting for longer then 15 minutes. Vitals Q4H Check WBC daily Avoid reusable plates/silverware etc,

A nurse is assisting in preparing a plan of care for a client with acquired immunodeficiency syndrome (AIDS) who has nausea. Which dietary measure should the nurse include in the plan? Dairy products with each snack and meal Red meat daily Adding spices to food to make the taste more palatable Foods that are at room temperature

Foods that are at room temperature

How long do you go to the doctor for pregnancy once a month?

For 28 weeks.

A 12-15 yr old is in what Piaget stage?

Formal operations

How long do you go to the doctor every two weeks for pregnancy?

From week 28-36

How long do you go to the doctor for pregnancy every week?

From week 36-42

Casues of hypokalemia: GRAPHIC IDEA

G: GI losses R: renal A: aldosterone P: periodic paralysis H: hypothermia I: insulin excess C: cushings I: insuficient intake D: diuretics E: elevated beta adrenergic activity A: alkalosis

Diseases cause by steptococcus pyogenes: GET NIPPLES

G: glomerulonephritis E: endocarditis T: toxic shock syndrome N: necrotizing fasciitis and myositis I: impetigo P: pharyngitis P: pneumonia L: lymphangitis E: erysipelas and cellulitis S: scarlet fever/rheumatic fever

S/S of hiatal hernia are?

GERD upon lying after meals

Kayexelate does what?

Gets rid of K+ slow and late K-exits-late

If asked how to treat A-fib first you?

Give Heparin then ABCD.... Heparin is instant Coumadin and Plavix take time.

PPE contains?

Gloves, gown, gogles and mask unless told otherwise.

What two drugs can increase blood sugar?

Glucagon and Epinephrine

APGAR: G

Grimace - cough/grimace/no response

What should you work on with a toddler (1-3 yr.) ?

Gross motor

Statins side effects: HMG-CoA

H: hepatotox, headache M: myalgia, myopathy G: GI symptoms C: CPK increased A: a-fib

MAOI side effects:HAHA

H: hypertension/hypertensive crisis A: anticholincergic H: hepatocellular jaundice A: arrhythmia, anorexia

Serotonin Syndrome characteristics: HARM

H: hyperthermia A: autonomic instability (delirium) R: rigidity M: myoclonus

Causes of shock: HAVANA

H: hypovolemia A: adrenal crisis V: vascular stasis A: acute respiratory obstruction N: neurogenic A: anaphylaxis

metabolic alkalosis

HCO3 greater than 26

metabolic acidosis

HCO3 less than 22

What lab is the best indicator for long term blood sugar maintenance?

HGBA1C aka glycosated hemoglobin

Rule of Nines (total surface area for burns)

Head 9% Chest 9% Abdomen 9% Upper Back 9% Lower Back 9% Anterior Leg 9% (each) Posterior Leg 9% (each) Entire Arm 9% (each) Genitalia/perineum 1%

Early decelerations?

Head pressed on. This is ok.

A client has been ordered norepinephrine (Levophed) for treatment of severe hypotension. The nurse plans to monitor the client for which adverse effect? A. Bradycardia B. Headache C. Infection D. Metabolic alkalosis

Headache

Calcium Channel Blocker side effects? (the 2 H's)

Headache and Hypotension

SE of Nifedipine?

Headache and Hypotension (H&H)

In aminoglycoside use monitor?

Hearing, balance, tinnitus & creatinine (best indicator of renal function)

A client is admitted to the hospital with suspected Goodpasture's syndrome. Which findings will the nurse expect to observe? A. Bradycardia B. Hemoptysis C. Increased urine output D. Weight loss

Hemoptysis

What 4 things are always considered unstable?

Hemorrhage Hypoglycemia Fevers over 104* Pulselessness and breathlessness

Post Op Thyroidectomy risks 1st 12 hrs?

Hemorrhage and airway. After 1st 12hrs assume they are stable.

Aminoglycosides are given PO for what two reasons?

Hepatic Encephalopathy and Pre-op bowel surgery.

Administration of an Enema

Here, position the patient in *left side-lying* (Sims) w/ Knee Flexed.

In a vented client respiratory alkalosis means the vent setting may be too?

High

An Apical chest tube is placed?

High (for air) A for air

Hepatic Encephalopathy is caused by?

High ammonia levels

Thyroid storm treatment?

High flow O2, 5 ice packs= 2 under each arm, 2 groin, 1 back of neck.

14. To reduce the chance of plantar flexion (footdrop) in a client on prolonged bed rest, the nurse should implement the use of: 1. Trapeze bars 2. High-top sneakers 3. Trochanter rolls 4. Thirty-degree lateral positioning

High-top sneakers

For a laminectomy what incision has the most drainage/bleeding?

Hip site

For a laminectomy what incision site is more painful?

Hip site

Cyclosponine side effects: 5H

Hirsutism Hyperplasia of gums Harm to kidneys Hypertension Hyperglycemia

Patient-Centered Collaborative Care for Allergic Rhinitis

History Allergy testing Oral food challenges

If the INR is over 4 you?

Hold all Coumadin/Warafin Assess bleeding Prepare to give K+ Call doctor

If the potassium is high (5.4-5.9 high but still within the 5's) you?

Hold potassium if in the IV Assess the heart Prepare to give D5W with insulin Call the doctor

With a Total Thyroidectomy what do they need for life?

Hormone replacement (Synthroid)

If a patient with hypothyroidism is NPO for surgery you still give them what?

Hormone replacement medication.

Treatment for hypothyroidism?

Hormone supplement

How do the following impact blood pressure? A. Blood pressure cuff too narrow B. Blood pressure cuff too wide C. Assessing immediately after smoking D. Assessing immediately after eating E. Assessing when the client is in mild-to-moderate pain F. Assessing when the client experiences severe pain G. Assessing immediately after exercise

How do the following impact blood pressure? A. Blood pressure cuff too narrow: False Increase B. Blood pressure cuff too wide: False Decrease C. Assessing immediately after smoking: Increase D. Assessing immediately after eating: Increase E. Assessing when the client is in mild-to-moderate pain: Increase F. Assessing when the client experiences severe pain: Increase. Eventually chronic pain modulates to decrease. G. Assessing immediately after exercise: Immediately upon stopping it is increased, but within 5 minutes decreases.

Which insulin works the fastest?

Humalog

Hyperthyroidism (AKA graves disease) is the same as saying?

Hypermetabolism

S/S of Addison's Disease?

Hyperpigmentation and do not adapt well to stress.

The nurse is reviewing the medical record of a client who is prescribed a decongestant. The nurse plans to contact the client's health care provider if the client has which condition? A. Cataracts B. Crohn's disease C. Diabetes mellitus D. Hypertension

Hypertension

Dobutamine - SE

Hypertension PVCs asthmatic episodes headache

What are you at risk for following a Total Thyroidectomy?

Hypocalcemia (parathyroid regulates calcium)

Hypothyroidism is the same as saying?

Hypometabolism

These meds are used for

Hypovolemic shock cardiac arrest anaphylaxis

Cranial Nerves I-XII Oh, Oh, Oh, To Touch And Feel Very Good Velvet, Ah Heaven!

I: Olfactory Nerve II: Optic Nerve III: Oculomotor nerve IV: Trochlear nerve V: Trigeminal nerve (dentist nerve) VI: Abducens nerve VII: Facial nerve VIII: Vestibulocochlear nerve (auditory nerve) IX: Glossopharyngeal nerve X: Vagus nerve XI: Accessory Nerve (spinal accessory nerve) XII: Hypoglossal nerve

Cranial Nerves sensory vs motor S=sensory M=motor B=both

I: Some II: Say III: Money IV: Matters V: But VI: My VII: Brother VIII: Says IX: Big X: Brains XI: Matter XII: Most

Antipsychotics side effects: ISHADE

I: impotence S: sedation, seizures H: hypotension A: akathisia (inability to sit still) D:dermatological E: extrapyramidal (dystonias, rigidity, tremor, tachycardia)

Insulin function on cells

INsulIN stimulates 2 things to go IN 2 cells: K and glucose

DKA treatment?

IV at high flow rate (200hr.) with insulin R (it doesnt matter what solution its in)

What routes can you give Heparin?

IV or SUBQ

Aminoglycoside route of administration?

Im or IV

When do you use scrubbing?

Immunosuppresion, surgery, transplant, chemo, HIV

Antidepressants work?

In 2-4 weeks

Where do you remove PPE?

In the room.

deformaties of clubfoot: InAdEquate

In: inversion Ad: adduction Equate: equinus

If the pH is up my patient with show signs and symptoms of?

Increase... like tachycardia,diarrhea and borborygmi

High pressure alarms are caused by what three types of obstructions?

Kinking, Water in dependant loops and mucus in the airway.

How do you treat painful back labor?

Knee to chest position You use your fist and press on the patients sacrum.

"terol" - albuterol, arformoterol, formoterol, levalbuterol, salmeterol

beta agonist - bronchodilator

Hypersensitivities/Allergies

Increased/excessive response to presence of antigen exposure Degree of reaction ranges

A nurse is assisting in developing a plan of care for a client with immunodeficiency. The nurse understands that which problem is a priority for the client? Infection Inability to cope Lack of information about the disease Feeling uncomfortable about body changes

Infection

If a question has a drug with "zap " in it, the answer is probably?

Infection related

What interventions do you do for all other OB complications? (LION)

L-erft side position I-ncrease IV O-2 N-otify doctor If Pitosin is running stop this first then LION

What is the first thing you assess in a Med-Surg situation?

L.O.C. = Bob, Bob, Bob are you ok????

Prozac causes what?

Insomnia

30. The nurse caring for a 38-year-old female client with multiple fractures in the trauma intensive care unit knows that this client is at high risk for pulmonary complications such as atelectasis from her immobility. One of the interventions that the nurse can do to help prevent this from occurring is to: 1. Keep the PaO2 level at or above 94% 2. Instruct the client to deep breathe and cough every hour while awake 3. Turn the client every 2 hours 4. Keep the client on the ventilator as long as possible

Instruct the client to deep breathe and cough every hour while awake

Type I diabetes (IJK)?

Insulin dependant Juvenile onset Ketosis prone ( makes ketones)

5. A client has sequential compression stockings in place. The nurse evaluates that they are implemented appropriately by the new staff nurse when the: 1. Initial measurement is made around the client's calves 2. Intermittent pressure is set at 40 mm Hg 3. Stockings are wrapped directly over the leg from ankle to knee 4. Stockings are removed every hour during application

Intermittent pressure is set at 40 mm Hg

What do you do if another staff member is placing the patient in physical or psychological harm?

Intervene and take over.

At birth if the mother was addicted to a substance always assume the newborn is in?

Intoxication

What three things should you ask yourself when choosing appropriate toys for kids?

Is it safe Is it appropriate Is it feasible

When a client is sick what does it do to there blood sugar?

It increases it.

Low pressure alarms are triggered when?

It is to easy to push air in.

Examples of rapidly metabolizing carbs?

Juice, hard candy, icing, honey, syrup

What is flight of ideas?

Jump from word to word. (This room is big, I liked the movie BIG when they were on the piano, Elvis could play the piano).

What is word salad?

Jump from word to word. Bob, Car, Sleep, Foot etc..

Nere push what electrolyte?

K+

If in OB its low or late you?

LION

What is the #1 post-op answer for the NCLEX?

Log roll the patient

Antidepressants can be taken?

Long

Tranquilizers shouldn't be taken?

Long

Deconoate means?

Long IM form given to noncompliant patients.

What is the one circumstance that you as a nurse would support denial?

Loss and Grief

In a vented client respiratory acidosis means the vent may be too?

Low

A Basilar chest tube is placed?

Low (for blood) B for blood

Lumbar =

Lower

Osmotic Diuretics: MIG

M: mannitol I: isosorbide G: glycerol

Causes of hyperkalemia: MACHINE

M: meds (ACE inhibitors, steroids, beta blockers A: acidosis C: cellular destruction H: hypoaldosteronism, hemolysis I: intake excessive N: nephrons (renal) failure E: excretion impaired

s/s of hyperkalemia: MURDER

M: muscle weakness U: urine oliguria or anuria R: respiratory distress D: decreased cardiac contractility E: EKG chagnes (peaked T waves or small P waves) R: reflexes hyper or hypo

Morphine side effects: MORPHINES

M: myosis O: orthostatic hypotension R: respiratory depression P: pneumonia (aspiration) H: hypotension I: infrequent waste release (constipation urinary retention) N: nausea E: emesis S: sedation

CK: creatine kinase diagnose acute MI MM vs MB bands

MM bands = skeletal muscle MB bands = heart muscle

If the pH and the BiCarb are both in the same direction then it is?

Metabolic

If it is not lung or prolonged vomiting or suctioning I choose?

Metabolic Acidosis

If my patient has prolonged gastric vomiting or suction I choose?

Metabolic Alkalosis

If a child has O2 is use what type of toys can't be used?

Metal

Lithium toxic SE?

Metallic taste Severe diarrhea

What is an illusion?

Misinterpretation of reality. It is a sensory experience.

If client exercises less they need?

More insulin

Be concerned about PVC's if? ( the 6, 6 T's of PVC's)

More then 6 per minute, 6 in a row

In Peds always give?

More time

All congenital heart defects have what?

Murmur and an echocardiogram done

What is Baclofen(Lioresil, Flexeril)?

Muscle relaxer

Universal s/s of an electrolyte imbalance is?

Muscle weakness (AKA paresis)

Baclofen(Lioresil, Flexeril) SE?

Muscle weakness and drowsiness

What is the best toy for a 0-6 month old?

Musical mobile (get rid of this at 6 months or when the child is sitting up)

What is the name of the disease for hypothyroidism?

Myexedema

NPH insulin all have a what in them?

N

If an elderly patient OD's on Haldol what are they at risk for?

NMS -Neuroleptic Malignant Syndrome

"fenac" - bromfenac, diclofneac, nepafenac "profen" - fenoprofen, fluribiprofen, ibuprofen, ketoprofen

NSAID

Cervical =

Neck

The rule of ABC"s doesnt work in what area of health?

Neuro

WBC (in order of decreasing numbers): Never Let Monkeys Eat Bananas

Never: neutrophils Let: lymphocytes Monkeys: monocytes Eat: eosinophils Bananas: basophils

Are patients with Alcohol Withdrawal Syndrome a danger to themselves or others?

No

Can Alcohol Withdrawal Syndrome kill you?

No

Can the sink have handles for scrubbing?

No

Does a pneumonectomy get a chest tube?

No

Is Wernickes reversible?

No

Is bubbling in the suction control chamber intermittently good?

No it is bad. You need to dial up the suction, report and record.

Is bubbling in the water seal continuously good?

No it is bad. You need to find the air leak, tape it, report it and then record it.

How long can you clamp a chest tube?

No longer then 15 seconds without a doctors order.

Do you sedate a patient with hypothyroidism?

No you could put them into a coma.

Type II diabetes? (non all of the above)

Non insulin dependant Adult onset Non ketosis prone

For Peds when in doubt call it?

Normal

Isotonic fluids

Normal Saline lactated ringers 5% dextrose in water

Acrocyanosis is?

Normal cyanosis of the babys hands and feet which appears intermittently over the 1st 7-10 days.

Hyperbilirubinemia is?

Normal, physiologic jaundice appears after 24 hours of age and disappears at about one week.

What defects have left to right shunts and are acyanotic?

Not trouble defects

Atropine tox

mad as a hatter (confusion) dry as the street hot as the summer red as a beet

The nurse interprets that the client who is prescribed zalcitabine (Hivid) is experiencing an adverse effect of this medication when which event is reported by the client? Diarrhea Tinnitus Burning with urination Numbness in the legs

Numbness in the legs

Earliest sign of an electrolyte overload is?

Numbnesss and tingling (AKA parasthesia)

9. A client is getting up for the first time after a period of bed rest. The nurse should first: 1. Assess respiratory function 2. Obtain a baseline blood pressure 3. Assist the client with sitting at the edge of the bed 4. Ask the client if he or she feels light-headed

Obtain a baseline blood pressure

The second stage of labor and delivery is all about?

Order

What should you monitor with Zoloft use?

Other drugs levels

Toxic effects of aminoglycosides?

Ototoxicity, nephrotoxicity and cranial nerve 8 (vestibulocochlear nerve) which senses sound.

Where do you put on PPE?

Outside the room.

Lanoxin (digoxin) toxic level?

Over 2.0

Aminophylline toxic lever?

Over 20

Dilantin toxic level?

Over 20

Toxic bilirubin level?

Over 20 neonate only

signs of inflammation: PRISH

P: pain R: redness I: immobility (loss of function) S: swelling H: heat

Primary Syphilis: PRESS

P: painless lesion R: regional lymphadenopathy E: exudate S: single lesion S: sexual contact is cause

"afil" - avanafil, sildenafil, tadalafil, vardenafil

PDE inhibitor (treats erectile dysfunction)

How do you assess the intensity of a contraction?

Palpate with one hand over fundus and with the fingertips.

What is toddler (1-3 yr.) play characterized by?

Parallel play (with another child but alone)

What are the three types of delusions?

Paranoid or Persecutory, Grandiose and Somatic

26. A 78-year-old inactive client diagnosed with acute renal failure is at risk for which of the following skeletal maladies? 1. Rickets 2. Osteomyelitis 3. Pathological fractures of long bones 4. Compression fractures of the spinal column

Pathological fractures of long bones

"cillin" - amoxicillin, ampicillin, dicloxacillin, nafcillin, oxacillin

Penicillin antibiotic

Nephrotic Syndrome: People Have Endless Appetites

People: Proteinuria Have: hyperlipidemia Endless: edema Appetities: albuminuria & hypoalbuminemia

PVC is?

Periodic wide, bizarre QRS's

What are the two long term problems from diabetes?

Peripheral neuropathy and poor tissue perfusion.

For walkers remember you?

Pick it up, set it down and walk to it.

What are the two oxytocic's?

Pitocin=Oxytocin Cervidil= Prostaglandin

A client who is human immunodeficiency virus (HIV) positive has had a Mantoux skin test. The results show a 7-mm area of induration. The nurse evaluates that this result is:

Positive

Hydroxyzine - NC

no dependence, tolerance, or intoxication

What post-op complication do you watch for with a cervical laminectomy?

Pneumonia

What post-op complication do you watch for with a thoracic laminectomy?

Pneumonia Paralytic ileus

6Ps of dyspnea

Pneumonia Pulmonary bronchial constriction Possible foreign body Pulmonary embolus Pneumothorax Pump failure

How do you confront?

Point out the difference between what they say and what they do.

s/s hyperglycemia: 3 Ps

Polyphagia Polydipsia Polyuria (also: hot and dry, sugar high)

Diabetes S/S?

Polyuria, Polydypsia and polyphagia (hunger)

What is Opisthotonos?

Position of hyperextension seen with kernicterus. Place them on there side if present.

As the pH goes so goes my patient except for?

Potassium

Which electrolyte replacement should the nurse anticipate being ordered by thehealth-care provider in the client diagnosed with DKA who has just been admitted tothe ICD? 1.Glucose. 2.)Potassium. 3.Calcium. 4.Sodium

Potassium"1.Glucose is elevated in DKA; therefore, theHCP would not be replacing glucose. 2.(CORRECT)-->The client in DKA loses potassium from increased urinary output, acidosis, cata-bolic state, and vomiting. Replacement isessential for preventing cardiac dysrhyth-mias secondary to hypokalemia. 3.Calcium is not affected in the client with DKA.4.The IV that is prescribed 0.9% normal salinehas sodium, but it is not specifically orderedfor sodium replacement. This is an isotonicsolution. TEST-TAKING HINT: Option "1" should be elim-inated because the problem with DKA iselevated glucose so the HCP would not bereplacing it. The test taker should use physiol-ogy knowledge and realize potassium is in thecell."

A 3-6 yr old is in what Piaget stage?

Pre-operational

If platelets are below 40,000 you?

Prepare platelet transfusion Call the doctor

Piagets sensorimotor stage is characterized by?

Present oriented Only think about what they SENSE or are DOING now.

What sort of play do preschoolers (3-6 yr.) prefer?

Pretend play

Contact precautions have what 5 things?

Private room - door can be open Gloves Gown- if giving direct care Handwashing Disposable supplies

N/I for Delirium Tremens?

Private room near nurses station, NPO/Clear liquids, Restricted bed rest, restraints, tranquilizer, multivitamin (B1 vit.) and antihypertensive.

Droplet precations have what 6 things?

Private room- door can be open Gloves Mask- #1 in this group Handwashing Pt wears mask when leaving room Disposable/dedicated equipment

Airborne precautions have what 9 things?

Private room-door closed Mask Gloves Gown Handwashing Special filter respirator masks Pt wears a mask when leaving room Disposable/dedicated supplies Negative airflow room

Attenuated Vaccines: ROME Is My Best Place To Yell

R: rubella O: oral polio M: measles E: epidemic typhus Is: influenza My: mumps Best: BCG Place: plague To: typhoid oral Yell: yellow fever

If my patient is overventilating I should choose?

Respiratory Alkalosis

Antidote for Heparin is?

Protamine Sulfate

What is Wernickes (Korsakoffs) Syndrome?

Psychosis induced by vitamin B1 (Thiamine) deficiency.

What group of drugs is the most commonly tested on the NCLEX?

Psychotropic

What are phenothiazines?

Psychotropic drugs

What do you monitor in Coumadin?

Pt-INR(this is only for Coumadin)q

A client with acquired immunodeficiency syndrome (AIDS) has difficulty swallowing. The nurse has given the client suggestions to minimize the problem. The nurse determines that the client has understood the instructions if the client verbalized to increase intake of foods such as: Raw fruits and vegetables Hot soup Peanut butter Puddings

Puddings

APGAR: P

Pulse - >100/<100/Absent

Extremity assessment post partum should check?

Pulses Edema S/S of thrombophlebitis- bilateral calf circumference is the best way to check. If they are equal its ok if not its positive for whichever calf is bigger)

How do you treat prolapsed cord? 911!! OB emergency

Push head back in Position in knee chest position or Trendelenburg

What is Chvosteks sign?

Push the cheek and it spasms

What is the #1 nursing diagnosis for Benzo's?

Safety

When drawing up insulins its?

RN (regular then N) draw what you are RN)

Acidosis/Alkalosis: ROME

RO: respiratory opposite (pH up PCO2 down=alkalosis pH down CO2 up=acidosis) ME: metabolic equal (pH up HCO3 up=alkalosis pH down HCO3 down=acidosis)

Contact precautions are used for what 4 types of diseases?

RSV, Herpatic infection, Staph infections and Enteric (bowel) infections

Atrail Flutter is?

Rapid P-wave repolarizations in a saw tooth pattern.

Treatment for low blood sugar in type I diabetes?

Rapidly metabolizable carb or ideallly a carb and protein combination or carb and starch. If they are unconcious NPO give glucagon IM or dextrose IV.

Truest most valid sign of labor is?

Regular contractions

What are the 4 types of insulin?

Regular, NPH, Humalog and Lantus

Treatment for HHNK is?

Rehydrate but no insulin in the bag because it is not type I diabetes. You only give insulin in the bag with type I diabetes because they burn fat and it turns to ketones.

Lie is?

Relationship between the spine of the baby and the spine of the mom. Vertical (parallel) is ok, Tranverse (perpindicular) is bad.

Which of these nursing actions is most appropriate for the nurse working in an allergy clinic to delegate to a nursing assistant? A. Plan the schedule for desensitization therapy for a client with allergies. B. Monitor the client who has just received skin testing for signs of anaphylaxis. C. Educate a client with a latex allergy about other substances with cross-sensitivity to latex. D. Remind the client to stay at the clinic for 30 minutes after receiving intradermal allergy testing.

Remind the client to stay at the clinic for 30 minutes after receiving intradermal allergy testing.

The client is being discharged from the hospital after an allergic reaction to environmental airborne allergens. Which instruction is most important for the nurse to include in this client's discharge teaching plan? A. Wash fruits and vegetables with mild soap and water before eating. B. Intermittent exposure to known allergens will produce immunity. C. Remove cloth drapes, carpeting, and upholstered furniture. D. Be cautious when eating unprocessed honey.

Remove cloth drapes, carpeting, and upholstered furniture.

The client recently admitted to the hospital is to receive an antibiotic intravenously for the first time for a urinary tract infection. Before checking the five R's prior to administration, what is the nurse's first action? A. Review the clinical records and ask the client about any known allergies. B. Check with the pharmacy for any known allergies for this client. C. Check the client's identification band for any allergies. D. Ask the nurse who previously cared for the client about any known allergies.

Review the clinical records and ask the client about any known allergies.

What do you use to clamp a chest tube and why?

Rubber tipped double clamps. Rubber because it won't pierce the tube and double because were nurses and if one is good two is better.

Congenital signs of rubella: "Rubber Ducky, I so Blue"

Rubber: rubella Ducky: ductus arteriosus I: eyes B: blueberry muffin rash

What are the four types of functional psychosis?

Schizophrenia, Schizoaffective disorder, Major depression/mania (bipolar).

N/I for Alcohol Withdrawal Syndrome?

Semi-private room anywhere, regular diet, up and ad-lib, no restraint, tranquilizer, multivitamin (B1) and antihypertensive.

A 0-2 yr old is in what Paiget stage?

Sensorimotor

St. John's wort and Zoloft use can cause?

Serotonin Syndrome

To treat dependency/codependency you ?

Set limits and enforce them. You also need to work or the self estreem of the codependent.

How do you treat manipulation?

Set limits and enforce.

Clozaril(Clozapine) is used to treat?

Severe schizophrenia

What can children under for not have?

Small toys

Epsteins pearls are?

Small,white epithelial cysts on babys gums.

Risk factors for type 2 diabetes include all of the following except: "a. Advanced age b. Obesity c. Smoking d. Physical inactivity"

Smoking "Additional risk factors for type 2 diabetes are a family history of diabetes, impaired glucose metabolism, history of gestational diabetes, and race/ethnicity. African-Americans, Hispanics/Latinos, Asian Americans, Native Hawaiians, Pacific Islanders, and Native Americans are at greater risk of developing diabetes than whites."

What do you use in handwashing?

Soap and water

What do you teach patients taking phenothiazines to report?

Sore throat and any signs of infection.

Patients taking Zoloft should NOT take what?

St. John's wort

Oxytocics do what to labor?

Start it

Engagement is?

Station 0

Sulfonamide side effects: 4S

Steven-Johnson syndrome Skin rash Solubility low (causes crystalluria) Serum albumin displaced (causes newborn kernicterus and potentiation of other serum albumin-binders like warfarin)

When talking about arrythmias the word 'bizarre" means?

Tachy

Teach diabetics who are sick to?

Take insulin, take sips of water to stay hydrated and stay as active as possible.

What is the best toy for a 9-12 month old?

Talking toy.

Family can only do what you ____ them for the patient?

Teach

As a nurse when, what and how do you teach a child in the pre-operational stage?

Teach the day of- to avoid nightmares What you're going to do Do this through play

As a nurse when, what and how do you teach a child in the concrete operations stage?

Teach them ahead of time What you're going to do Use visual and audio and use age appropriate reading

As a nurse when, what and how do you teach a child in sensorimotor stage?

Teach when you're doing it What you're doing to them Do it verbally

If another staff member is doing something illegal you?

Tell the supervisor.

Nevus/Nevi is?

The generic term for a birthmark.

The first stage of labor is?

The labor part.

For a prioritization question the most important information is?

The modifying phrase

If my pH is down my potassium (K+) is?

Up

For Peds when in doubt pick?

The older age.

Presentation is?

The part of the body that enters the birth canal first.

S/S of low blood sugar are?

The same S/S of shock and being drunk... clammy, weak, slurred speach etc.

S/S of a Thyroid storm are?

The same as graves disease but incredibly higher. Exp: HR 180, Temp 108*, psychotic delirium.

What side do you advance the cane with?

The weak side for a wide base support. *Step with opposites*

How can you diffirentiate between illusions and hallucinations?

There is a referent ( something to which they refer). So the patient takes something from reality and has a sensory response.

High pressure alarms are triggered when?

They cannot push air in

Withdrawal of a downer causes everything to go?

Up

S/S of Cushing's Syndrome?

Think of the Cush Man... Moon face, buffalo hump, thin legs and bones, striae (stretch marks), male breasts, hirstusism etc,

Effacement is?

Thinning of the cervix (0-100%)

Station is?

This refers to the baby's presenting part (normally the head) to the mothers ischeal spine. So if the baby is above the ischeal spine they are given a - number, if they are below they are given a + number. + numbers are positive news and - numbers are negative news.

If it has "thro" in it you?

Throw it out...Zithromycin

For a Sub-total Thyroidectomy you are at risk for what?

Thyroid storm

Post Op thyroidectomy risks 12-48 hrs for Sub-total Thyroidectomy?

Thyroid storm

35. The nurse and a nursing assistive personnel (NAP) are going to move an older adult client up in bed. Before moving the client, the nurse explains to the NAP that they will need to lift the client off the bed with an assistive device instead of using the drawsheet. The most important reason for using the assistive device is: 1. To avoid frightening the client 2. To avoid shearing the client's skin 3. To avoid getting "written up" for not following lift procedures 4. Because the nurse is tired

To avoid shearing the client's skin

A nurse is collecting data on a client with rheumatoid arthritis. The nurse looks at the client's hands and notes these characteristic deformities. The nurse identifies this deformity as: Refer to figure. Ulnar drift Rheumatoid nodules Swan neck deformity Boutonniere deformity

Ulnar drift

How do you give Survanta?

To the baby After delivery By inhalation

How do you give Betamethasone?

To the mom Before delivery IM

Reason for laminectomy is?

To treat nerve root compression.

Diabetic Ketoacidosis (AKA high blood sugar in Type I diabetes) is caused by what 3 things?

Too much food Not emough insulin Not enough exercise

What are the two types of Thyroidectomys?

Total and Sub-total.

Zoloft decreases metabolism and can cause?

Toxic drug levels

Benzodiazepines are considered minor what?

Tranquilizers

Major doses of phenothiazines are?

Tranquilizers..... BIG GUNS.

How do you treat dyspnea during pregnancy? (2nd and 3rd trimester)

Tri-pod position (sit, lean forward, elbows on knees)

heart valves in order of blood flow "toilet paper my ass"

Tricuspid Pulmonic Mitral Aortic

13. To reduce the chance of external hip rotation in a client on prolonged bed rest, the nurse should implement the use of a: 1. Footboard 2. Trochanter roll 3. Trapeze bar 4. Bed board

Trochanter roll

What defects have right to left shunts and are cyanotic?

Trouble defects

33. The nurse understands that a pressure ulcer is an impairment of the skin as a result of prolonged ischemia. One of the easiest ways to prevent a pressure ulcer from occurring in an immobile client is to: 1. Keep the skin dry 2. Provide range of motion every shift 3. Use lift equipment when transferring a client 4. Turn the client a minimum of every 2 hours

Turn the client a minimum of every 2 hours

If mucus is present you?

Turn them, cough and have them deeo breath first. If ineffective you then suction.

Hypersensitivity reactions: ACID (types 1-4)

Type 1 - A: anaphylactic Type 2 - C: cytotoxic Type 3 - I: immune complex Type 4 - D: delayed hypersensitivity

Latex Allergy

Type I hypersensitivity reaction Protein found in natural latex rubber products is specific allergen Allergen causes interaction with IgE Incidence of latex allergy is increasing Health care workers especially susceptible

To avoid a severe hypertensive crisis patients taking MAOI's must avoid all foods with?

Tyramine

When going up the stairs or down the stairs with crutched remember?

UP with the GOOD and DOWN with the BAD

What post-op complication do you watch for with a lumbar laminectomy?

Urinary retention

Describe swing through?

Used for two braced extremeties 1. Crutches 2. Legs... they kind of hop.

Post partum assessment should include what?

Uterus Lochia Extremities

What are the 6 arrythmias you are tested over on the NCLEX?

V-fib, A-fib, A-flutter, PVC, A-systole and V-tach

What are the 4 potentially life threatening arrythmias?

V-tach, A-fib, A-flutter and PVC

Valporic acid side effects: VALPROATE

V: vomiting A: alopecia L: liver tox P: pancreatitis, pancytomenia R: retention of fat (wt gain) O: oedema (edema) A: appetite increase T: tremor, thrombocytopenia E: enzyme inducer

What are the 4 things you do 4 times an hour for the 4th stage of labor?

VS- check for S/S of shock Fundus- if boggy, massage. If displaced,void/cath Perineal pad- excessive lochia= pad saturated Q15min 911 Roll on side and check the pad for bleeding.

Calcium Channel Blockers are like what for the heart?

Valium

What is the saying to help remember the four defects of Tetrology of Fellot?

VarieD PictureS Of A RancH

Norepinephrine - NC (nursing considerations)

Vasoconstrictor to increase BP and cardiac output causes reflex bradycarida monitor output infuse w/ dextrose not NS monitor BP protect from light

"QRS" refers to?

Ventricular

What are the four defects of Tetrology of Fellot?

Ventricular Defect, Pulmonic Stenosis, Overriding Aorta and Right Hypertrophy

If variable its?

Very bad and you push and position.

If the fundus is not above the naval the baby is not?

Viable

What is the #1 cause of DKA?

Viral upper respiratory infection within last 2 weeks.

31. The nurse is caring for a 48-year-old male client who was involved in a motor vehicle accident and had a fractured pelvis, a ruptured spleen, and multiple contusions. The client has been in the hospital for 5 days on bed rest. The nurse knows that this client is at risk for venous thrombus formation because of prolonged bed rest, potential damage to vessel walls during surgery, and the platelets he received in the trauma unit. These three factors are also known as: 1. Trigeminy 2. Virchow's triad 3. Trigone 4. Hutchinson's triad

Virchow's triad

What is Kernicterus?

When bilirubin is over 20 and in the CSF(cerebrospinalfluid)

What is dependency?

When the abuser gets a significant other so make decisions for them or do thing for them.

What is manipulation?

When the abuser gets the significant other fo do things for them that is not in the best interest of the significant other. This can be dangerous and harmful to the significant other.

What is codependency?

When the significant other gets positive self esteem from doing things or making decisions for an abuser.

What is a narrowed self concept?

When they refuse to leave the room or get dressed. (DON'T force them to do it)

What is ideas of reference?

When they think everything is about them. (Everyone is talking about me)

What is aversion therapy?

When you try and make the patient hate something.

What is the saying that can be used to help remember Baclofen(Lioresil,Flexiril)?

When you're on Baclofen you're on your back loafin.

Lochia color alba?

White- albino white

Vernix caseosa is?

Whitish, cheese like substance which covers the skin on an unborn baby.

V-tach is?

Wide bizarre QRS's

With humalog when do you give it?

With meals

"sartan" - candesartan, irbesartan, losartan, olmesartan, valsartan

angiotensin II receptor antagonist: ARB

alkalosis pH

above 7.45

"mustine" - carmustine, estramustine, lomustine, bendamustine

alkylating agent - antineoplastic

"zosin" - afdalzuzosin, doxazosin, prazosin, terazosin

alpha blockers

Patients with type 1 diabetes mellitus may require which of the following changes to their daily routine during times of infection? "a. no change b. less insulin c. more insulin d. oral diabetic agents"

answer C: during times of infection and illness diabetic patients may need even more insulin to compensate for increased blood glucose levels.

"dazole"- albendazole, mebendazole, metronidazole, tinidazole

anthelmintic, antibiotic, antibacterial

"mycin" - azithromycin, clarithromycin, clindamycin, erythromycin

antibiotic, antibacterial

"sulfa" - sulfacetamide, sulfadizine, sulfamethoxazole, sulfasalazine

antibiotic, antinfective, antiinflamatory

"zodone" - neazodone, trazodone, vilazodone

antidepressant

"gliptin" - saxagliptin, sitagliptin, linagliptin

antidiabetic: inhibits DPP-4 enzyme

"glitazone" piopglitazone, rosiglitazone, troglitazone

antidiabetic: thiazolidinedione

"nazole" fluconazole, ketoconazole, miconazole, terconazole, tioconazole

antifungal

"iramine" - brompheniramine, chlorpheniramine, pheniramine

antihistamine

"tadine" - cyproheptadine, desloratadine, loratadine, olopatadine

antihistamine

"triptan" - almotriptan, eletriptan, rizatriptan, sumatriptan, zolmitriptan

antimigrane - selective serotonin receptor agonist

"tinib" - crizotinib, dasatinib, erlotinib, gefitinib, imatinib

antineoplastic - kinase inhibitor

"rubicin" - doxorubicin, epirubicin, idarubicin, valrubicin

antineoplastic cytotoxic agent - antitumor

"parin" - dalteparin, enoxaparin, fondaparinux, heparin, tinzaparin

antithrombotic anticoagulant

"antadine" - amantadine, rimantadine "vir" abacavir, eavirenz, enuviritde, adeovir, ribavirin, acyclovir, valacyclovir, ganciclovir, oseltamivir "vudine" - lamivudine, stavudine, telbivudine, zidovudine

antiviral

What are the three reasons that cause low blood sugar in type I diabetes?

not enough food, to much insulin (#1) and too much exercise.

"peridone" - iloperidone, paliperidone, risperidone

atypical antipsychotic

"bital" - butabarbital, butabital, phenobarbital, secobarbital

barbiturate (sedative)

acidosis pH

below 7.35

"dronate" alendronate, etidronate, ibandronate, risedronate

bisphosphonate: bone resorption inhibitor (prevents osteoporosis)

antihistamines

block histamine from binding to the receptor, preventing vasodilation and capillary leak; also decreases secretions S/e: sedation

diphenhydramine - NC

blocks effects of histamine on bronchioles, GI tract, and blood vessels

"zolamide" - acetazolamide, brinzolamide, dorzolamide, methazolamide

carbonic anhydrase inhibitor

decongestants

cause vasoconstriction of inflammed tissue, reducing edema; not for patient's with high blood pressure, glaucoma, or UTI S/e: dry mouth, increased BP, sleep difficulties

"cef" "ceph" - ceaclor, cefdinir, cefixime, cefprozil, cephalexin

cephalosporin antibiotic

"asone" -betamethasone, dexamethaxone, diflorasone, fluticasone, mometasone "cort" - clocortolone, ludrocortisone, hydrocortisone "olone" - flucoinolone, fluorometholone, prednisolone, triamcinolone, nandrolone, oxandrolone, oxymetholone "onide" - budesonide, ciclesonide, desonide, fluocinonide, halcionide "pred" - loteprednol, prednicarbate, prednisolone, prednisone

corticosteroids: alters normal immune response - increases susceptibility to infection - may mask symptoms of infection

Assess for infiltration...

cool skin, swelling, pain

Lochia color serosa?

pink- rose pink

Hydroxyzine - SE

drowsiness, ataxia, leukopenia, hypotension

Benzodiazepines

diazepam, alprazolam, clonazepam, lorazepam, oxazepam, midazalam

Sodium nitroprusside - NC

dilates cardiac veins and arteries decreases preload and afterload increases myocardial perusion

Treat infiltration

discontinue IV apply warm compress elevate arm start new IV in different site (or proximal to infiltrated vein)

Treat phlebitis

discontinue IV apply warm compress restart IV in new site

diphenhydramine - SE

drowsiness confusion insomnia headache vertigo photosensitivity

Milrinone - SE

dysrhythmia thrombocytopenia jaundice

Hypoxia symptoms (early and late) RAT BED

early R: restlessness A: anxiety T: tachycardia/tachypnea late B: bradycardia E: extreme restlessness D dyspnea

Pemphigus

emphigus is an autoimmune disease that causes blistering in the epidermis. The client has large flaccid blisters (bullae). Because the blisters are in the epidermis, they have a thin covering of skin and break easily, leaving large denuded areas of skin. On initial examination, clients may have crusting areas instead of intact blisters. Option 1 describes eczema, option 3 describes herpes zoster, and option 4 describes psoriasis.

change bag

every 24h

change tubing

every 72h

"gestrel" - desongestrel, etonogestrel, levoniorgestrel, norgestrel

female hormone (progestin)

hypercalcemia

greater than 10

hypernatremia

greater than 145

hypermagnesemia

greater than 2.3

hyperkalemia

greater than 5

Norepinephrine - SE (side effects)

headache palpitations nervousness epigastric distress angina hypertension tissue necrosis w/ extravasation

Isoproterenol - SE

headache palpitations tachycardia changes in BP angina bronchial asthma

Sodium nitroprusside - SE

hypotension

Dopamine - SE

increased ocular pressure ectopic beats nausea tachycardia chest pain dysrhthmias

hypomagnesemia

less than 1.3

urine culture and sensitivity

less than 100,000 colonies/ml

hyponatremia

less than 135

LDL: low density lipoproteins

less than 160

hypokalemia

less than 3.5

hypocalcemia

less than 9

Benzodiazepine - SE

lethargy, hypotension, respiratory depression, light-headedness, confusion, orthostatic hypotension, amnesia

Buspirone - SE

light-headedness, confusion, hypotension, palpitations

Buspirone - NC

little sedation 3 wks till effective not PRN no abuse potential avoid alcohol and grapefruit juice monitor for worsening depression and SI

"caine" - bupivacaine, lidocaine, mepivacaine, prilocaine, proparacaine

local anesthetic

"semide" - furosemide, torsemide

loop diuretic

Dopamine - NC

low-dose dilates renal and coronary arteries high-dose vasoconstrictor incresaes myocardial oxygen consumption monitor BP, peripheral pulses, urinary output headache is early sign of toxicity

"mab" - adalimumab, daclizumab, infiximab, omalizumab, trastuzamab

monoclonal antibody

"nacin" - dariffenacin, solifenacin

muscarinic antagonist (anticholinergics)

Epinepherine - SE

nervousness restlessness dizziness local necrosis of skin

Lithium toxic level?

over 2.0

Phenylephrine - SE

palpitations tachycardia hypertension dysrhythmia angina tissue necrosis w/ extravasation

Milrinone - NC

positive inotropic agent smooth muscle relaxant treats severe HF

Phenylephrine - NC

potent alpha 1 agonist treats hypotension

Rule of Palms (total surface area for burns)

surface area of patient's palm = 1% total body surface area

"phylline" - aminophylline, dyphylline, oxtriphylline, theophylline

xanthine derivative - bronchodilator

A client with acquired immunodeficiency syndrome (AIDS) is experiencing shortness of breath related to Pneumocystis jiroveci pneumonia. Which measure should the nurse suggest to assist the client in performing activities of daily living? Provide supportive care with hygiene needs. Provide meals and snacks with high protein, high calorie, and high nutritional value. Provide small, frequent meals. Offer low microbial food.

√ Provide supportive care with hygiene needs.

A client with acquired immunodeficiency syndrome has a respiratory infection from Pneumocystis jiroveci and a nursing diagnosis of Impaired Gas Exchange written in the plan of care. Which of the following indicates that the expected outcome of care has not yet been achieved? The client limits fluid intake. The client has clear breath sounds. The client expectorates secretions easily. The client is free of complaints of shortness of breath.

√ The client limits fluid intake.

2nd/3rd trimester weight gain?

1 lb a week

High baseline variability?

HR changing a lot. This is ok.

Variable decelerations?

HR up or down, cord compression = BERY BAD

How do you treat back pain in pregnancy? (2nd and 3rd trimester)

Pelvic tilt exercise

If O2 sensor line is disconnected you?

Reconnect

If the tubing is disconnected you?

Reconnect

What do you do if the patients disconnected tube is on the chest?

Reconnect ... if its above the waist its ok.

The fourth stage of labor is?

Recovery (1st 2 hours after the placenta is delivered)

Erythema toxicum neonatorum is?

Red papular rash on babys torso which is benign and disappears after a few days.

Lochia color rubra?

Red- rub it red

Which of the following nursing activities is of highest priority for maintaining medical asepsis? 1) Washing hands 2) Donning gloves 3) Applying sterile drapes 4) Wearing a gown

*1) Washing hands*

APAGR: R

Respirations - strong cry/weak cry/absent

If my patient is underventilating I should choose?

Respiratory Acidosis

Piagets concrete operations stage is characterized by?

Rule oriented Live and die by the rules Cannot abstract

The nurse is caring for a client who has normal glucose levels at bedtime, hypoglycemia at 2am and hyperglycemia in the morning. What is this client likely experiencing? "A. Dawn phenomenon B. Somogyi effect C. An insulin spike D. Excessive corticosteroids"

"1. B The Somogyi effect is when blood sugar drops too low in the morning causing rebound hyperglycemia in the morning. The hypoglycemia at 2am is highly indicative. The Dawn phenomenon is similar but would not have the hypoglycemia at 2am."

"The nurse is teaching a community class to peole with Type 2 diabetes mellitus. Which explanation would explain the development of Type 2 diabetes? 1. The islet cells in the pancreas stop producing insulin. 2. The client eats too many foods that are high in sugar. 3 The pituitary gland does not produce vasopression. 4. The cells become resistant to the circulating insulin.

"1. This is the cause of Type 1 diabetes mellitus. 2. This may be a reason for obesity, which may lead to Type 2 diabetes, but eating too much sugar does not cause diabetes. 3. This is the explanation for diabetes insipidus, which should not be confused with diabetes mellitus. 4. (CORRECT) Normally insulin binds to special receptor sites on the cells and initiates a series of reactions involved in metabolism. In Type 2 diabetes these reactions are diminished primarily as a result of obesity and aging."

"The nurse is caring for a woman at 37 weeks gestation. The client was diagnosed with insulin-dependent diabetes mellitis (IDDM) at age 7. The client states, ""I am so thrilled that I will be breastfeeding my baby."" Which of the following responses by the nurse is best? "1. You will probably need less insulin while you are breastfeeding. 2. You will need to initially increase your insulin after the baby is born. 3. You will be able to take an oral hypoglycemic instead of insulin after the baby is born. 4. You will probably require the same dose of insulin that you are now taking."

"1. breastfeeding has an antidiabetogenic effect, less insulin is needed. (correct) 2. insulin needs will decrease due to antidiabetogenic effect of breastfeeding and physiological changes during immediate postpartum period. 3. client has IDDM, insulin required. 4. during third trimester insulin requirements increase due to increased insulin resistance"

"The nurse is caring for a patient whose blood glucose level is 55mg/dL. What is the likely nursing response? "A. Administer a glucagon injection B. Give a small meal C. Administer 10-15 g of a carbohydrate D. Give a small snack of high protein food"

"C The client has low hypoglycemia. This is generally treated with a small snack."

Polydipsia and poly uria related to diabetes mellitus are primarily due to: "a.The release of ketones from cells during fat metabolism b. Fluid shifts resulting from exposure to high levels of hyperglycemia c. Damage to the kidneys from exposure to high levels of glucose d. changes in RBCs resulting from attachemnt of excessibe glucose to hemoglobin"

"Coorect answer: d. Rationale: The osmotic effect of glucose produces the manifesatiaions of polydispsia and poly uria."

HDL: high density lipoproteins (man and woman)

(man) 35-70 (woman) 35-85

RBC ct (man and woman)

(man) 4.5-6 (woman) 4-5

Hct (man and woman)

(man) 42-52% (woman) 35-45%

ESR: sedimentation rate (man and woman)

(man) less than 20mm/h (woman) less than 30mm/h

INR: international normalized ratio effectiveness of anticoag therapy (normal, a-fib, heart valve)

(normal) 1.0 (a-fib, DVT, and pulmonary embolism) 2-3 (heart valves) 2.5-3.5

When should the nurse assess pain? 1) Whenever a full set of vital signs is taken 2) During the admission interview 3) Every 4 hours for the first 2 days after surgery 4) Only when the patient complains of pain

*1) Whenever a full set of vital signs is taken*

"The client diagnosed with Type 1 diabetes has a glycosylated hemoglobin (A1 c) of 8.1%. Which interpretation should the nurse make based on this result? 1.This result is below normal levels.2.This result is within acceptable levels. 3.This result is above recommended levels 4.This result is dangerously high.

"1.The acceptable level for an A1c for a client with diabetes is between 6% and 7%, which corresponds to a 120-140 mg/dL average blood glucose level. 2.This result is not within acceptable levelsfor the client with diabetes, which is 6% to7%. 3.(CORRECT) This result parallels a serum blood glucoselevel of approximately 180 to 200 mg/dL. An A1 c is a blood test that reflects average blood glucose levels over a period of 2-3months; clients with elevated blood glucose levels are at risk for developing long-term complications. 4.An A1c of 13% is dangerously high; it reflects a 300-mg/dL average blood glucose level overthe past 3 months."

"An 18-year-old female client, 5'4'' tall, weighing 113 kg, comes to the clinic for a non-healing wound on her lower leg, which she has had for two weeks. Which disease process should the nurse suspect the client is developing? "A. Type 1 diabetes B. Type 2 diabetes C. Gestational diabetes D. Acanthosis nigricans"

"A: Type 1 diabetes usually occurs in young clients who are underweight. In this disease, there is no production of insulin from the beta cells in the pancreas. People with type 1 diabetes are insulin dependent with a rapid onset of symptoms, including polyuria, polydipsia, and polyphagia. CORRECT -->B. Type 2 diabetes is a disorder usually occurring around the age of 40, but it is now being detected in children and young adults as a result of obesity and sedentary lifestyles. Non-healing wounds are a hallmark sign of type 2 diabetes. This client weights 248.6 lbs and is short. C. Gestational diabetes occurs during pregnancy. There is no mention of this. D. Acanthosis nigricans (AN), dark pigmentation and skin creases in the neck, is a sign of hyperinsulinemia. The pancreas is secreting excess amounts of insulin as a result of excessive caloric intake. It is identified in young children and is a precursor to the development of type 2 diabetes."

The nurse is educating a pregnant client who has gestational diabetes. Which of the following statements should the nurse make to the client? Select all that apply. "a. Cakes, candies, cookies, and regular soft drinks should be avoided. b. Gestational diabetes increases the risk that the mother will develop diabetes later in life. c. Gestational diabetes usually resolves after the baby is born. d. Insulin injections may be necessary. e. The baby will likely be born with diabetes f. The mother should strive to gain no more weight during the pregnancy.

"ANS: A, B, C, D Gestational diabetes can occur between the 16th and 28th week of pregnancy. If not responsive to diet and exercise, insulin injections may be necessary. Concentrated sugars should be avoided. Weight gain should continue, but not in excessive amounts. Usually, gestational diabetes disappears after the infant is born. However, diabetes can develop 5 to 10 years after the pregnancy"

"Prediabetes is associated with all of the following except: " a. Increased risk of developing type 2 diabetes b. Impaired glucose tolerance c. Increased risk of heart disease and stroke d. Increased risk of developing type 1 diabetes"

"ANSWER: D Persons with elevated glucose levels that do not yet meet the criteria for diabetes are considered to have prediabetes and are at increased risk of developing type 2 diabetes. Weight loss and increasing physical activity can help people with prediabetes prevent or postpone the onset of type 2 diabetes."

When preparing the client for allergy testing, the nurse provides the client with which instruction? A. "Avoid taking your loratadine (Claritin) and triamcinolone (Azmacort) inhaler for 5 days before testing." B. "It is okay to use your fluticasone propionate (Flonase) nasal spray before testing." C. "Aspirin in a low dose is allowed to be taken before testing." D. "You can take antihistamine nasal sprays before testing."

"Avoid taking your loratadine (Claritin) and triamcinolone (Azmacort) inhaler for 5 days before testing."

During a diabetes screening program, a patient tells the nurse, "My mother died of complications of type 2 diabetes. Can I inherit diabetes?" The nurse explains that "a.) as long as the patient maintains normal weight and exercises, type 2 diabetes can be prevented. b.) the patient is at a higher than normal risk for type 2 diabetes and should have periodic blood glucose level testing. c.) there is a greater risk for children developing type 2 diabetes when the father has type 2 diabetes. d.) although there is a tendency for children of people with type 2 diabetes to develop diabetes, the risk is higher for those with type 1 diabetes."

"B Rationale: Offspring of people with type 2 diabetes are at higher risk for developing type 2 diabetes. The risk can be decreased, but not prevented, by maintenance of normal weight and exercising. The risk for children of a person with type 1 diabetes to develop diabetes is higher when it is the father who has the disease. Offspring of people with type 2 diabetes are more likely to develop diabetes than offspring of those with type 1 diabetes."

Analyze the following diagnostic findings for your patient with type 2 diabetes. Which result will need further assessment? A) BP 126/80 B) A1C 9% C)FBG 130mg/dL D) LDL cholesterol 100mg/dL

"B) A1C 9% Rationale: Lowering hemoglobin A1C (to average of 7%) reduces microvascular and neuropathic complications. Tighter glycemic control(normal A1C < 6%) may further reduce complications but increases hypoglycemia risk."

28. The nurse is providing ancillary personnel with instructions regarding the performance of passive range-of-motion (ROM) exercises for a client experiencing paralysis from the waist down (paraplegia) as a result of an automobile accident. Which of the following statements made by the ancillary personnel reflects the greatest insight regarding the frequency with which the intervention should be provided for this client? 1. "I will do a whole body range of motion as I complete her daily bath." 2. "Bath time, bedtime, after lunch, and at least once more; she can pick when." 3. "It works well with her bath and when she is being prepared for bed at night." 4. "I'll ask her when she wants me to exercise her joints in addition to bath time."

"Bath time, bedtime, after lunch, and at least once more; she can pick when."

18. Which of the following statements made by ancillary staff reflects the most informed knowledge regarding the benefit of having a client assist with his or her own activities of daily living (ADLs) to that client's activity tolerance? 1. "The more he does for himself, the more he will be able to do for himself." 2. "He doesn't like washing and dressing himself, but it makes him stronger." 3. "Doing for himself makes him tired, but in the long run he has more energy and strength when he does." 4. "By washing and dressing himself he is building muscle strength that lets him actually walk a little better."

"By washing and dressing himself he is building muscle strength that lets him actually walk a little better."

"The client, an 18-year-old female, 5'4'' tall, weighing 113 kg, comes to the clinic for a wound on her lower leg that has not healed for the last two (2) weeks. Which diseaseprocess would the nurse suspect that the client has developed? "1.Type 1 diabetes. 2.Type 2 diabetes. 3.Gestational diabetes. 4.Acanthosis nigricans"

"Correct Answer: 2 Type 2 diabetes is a disorder that usually occurs around the age of 40, but it is now being detected in children and young adultsas a result of obesity and sedentary life-styles. Wounds that do not heal are a hall-mark sign of Type 2 diabetes. This client weighs 248.6 pounds and is short"

Which statement by the patient with type 2 diabetes is accurate. "a. ""I am supposed to have a meal or snak if I drink alcohol"" b. ""I am not allowed to eat any sweets because of my diabetes."" c. I do not need to watch what I eat because my diabetes is not the bad kind."" d. The amunt of fat in my diet is not important; it is just the carbohydrates that raise my blood sugar."""

"Correct Answer: A Alcohol should be consumed with food to reduce the risk of hypoglycemia."

A client with diabetes melllitus has a blood glucose of 644mg/dl. The nurse intreprets that this client is most at risk of developing which type of acid base imbalance? "A. Metabolic acidosis B. Metabolic alkalosis C. Respiratory Acidosis D. Respiratory Alkalosis"

"Correct Answer: A, Metabolic Acidosis Rationale: DM can lead to metabolic acidosis. When the body does not have sufficient circulating insulin, the blood glucose level rises. At the same time, the cells of the body use all available glucose. The body then breaks down glycogen and fat for fuel. The by-products of fat metabolism are acidotic and can lead to the condition known as diabetic ketoacidosis."

"Which of the following persons would most likely be diagnosed with diabetes mellitus? A 44-year-old.. "A. Caucasian Woman B. Asian Woman C. African-American woman D. Hispanic Male

"Correct answer: African-American woman Rationale: Age-specific prevalence of diagnosed diabetes mellitus (DM) is higher for African-Americans and Hispanics than for Caucasians. Among those younger than 75, black women had the highest incidence."

"Excessive thirst and volume of very dilute urine may be symptoms of: "A. Urinary tract infection B. Diabetes insipidus C. Viral gastroenteritis D.Hypoglycemia"

"Correct answer: B Diabetes insipidus is a condition in which the kidneys are unable to conserve water, often because there is insufficient antidiuretic hormone (ADH) or the kidneys are unable to respond to ADH. Although diabetes mellitus may present with similar symptoms, the disorders are different. Diabetes insipidus does not involve hyperglycemia."

"A patient with type 1 diabetes has received diet instruction as part of the treatment plan. The nurse determines a need for additional instruction when the patient says, "a. ""I may have an occasional alcoholic drink if I include it in my meal plan."" b. ""I will need a bedtime snack because I take an evening dose of NPH insulin."" c. ""I will eat meals as scheduled, even if I am not hungry, to prevent hypoglycemia."" d. ""I may eat whatever I want, as long as I use enough insulin to cover the calories.

"D. ""I may eat whatever I want, as long as I use enough insulin to cover the calories."" Rationale: Most patients with type 1 diabetes need to plan diet choices very carefully. Patients who are using intensified insulin therapy have considerable flexibility in diet choices but still should restrict dietary intake of items such as fat, protein, and alcohol. The other patient statements are correct and indicate good understanding of the diet instruction."

"The principal goals of therapy for older patients who have poor glycemic control are: "A. Enhancing quality of life. B. Decreasing the chance of complications. C. Improving self-care through education. D. All of the above."

"D. All of the above. Rationale: The principal goals of therapy for older persons with diabetes mellitus and poor glycemic control are enhancing quality of life, decreasing the chance of complications, improving self-care through education, and maintaining or improving general health status."

"A nurse shoud recognize which symptom as a cardinal sign of diabetes mellitus? "a. Nausea b. Seizure c. Hyperactivity d. Frequent urination

"D. Frequent Urination Polyphagia, polyuria, polydipsia, and weight loss are cardinal signs of DM. Other signs include irritability, shortened attention span, lowered frustration tolerance, fatigue, dry skin, blurred vision, sores that are slow to heal, and flushed skin."

The nurse is reviewing discharge teaching with the client who suffered an anaphylactic reaction to a bee sting. Which statement by the client indicates the need for further teaching? A. "I must wear a medical alert bracelet stating that I am allergic to bee stings." B. "I need to carry epinephrine with me." C. "My spouse must learn how to give me an injection." D. "I am immune to bee stings now that I have had a reaction."

"I am immune to bee stings now that I have had a reaction."

19. Which of the following statements regarding physical activity and its effect on activity tolerance made by a client shows the most informed knowledge regarding the connection between the two? 1. "I know I need to walk more if I want to get stronger." 2. "I don't like walking, but I do it because I know it will make me stronger." 3. "I try to walk a little farther each afternoon so I can dance at my grandson's wedding." 4. "I walk with my son three evenings a week because it's good for his weight and for my bones."

"I try to walk a little farther each afternoon so I can dance at my grandson's wedding."

A patient with a history of mitral valve replacement, hypertension, and type 2 diabetes mellitus undergoes emergency surgery to remove an embolus in her right leg. Which factor contraindicates the use of epidural analgesia in this patient? 1) Anticoagulant therapy 2) Diabetes mellitus 3) Hypertension 4) Embolectomy

*1) Anticoagulant therapy*

29. The nurse is discussing joint mobility exercises with a client who experienced a stroke and now has left-sided weakness. Which of the following statements made by the client reflects the greatest insight regarding the best method for him to maintain mobility of the joints on his left side? 1. "My wife knows how to do those exercises for the joints on my left side." 2."Physical therapy really exercises my left side when I go there every afternoon." 3. "I'll remind the staff to exercise my left side when they come to help me with my bath and getting dressed." 4. "I will do those passive range of motion exercises you taught me to my left side at least 3 times a day."

"I will do those passive range of motion exercises you taught me to my left side at least 3 times a day."

22. A 16-year-old had a full leg cast for 4 months, and it is being removed today. Which of the following statements made by the client shows the most informed understanding of the effects of immobilization of a muscle on its strength and stamina? 1. "I'm hoping to be back at soccer practice in 3 weeks." 2. "Walking and riding my bike will help regain the muscle." 3. "I'll practice the strengthening routine the physical therapist taught me, so I can play baseball in the spring." 4. "There was a good bit of muscle and strength loss, but I'll work at getting it back like it was before the break."

"I'll practice the strengthening routine the physical therapist taught me, so I can play baseball in the spring."

24. Which of the following statements made by a nurse caring for a client who experienced a myocardial infarction 8 hours ago shows the greatest insight as to the purpose for keeping the client on bed rest? 1. "This has been exhausting; she needs a period of uninterrupted rest." 2. "The pain she experienced is exhausting; it's imperative that she rest." 3. "Keeping her on bed rest decreases the need her body has for oxygen" 4. "She needs complete rest; she is really very ill, especially her heart."

"Keeping her on bed rest decreases the need her body has for oxygen"

12. To provide for the psychosocial needs of an immobilized client, an appropriate statement by the nurse is which of the following? 1. "The staff will limit your visitors so that you will not be bothered." 2. "A roommate can be a real bother. You'd probably rather have a private room." 3. "Let's discuss the routine to see if there are any changes we can make." 4. "I think you should have your hair done and put on some makeup."

"Let's discuss the routine to see if there are any changes we can make."

"When an older adult is admitted to the hospital with a diagnosis of diabetes mellitus and complaints of rapid-onset weight loss, elevated blood glucose levels, and polyphagia, the gerontology nurse should anticipate which of the following secondary medical diagnoses? "1.Impaired glucose tolerance 2.Gestational diabetes mellitus 3.Pituitary tumor 4. Pancreatic tumor

"Pancreatic tumor Rationale: The onset of hyperglycemia in the older adult can occur more slowly. When the older adult reports rapid-onset weight loss, elevated blood glucose levels, and polyphagia, the healthcare provider should consider pancreatic tumor."

A 54-year-old patient admitted with type 2 diabetes, asks the nurse what "type 2" means. Which of the following is the most appropriate response by the nurse? "1. ""With type 2 diabetes, the body of the pancreas becomes inflamed." 2. "With type 2 diabetes, insulin secretion is decreased and insulin resistance is increased." 3. "With type 2 diabetes, the patient is totally dependent on an outside source of insulin." 4. "With type 2 diabetes, the body produces autoantibodies that destroy b-cells in the pancreas.""

"Right Answer: 2 Rationale: In type 2 diabetes mellitus, the secretion of insulin by the pancreas is reduced and/or the cells of the body become resistant to insulin"

All congenital heart defects that are trouble start with a ?

"T"

The nursing instructor asks the student nurse to explain a type IV hypersensitivity reaction. Which statement by the student best describes type IV hypersensitivity? A. "It is a reaction of immune globulin (Ig)G with the host cell membrane or antigen." B. "The reaction of sensitized T-cells with antigen and release of lymphokines activate macrophages and induce inflammation." C. "It results in release of mediators, especially histamine, because of the reaction of IgE antibody on mast cells." D. "An immune complex of antigen and antibodies is formed and deposited in the walls of blood vessels."

"The reaction of sensitized T-cells with antigen and release of lymphokines activate macrophages and induce inflammation."

Which of the following factors has the greatest positive effect on sleep quality? 1) Sleeping hours in synchrony with the person's circadian rhythm 2) Sleeping in a quiet environment 3) Spending additional time in stage IV of the sleep cycle 4) Napping on and off during the daytime

*1) Sleeping hours in synchrony with the person's circadian rhythm*

Blood sugar is well controlled when Hemoglobin A1C is... "a. Below 7% b. Between 12%-15% c. Less than 180 mg/dL d. Between 90 and 130 mg/dL"

"a. Below 7% A1c measures the percentage of hemoglobin that is glycated and determines average blood glucose during the 2 to 3 months prior to testing. Used as a diagnostic tool, A1C levels of 6.5% or higher on two tests indicate diabetes. A1C of 6% to 6.5% is considered prediabetes."

"A client who is started on metformin and glyburide would have initially presented with which symptoms? "a. Polydispisa, polyuria, and weight loss b. weight gain, tiredness, & bradycardia c. irritability, diaphoresis, and tachycardia d. diarrhea, abdominal pain, and weight loss

"a. Polydispisa, polyuria, and weight loss"Symptoms of hyperglycemia include polydipsia, polyuria, and weight loss. Metformin and sulfonylureas are commonly ordered medications. Weight gain, tiredness, and bradycardia are symptoms of hypothyroidism. Irritability, diaphoresis, and tachycardia are symptoms of hypoglycemia. Symptoms of Crohn's disease include diarrhea, abdominal pain, and weight loss."

All aminoglycosides end in?

"mycin" Vancomycin

Most second generation tranquilizers have what in them?

"zap"

Benzo's have what in the name?

"zep"

Phenothiazines end in?

"zine"

What is the best rule out of all 4 for prioritization?

#4

To Prevent Dumping Syndrome: Care

(Post-Operatvie Ulcer/Stomach Surgeries) Care Includes: Eat in the reclining position, lie down after meals for 20-30 minutes. Also *restrict fluids during meals, LOW CHO and fiber Diet*, and small frequent meals.

Addison's Disease

(Remember: *DOWN, DOWN, DOWN, UP, DOWN*) - HYPOnatremia, HYPOtension, DECREASED blood volume, HYPERkalemia, and HYPOglycemia

Cushing's Disease

(Remember: *UP, UP, UP, DOWN, UP*) - HYPERnatremia, HYPERtension, INCREASED blood volume, HYPOkalemia, HYPERglycemia

PTT: partial thromboplastin time used in heparin thereapy (lower and upper limit)

(lower limit of normal) 20-25sec (upper limit of normal) 32-39sec

Hg (man and woman)

(man) 13-18 (woman) 12-16

A patient with severe hemorrhoids is incontinent of liquid stool. Which of the following interventions is contraindicated? 1) Apply an indwelling fecal drainage device. 2) Apply an external fecal collection device. 3) Place an incontinence garment on the patient. 4) Place a waterproof pad under the patient's buttocks.

*1) Apply an indwelling fecal drainage device.* An indwelling fecal drainage device is contraindicated for children; for more than 30 consecutive days of use; and for patients who have severe hemorrhoids, recent bowel, rectal, or anal surgery or injury; rectal or anal tumors; or stricture or stenosis. External devices are not typically used for patients who are ambulatory, agitated, or active in bed because the device may be dislodged, causing skin breakdown. External devices cannot be used effectively when the patient has Impaired Skin Integrity because they will not seal tightly. Absorbent products are not contraindicated for this patient unless Impaired Skin Integrity occurs. Even with absorbent products or an external collection device, the nurse should place a waterproof pad under the patient to protect the bed linens.

At last measurement, the client's vital signs were as follows: oral temperature 98°F (36.7°C), heart rate 76, respiratory rate 16, and blood pressure (BP) 118/60. Four hours later, the vital signs are as follows: oral temperature 103.2°F (38.5°C), heart rate 76 beats/minute, respiratory rate 14 breaths/minute, and blood pressure 120/66. Which should the nurse's first intervention be at this time? 1) Ask the client if he has had a warm drink in the last 30 minutes. 2) Notify the primary care provider of the client's temperature. 3) Ask the client if he is feeling chilled. 4) Take the temperature by a different route.

*1) Ask the client if he has had a warm drink in the last 30 minutes.*

In a small rural hospital they work with a wide variety of clients. Of this afternoon client's admitted, the nurse acknowledges the client with the highest susceptibility to infection is the individual with: 1) Burns 2) Diabetes 3) Pulmonary emphysema 4) Peripheral vascular disease

*1) Burns*

The nurse assesses the following changes in a client's vital signs. Which client situation should be reported to the primary care provider? 1) Decreased blood pressure (BP) after standing up 2) Decreased temperature after a period of diaphoresis 3) Increased heart rate after walking down the hall 4) Increased respiratory rate when the heart rate increases

*1) Decreased blood pressure (BP) after standing up* Orthostatic Hypotension

In preventing and controlling the transmission of infections, the single most important technique is: 1) Hand hygiene 2) The use of disposable gloves 3) The use of isolation precautions 4) Sterilization of equipment

*1) Hand hygiene*

The nurse measures the urine output of a patient who requires a bedpan to void. Which action should the nurse take first? Put on gloves and: 1) Have the patient void directly into the bedpan. 2) Pour the urine into a graduated container. 3) Read the volume with the bedpan on a flat surface at eye level. 4) Observe color and clarity of the urine in the bedpan.

*1) Have the patient void directly into the bedpan.* First, the nurse should put on gloves and have the patient void directly into the bedpan. Next, she should pour the urine into a graduated container, place the measuring device on a flat surface, and read the amount at eye level. She should observe the urine for color, clarity, and odor. Then, if no specimen is required, she should discard the urine in the toilet and clean the container and bedpan. Finally, she should record the amount of urine voided on the patient's intake and output record.

A patient tells you that she has trouble falling asleep at night, even though she is very tired. A review of symptoms reveals no physical problems and she takes no medication. She has recently quit smoking, is trying to eat healthier foods, and has started a moderate-intensity exercise program. Her sleep history reveals no changes in bedtime routine, stress level, or environment. Based on this information, the most appropriate nursing diagnosis would be Disturbed Sleep Pattern related to: 1) Increased exercise. 2) Nicotine withdrawal. 3) Caffeine intake. 4) Environmental changes.

*1) Increased exercise.*

The surgeon orders hourly urine output measurement for a patient after abdominal surgery. The patient's urine output has been greater than 60 ml/hour for the past 2 hours. Suddenly the patient's urine output drops to almost nothing. What should the nurse do first? 1) Irrigate the catheter with 30 ml of sterile solution. 2) Replace the patient's indwelling urinary catheter. 3) Infuse 500 ml of normal saline solution IV over 1 hour. 4) Notify the surgeon immediately.

*1) Irrigate the catheter with 30 ml of sterile solution.* If the patient's urinary output suddenly ceases, the nurse should irrigate the urinary catheter to assess whether the catheter is blocked. If no blockage is detected, the nurse should notify the surgeon. The surgeon may request that the catheter be changed if irrigation does not help or if the tubing is not kinked. However, the nurse should not change a catheter in the immediate postoperative period without consulting with the surgeon. The surgeon may prescribe an IV fluid bolus if the patient is suspected to have a deficient fluid volume.

What is typically the most reliable indicator of pain? 1) Patient's self-report 2) Past medical history 3) Description by caregiver(s) 4) Behavioral cues

*1) Patient's self-report*

How should the nurse dispose of the breakfast tray of a patient who requires airborne isolation? 1) Place the tray in a specially marked trash can inside the patient's room. 2) Place the tray in a special isolation bag held by a second healthcare worker at the patient's door. 3) Return the tray with a note to dietary services so it can be cleaned and reused for the next meal. 4) Carry the tray to an isolation trash receptacle located in the dirty utility room and dispose of it there.

*1) Place the tray in a specially marked trash can inside the patient's room.*

A nurse is teaching a group of mothers about first aid. Should poison come in contact with their child's clothing and skin, which action should the nurse instruct the mothers to take first? 1) Remove the contaminated clothing immediately. 2) Flood the contaminated area with lukewarm water. 3) Wash the contaminated area with soap and water and rinse. 4) Call the nearest poison control center immediately.

*1) Remove the contaminated clothing immediately.* Remove contaminated clothing immediately - then wash with water - irrigate it and contact poison control.

Physiological changes associated with aging place the older adult especially at risk for which nursing diagnosis? 1) Risk for Falls 2) Risk for Ineffective Airway Clearance (choking) 3) Risk for Poisoning 4) Risk for Suffocation (drowning)

*1) Risk for Falls* Risk for Falls due to loss of muscle strength and joint mobility

The patient takes anticoagulants. Which instruction is most important for the nurse to include on the patient's care plan? "Teach the patient to: 1) use an electric razor for shaving." 2) apply skin moisturizer." 3) use less soap when bathing." 4) floss teeth daily."

*1) use an electric razor for shaving."* The nurse should instruct the patient prescribed an anticoagulant to use an electric razor instead of a double-edge razor for shaving to prevent the risk of excess bleeding. Older adults should be encouraged to use skin moisturizers and use less soap while bathing to combat excess drying of the skin that occurs as a result of aging. However, even if this patient is an older adult, a risk for bleeding takes priority over a risk for dry skin. Everyone should be encouraged to floss their teeth daily; however, some patients with severe bleeding risk may be told not to floss.

Which of the following statements by the nurse reflects a need for immediate follow-up regarding the physical effects of chronic pain on body function? 1. "His pulse and blood pressure are within his normal baseline limits, so I'm sure the pain medication is working." 2. "Please take his pulse and blood pressure, and let me know if they are elevated above his normal baselines." 3. "If his pulse and blood pressure are above his normal baseline, let me know, and I will medicate him for pain." 4. "Unmanaged pain usually manifests itself in both an elevated pulse and blood pressure."

*1. "His pulse and blood pressure are within his normal baseline limits, so I'm sure the pain medication is working."* Except in cases of severe traumatic pain, which sends a person into shock, most people reach a level of adaptation in which physical signs return to normal. Thus clients in pain will not always have changes in their vital signs. Changes in vital signs are more often indicative of problems other than pain. Although the remaining options recognize the phenomena, they are not assuming that no elevation of vital signs means the absence of pain.

A client who is 2 days' postoperative reports feeling "constipated" to the nurse. The client has good bowel sounds in all four quadrants and has tolerated liquids well. Her pain is being controlled with an opioid analgesic. Which of the following interventions should the nurse try initially? 1. "Let me get you some apple juice." 2. "Ambulating may get your bowels moving." 3. "I'll see about getting a different pain medication." 4. "Your health care provider might prescribe an enema if I call."

*1. "Let me get you some apple juice."*

A 74-year-old client has been having sleeping difficulties. To have a better idea of the client's problem, the nurse should respond: 1. "What do you do just before going to bed?" 2. "Let's make sure that your bedroom is completely darkened at night." 3. "Why don't you try napping more during the daytime?" 4. "Do you eat a small snack before going to bed?"

*1. "What do you do just before going to bed?"* To assess the client's sleeping problem, the nurse should inquire about predisposing factors, such as by asking "What do you do just before going to bed?" Assessment is aimed at understanding the characteristics of any sleep problem and the client's usual sleep habits so that ways for promoting sleep can be incorporated into nursing care. Older adults sleep best in softly lit rooms. Napping more during the daytime is often not the best solution. The nurse should first assess the client's sleeping problem. The client does not always have to eat something before going to bed.

The nurse is discussing a middle-age adult male client's report of nocturia. The client has diabetes that is managed with diet and exercise as well as hypertension that is currently well-controlled with medication. The nurse should include which of the following as possible causes for his frequent urination at night? (Select all that apply.) 1. An enlarged prostate gland 2. Poorly controlled blood glucose 3. Drinking a cup of tea before bed 4. Possible side effect of his medication 5. Taking his diuretic too close to bedtime 6. Consuming too many liquids during the day

*1. An enlarged prostate gland* *2. Poorly controlled blood glucose* *3. Drinking a cup of tea before bed* *5. Taking his diuretic too close to bedtime*

Which of the following symptomatology is reflective of a lower urinary tract infection? (Select all that apply.) 1. Chills and fever 2. Nausea and vomiting 3. Frequency or urgency 4. Cloudy or blood-tinged urine 5. Pelvic tenderness or flank pain 6. Burning or pain when voiding

*1. Chills and fever* *2. Nausea and vomiting* *3. Frequency or urgency* *4. Cloudy or blood-tinged urine* *6. Burning or pain when voiding*

Which of the following statements should the nurse use to instruct the nursing assistant caring for a client with an indwelling urinary catheter? 1. Empty the drainage bag at least every 8 hours. 2. Clean up the length of the catheter to the perineum. 3. Use clean technique to obtain a specimen for culture and sensitivity. 4. Place the drainage bag on the client's lap while transporting the client to testing.

*1. Empty the drainage bag at least every 8 hours.*

Nurses need to implement appropriate body mechanics in order to prevent injury to themselves and their clients. Which principle of body mechanics should the nurse incorporate into client care? 1. Flex the knees and keep the feet wide apart. 2. Assume a position far enough away from the client. 3. Twist the body in the direction of movement. 4. Use the strong back muscles for lifting or moving.

*1. Flex the knees and keep the feet wide apart.* The correct answer is to flex the knees and keep the feet wide apart. This will create a wide base of support, providing greater stability for the nurse and reducing the risk of back injury. The nurse should be positioned close to the client and use the arms and legs. Dividing balanced activity between arms and legs reduces the risk of back injury. Facing the direction of movement prevents abnormal twisting of the spine, also reducing the risk of back injury.

When a client's husband questions how a patient-controlled analgesia (PCA) pump works, the nurse explains that the client: 1. Has control over the frequency of the intravenous (IV) analgesia 2. Can choose the dosage of the drug received 3. May request the type of medication received 4. Controls the route for administering the medication

*1. Has control over the frequency of the intravenous (IV) analgesia* With a PCA system the client controls medication delivery. The PCA system is designed to deliver no more than a specified number of doses. The client does not choose the dosage. The health care provider prescribes the type of medication to be used. The advantage for the client is that he or she may self-administer opioids with minimal risk for overdose. The client does not control the route for administration. Systemic PCA typically involves IV drug administration but can also be given subcutaneously.

Communication involves both active listening and body language working together. The nurse actively listens to the client and: 1. Sits facing the client 2. Keeps the arms and legs crossed 3. Leans back in the chair away from the client 4. Avoids eye contact as much as is physically possible

*1. Sits facing the client* Active listening means to be attentive to what the client is saying both verbally and nonverbally. A nonverbal skill to facilitate attentive listening is to sit facing the client. This posture gives the message that the nurse is there to listen and is interested in what the client is saying. For active listening, the arms and legs should be uncrossed. This posture suggests that the nurse is "open" to what the client says. For active listening, the nurse should lean toward the client. This posture conveys that the nurse is involved and interested in the interaction. For active listening, the nurse should establish and maintain intermittent eye contact. This conveys the nurse's involvement in and willingness to listen to what the client is saying.

When discussing the benefits of physical activity and exercise with a client, the nurse identifies which of the following as a positive outcome to the client? (Select all that apply.) 1. Stress management 2. Enhanced cardiac output 3. Improved bone integrity 4. Facilitation of weight control 5. Increased cognitive function 6. Increased musculoskeletal flexibility

*1. Stress management* *2. Enhanced cardiac output* *3. Improved bone integrity* *4. Facilitation of weight control* *6. Increased musculoskeletal flexibility* Regular physical activity and exercise enhances functioning of all body systems, including cardiopulmonary functioning (endurance), musculoskeletal fitness (flexibility and bone integrity), weight control and maintenance (body image), and psychological well-being. Effects on cognitive function are not consistent.

Which of the following clients presents with an increased risk for urinary incontinence? (Select all that apply.) 1. The 74-year-old diagnosed with parkinsonism 5 years ago 2. The 25-year-old with Crohn's disease diagnosed 4 years ago 3. The 62-year-old Alzheimer's disease client diagnosed 8 years ago 4. The 34-year-old mother of two diagnosed with multiple sclerosis 8 years ago 5. The 73-year-old diagnosed with benign prostatic hyperplasia (BPH) 6 years ago 6. The 69-year-old client diagnosed with type 2 diabetes 9 years ago

*1. The 74-year-old diagnosed with parkinsonism 5 years ago* *3. The 62-year-old Alzheimer's disease client diagnosed 8 years ago* *4. The 34-year-old mother of two diagnosed with multiple sclerosis 8 years ago* *5. The 73-year-old diagnosed with benign prostatic hyperplasia (BPH) 6 years ago* *6. The 69-year-old client diagnosed with type 2 diabetes 9 years ago*

A timed urine specimen collection is ordered. The test will need to be restarted if which of the following occurs? 1. The client voids in the toilet. 2. The urine specimen is kept cold . 3. The first voided urine is discarded. 4. The preservative is placed in the collection container.

*1. The client voids in the toilet.*

A client with a fractured left femur has been using crutches for the past 4 weeks. The physician tells the client to begin putting a little weight on the left foot when walking. Which of the following gaits should the client be taught to use? 1. Two-point 2. Three-point 3. Four-point 4. Swing-through

*1. Two-point* The two-point gait requires at least partial weight bearing on each foot. The client moves a crutch at the same time as the opposing leg, so that the crutch movements are similar to arm motion during normal walking. In a three-point gait, weight is borne on both crutches and then on the uninvolved leg. The four-point gait gives stability to the client but requires weight bearing on both legs. Each leg is moved alternately with each opposing crutch so that three points of support are on the floor at all times. This client is only supposed to use partial weight bearing, so this gait would not be appropriate. Paraplegics who wear weight-supporting braces on their legs use the swing-through gait. It would not be appropriate for this client.

The nurse instructs the client that before the fecal occult blood test (FOBT) she may eat: 1. Whole wheat bread 2. A lean, T-bone steak 3. Veal 4. Salmon

*1. Whole wheat bread*

The nurse instructs a woman about providing a clean catch urine specimen. Which of the following statements indicates that the patient correctly understands the procedure? 1) "I will be sure to urinate into the 'hat' you placed on the toilet seat." 2) "I will cleanse my genital area from front to back before I collect the specimen midstream." 3) "I will need to lie still while you put in a urinary catheter to obtain the specimen." 4) "I will collect my urine each time I urinate for the next 24 hours."

*2) "I will cleanse my genital area from front to back before I collect the specimen midstream."* To obtain a clean catch urine specimen, the nurse should instruct the patient to cleanse the genital area from front to back and collect the specimen midstream. This follows the principle of going from "clean" to "dirty." The nurse should have the ambulatory patient void into a "hat" (container for collecting the urine of an ambulatory patient) when monitoring urinary output, but not when obtaining a clean catch urine specimen. A urinary catheter is required for a sterile urine specimen, not a clean catch specimen. A 24-hour urine collection may be necessary to evaluate some disorders but a clean catch specimen is a one-time collection.

The nurse must administer an enema to an adult patient with constipation. Which of the following would be a safe and effective distance for the nurse to insert the tubing into the patient's rectum? Choose all that apply. 1) 2 inches 2) 3 inches 3) 4 inches 4) 5 inches

*2) 3 inches* *3) 4 inches* When administering an enema, the nurse should insert the tubing about 3 to 4 inches into the patient's rectum. Two inches would not be effective because it would not place the fluid high enough in the rectum. Five inches is too much.

An older adult receiving hospice care has dementia as a result of metastasis to the brain. His bone cancer has progressed to an advanced stage. Why might the client fail to request pain medication as needed? The client: 1) Experiences less pain than in earlier stages of cancer. 2) Cannot communicate the character of his pain effectively. 3) Recalls pain at a later time than when it occurs. 4) Relies on caregiver to provide pain relief without asking.

*2) Cannot communicate the character of his pain effectively.*

The nurse notes that the electrical cord on an IV infusion pump is cracked. Which action by the nurse is best? 1) Continue to monitor the pump to see if the crack worsens. 2) Place the pump back on the utility room shelf. 3) A small crack poses no danger so continue using the pump. 4) Clearly label the pump and send it for repair.

*4) Clearly label the pump and send it for repair.* Label it and take it out of service - all organizations have labels which indicate the equipment is not working. Evaluate the policy to determine if Clinical engineering or biomed needs to be contacted.

The healthcare team suspects that a patient has an intestinal infection. Which action should the nurse take to help confirm the diagnosis? 1) Prepare the patient for an abdominal flat plate. 2) Collect a stool specimen that contains 20 to 30 ml of liquid stool. 3) Administer a laxative to prepare the patient for a colonoscopy. 4) Test the patient's stool using a fecal occult test.

*2) Collect a stool specimen that contains 20 to 30 ml of liquid stool.* To confirm the diagnosis of an infection, the nurse should collect a liquid stool specimen that contains 20 to 30 ml of liquid stool. An abdominal flat plate and a fecal occult blood test cannot confirm the diagnosis. Colonoscopy is not necessary to obtain a specimen to confirm the diagnosis.

When changing a diaper, the nurse observes that a 2-day-old infant has had a green black, tarry stool. What should the nurse do? 1) Notify the physician. 2) Do nothing; this is normal. 3) Give the baby sterile water until the mother's milk comes in. 4) Apply a skin barrier cream to the buttocks to prevent irritation.

*2) Do nothing; this is normal.* During the first few days of life, a term newborn passes green black, tarry stools known as meconium. Stools transition to a yellow green color over the next few days. After that, the appearance of stools depends upon the feedings the newborn receives. Sterile water does nothing to alter this progression. Meconium stools are more irritating to the buttocks than other stools because they are so sticky and the skin usually must be rubbed to cleanse it.

Which one of the following clients would probably have a higher than normal respiratory rate? A client who has: 1) Had surgery and is receiving a narcotic analgesic. 2) Had surgery and lost a unit of blood intraoperatively. 3) Lived at a high altitude and then moved to sea level. 4) Been exposed to the cold and is now hypothermic.

*2) Had surgery and lost a unit of blood intraoperatively.* Hypovolemia / shock. BP decreases, respiratory rate increases

After undergoing dural puncture while receiving epidural pain medication, a patient complains of a headache. Which action can help alleviate the patient's pain? 1) Encourage the client to ambulate to promote flow of spinal fluid. 2) Offer caffeinated beverages to constrict blood vessels in his head. 3) Encourage coughing and deep breathing to increase CSF pressure. 4) Restrict oral fluid intake to prevent excess spinal pressure.

*2) Offer caffeinated beverages to constrict blood vessels in his head.*

While performing a physical assessment, the student nurse tells her instructor that she cannot palpate her patient's bladder. Which statement by the instructor is best? "You should: 1) Try to palpate it again; it takes practice but you will locate it." 2) Palpate the patient's bladder only when it is distended by urine." 3) Document this abnormal finding on the patient's chart." 4) Immediately notify the nurse assigned to your patient."

*2) Palpate the patient's bladder only when it is distended by urine."* The bladder is not palpable unless it is distended by urine. It is not difficult to palpate the bladder when distended. The nurse should document her finding, but it is not an abnormal finding. It is not necessary to notify the nurse assigned to the patient.

Which is the most commonly reported "incident" in hospitals? 1) Equipment malfunction 2) Patient falls 3) Laboratory specimen errors 4) Treatment delays

*2) Patient falls* Patient falls, usually in an attempt to go to the bathroom

A client has severe right-sided weakness and is unable to complete bathing and grooming independently. Based on this observation, the nurse identifies a nursing diagnosis of: 1) Powerlessness 2) Self-care deficit 3) Tissue integrity impairment 4) Knowledge deficit of hygiene practices

*2) Self-care deficit* The client who is unable to complete bathing and grooming independently has a nursing diagnosis of self-care deficit. Being unable to complete bathing and grooming are not defining characteristics for the nursing diagnosis of powerlessness. Being unable to complete bathing and grooming are not defining characteristics for the nursing diagnosis of tissue integrity impairment. There is no indication this client has a knowledge deficit of hygiene practices.

A patient with diarrhea is incontinent of liquid stool. The nurse documents that he now has excoriated skin on his buttocks. Which finding by the nurse led to this documentation? 1) Skin was softened from prolonged exposure to moisture. 2) Superficial layers of skin were absent. 3) Epidermal layer of skin was rubbed away. 4) Lesion caused by tissue compression was present.

*2) Superficial layers of skin were absent.* Excoriation is a loss of the superficial layers of the skin caused by the digestive enzymes in feces. Maceration is the softening of skin from exposure to moisture. Abrasion, a rubbing away of the epidermal layer of the skin, especially over bony areas, is often caused by friction or searing forces that occur when a patient moves in bed. Pressure ulcers are lesions caused by tissue compression and inadequate perfusion that are a result of immobility.

In which situation would using standard precautions be adequate? (Select all that apply.) 1) While interviewing a client with a productive cough 2) While helping a client to perform his own hygiene care 3) While aiding a client to ambulate after surgery 4) While inserting a peripheral intravenous catheter

*2) While helping a client to perform his own hygiene care* *3) While aiding a client to ambulate after surgery* *4) While inserting a peripheral intravenous catheter*

Which of the following statements made by a nurse discussing the effect of an antibiotic on the gastrointestinal system reflects the best understanding of the possible occurrence of diarrhea? 1. "The GI tract naturally rids itself of bacterial toxins by increasing peristalsis, and that causes diarrhea." 2. "The antibiotic is responsible for killing off the GI tract's normal bacterial, and diarrhea is the result." 3. "For some, antibiotics irritate the mucous lining of the intestines, causing decreased absorption and diarrhea." 4. "When you are taking an antibiotic, your body is fighting off an infection, and peristalsis is faster and so diarrhea occurs."

*2. "The antibiotic is responsible for killing off the GI tract's normal bacterial, and diarrhea is the result."*

A patient in the emergency department is angry, yelling, cursing, and waving his arms when the nurse comes to the treatment cubicle. Which action(s) by the nurse are advisable? 1) Reassure the patient by entering the room alone. 2) Ask the patient if he is carrying any weapons. 3) Stay between the patient and the door; keep the door open. 4) Make eye contact while stating firmly "I will not tolerate cursing and threats."

*3) Stay between the patient and the door; keep the door open.* Make sure you do not get trapped. You should never enter the room alone if someone is threatening, the nurse must be calm and reassuring. Asking about weapons and setting limits may escalate the situation.

The nurse and a client are discussing possible behaviors that might be interfering with the client's ability to fall asleep. Which of the following assessment questions is most likely to identify possible problems with the client's sleep routine that possibly are contributing to the difficulty? 1. "When do you usually retire for the night?" 2. "What do you do to help yourself fall asleep?" 3. "How much time does it usually take for you to fall asleep?" 4. "Have you changed anything about your presleep ritual lately?"

*2. "What do you do to help yourself fall asleep?"* As people try to fall asleep, they close their eyes and assume relaxed positions. Stimuli to the RAS decline. If the room is dark and quiet, activation of the RAS further declines. At some point the BSR takes over, causing sleep. If the client engages in activities such as reading or watching television as a means of falling asleep, this could be causing the problem. Although the other questions are not inappropriate, they are not as directed toward the cause of the problem.

Which of the following statements made by an older adult reflects the best understanding of the role of fiber regarding bowel patterns? 1. "The more fiber I eat, the fewer problems I have with my bowels." 2. "Whole grain cereal and toast for breakfast keeps my bowels moving regularly." 3. "My wife makes whole grain muffins; they are really good and good for me too." 4. "I use to have trouble with constipation until I started taking a fiber supplement."

*2. "Whole grain cereal and toast for breakfast keeps my bowels moving regularly."*

The nurse recognizes that the leading cause of death for the otherwise healthy 1-year-old is: 1. Physical abuse 2. Accidental injury 3. Contagious diseases 4. Stranger abduction

*2. Accidental injury* Injuries are the leading cause of death in children older than 1 year of age and cause more deaths and disabilities than do all diseases combined.

Which of the following symptoms should the nurse assess with a client who is deprived of sleep? 1. Elevated blood pressure and confusion 2. Confusion and irritability 3. Inappropriateness and rapid respirations 4. Decreased temperature and talkativeness

*2. Confusion and irritability* Psychological symptoms of sleep deprivation include confusion and irritability. Elevated blood pressure is not a symptom of sleep deprivation. Rapid respirations are not a symptom of sleep deprivation. There may be a decreased ability of reasoning and judgment that could lead to inappropriateness. Decreased temperature is not a symptom of sleep deprivation. The client with sleep deprivation is often withdrawn, not talkative.

The nurse knows that which of the following habits may interfere with a client's sleep? 1. Listening to classical music 2. Finishing office work 3. Reading novels 4. Drinking warm milk

*2. Finishing office work* At home a client should not try to finish office work or resolve family problems before bedtime. Noise should be kept to a minimum. Soft music may be used to mask noise if necessary. Reading a light novel, watching an enjoyable television program, or listening to music helps a person to relax. Relaxation exercises can be useful at bedtime. A dairy product snack such as warm milk or cocoa that contains L-tryptophan may be helpful in promoting sleep.

While undergoing a soapsuds enema, the client complains of abdominal cramping. The nurse should: 1. Immediately stop the infusion 2. Lower the height of the enema container 3. Advance the enema tubing 2 to 3 inches 4. Clamp the tubing

*2. Lower the height of the enema container*

The nurse is working with a client who has left-sided weakness. After instruction, the nurse observes the client ambulate in order to evaluate the use of the cane. Which action indicates that the client knows how to use the cane properly? 1. The client keeps the cane on the left side. 2. Two points of support are kept on the floor at all times. 3. There is a slight lean to the right when the client is walking. 4. After advancing the cane, the client moves the right leg forward.

*2. Two points of support are kept on the floor at all times.* Two points of support, such as both feet or one foot and the cane, should be on the floor at all times. The cane should be kept on the stronger side, the client's right side. The client should keep his or her body upright and midline. Leaning can cause the client to lose his or her balance and fall. After advancing the cane, the client should move the weaker leg, the client's left leg, forward to the cane.

The nurse is caring for a hospitalized client who normally works the night shift at his job. The client states, "I don't know what is wrong with me. I have been napping all day and can't seem to think clearly." The nurse's best response is 1) "You are sleep deprived, but that will resolve in a few days." 2) "You are experiencing hypersomnia, so it will be important for you to walk in the hall more often." 3) "There has been a disruption in your circadian rhythm. What can I do to help you sleep better at night?" 4) "I will notify the doctor and ask him to prescribe a hypnotic medication to help you sleep."

*3) "There has been a disruption in your circadian rhythm. What can I do to help you sleep better at night?"* The data suggests that the patient is used to being awake at night and sleeping during the day. The hospital routine has disrupted this normal pattern.

A client who has been hospitalized for an infection states, "The nursing assistant told me my vital signs are all within normal limits; that means I'm cured." The nurse's best response would be: 1) "Your vital signs confirm that your infection is resolved; how do you feel?" 2) "I'll let your health care provider know so you can be discharged." 3) "Your vital signs are stable, but there are other things to assess." 4) "We still need to keep monitoring your temperature for a while."

*3) "Your vital signs are stable, but there are other things to assess."*

A nurse is teaching wellness to a women's group. The nurse should explain the importance of consuming at least how much fluid to promote healthy bowel function (assume these are 8-ounce glasses)? 1) 2 to 4 glasses a day 2) 4 to 6 glasses a day 3) 6 to 8 glasses a day 4) 8 to 10 glasses a day

*3) 6 to 8 glasses a day* A minimum of 6 to 8 glasses of fluid should be consumed each day to promote healthy bowel function.

Which urine specific gravity would be expected in a patient admitted with dehydration? 1) 1.002 2) 1.010 3) 1.025 4) 1.030

*4) 1.030* Normal urine specific gravity ranges from 1.010 to 1.025. Specific gravity less than 1.010 indicates fluid volume excess, such as when the patient has fluid overload (too much IV fluid) or when the kidneys fail to concentrate urine. Specific gravity greater than 1.025 is a sign of deficient fluid volume that occurs, for example, as a result of blood loss or dehydration.

For which patient can the nurse safely delegate morning care to the nursing assistive personnel (NAP)? Assume an experienced NAP, and base your decision on patient condition. Assume there are no complications other than the conditions stated. 1) 32-year-old admitted with a closed head injury 2) 76-year-old admitted with septic shock 3) 62-year-old who underwent surgical repair of a bowel obstruction 2 days ago 4) 23-year-old admitted with an exacerbation of asthma with dyspnea on exertion

*3) 62-year-old who underwent surgical repair of a bowel obstruction 2 days ago* Morning care for the patient who underwent surgical repair of a bowel 2 days ago can be safely delegated to the nursing assistive personnel because the patient should be stable. The patient who sustained a closed head injury may develop increased intracranial pressure during care. Therefore, he requires the critical thinking skills of a registered nurse to perform his morning care safely. The patient admitted with septic shock may easily become unstable with care; therefore, a registered nurse is required to provide his morning care safely. The patient admitted with an exacerbation of asthma who becomes short of breath with activity also requires the critical thinking skills of a registered nurse to detect respiratory compromise quickly.

Which is a major factor regulating sleep? 1) Electrical impulses transmitted to the cerebellum 2) Level of sympathetic nervous system stimulation 3) Amount of sleep a person has become accustomed 4) Amount of light received through the eyes

*3) Amount of sleep a person has become accustomed to*

Comparing the changes in vital signs as a person ages, which statement(s) is/are correct? (Select all that apply.) 1) Blood pressure decreases less than heart rate and respiratory rate. 2) Respiratory rate remains fairly stable throughout a person's life. 3) Blood pressure increases; heart rate and respiratory rate decline. 4) Men have higher blood pressure than women until after menopause.

*3) Blood pressure increases; heart rate and respiratory rate decline.* *4) Men have higher blood pressure than women until after menopause.*

The nurse is teaching a client who sustained an ankle injury about cold application. Which instruction should the nurse include in the teaching plan? 1) Place the cold pack directly on the skin over the ankle. 2) Apply the cold pack to the ankle for 30 minutes at a time. 3) Check the skin frequently for extreme redness. 4) Keep the cold pack in place for at least 24 hours.

*3) Check the skin frequently for extreme redness.*

The patient is diagnosed with obstructive sleep apnea. Identify the symptoms you would expect the client to exhibit. Choose all that apply. 1) Bruxism 2) Enuresis 3) Daytime fatigue 4) Snoring

*3) Daytime fatigue* *4) Snoring*

What position should the patient assume before the nurse inserts an indwelling urinary catheter? 1) Modified Trendelenburg 2) Prone 3) Dorsal recumbent 4) Semi-Fowler's

*3) Dorsal recumbent* The nurse should have the patient lie supine with knees flexed, feet flat on the bed (dorsal recumbent position). If the patient is unable to assume this position, the nurse should help the patient to a side-lying position. Modified Trendelenburg position is used for central venous catheter insertion. Prone position is sometimes used to improve oxygenation in patients with adult respiratory distress syndrome. Semi-Fowler's position is used to prevent aspiration in those receiving enteral feedings.

Which of the following goals is appropriate for a patient with a nursing diagnosis of Constipation? The patient increases the intake of: 1) Milk and cheese. 2) Bread and pasta. 3) Fruits and vegetables. 4) Lean meats.

*3) Fruits and vegetables.* The nurse should encourage the patient to increase his intake of foods rich in fiber because they promote peristalsis and defecation, thereby relieving constipation. Low-fiber foods, such as bread, pasta, and other simple carbohydrates, as well as milk, cheese, and lean meat, slow peristalsis.

From what stage of sleep are people typically most difficult to arouse? 1) NREM, alpha waves 2) NREM, sleep spindles 3) NREM, delta waves 4) REM

*3) NREM, delta waves*

The nurse is caring for a patient who underwent a bowel resection 2 hours ago. His urine output for the past 2 hours totals 50 mL. Which action should the nurse take? 1) Do nothing; this is normal postoperative urine output. 2) Increase the infusion rate of the patient's IV fluids. 3) Notify the provider about the patient's oliguria. 4) Administer the patient's routine diuretic dose early.

*3) Notify the provider about the patient's oliguria.* 50 mL in two hours is not normal output. The kidneys typically produce 60 ml of urine per hour. Therefore, the nurse should notify the provider when the patient shows diminished urine output (oliguria). Patients who undergo abdominal surgery commonly require increased infusions of IV fluid during the immediate postoperative period. The nurse cannot provide increased IV fluids without a provider's order. The nurse should not administer any medications before the scheduled time without a prescription. The provider may hold the patient's scheduled dose of diuretic if he determines that the patient is experiencing deficient fluid volume.

A client's axillary temperature is 100.8°F. The nurse realizes this is outside normal range for this client, and that axillary temperatures do not reflect core temperature. What should the nurse do to obtain a good estimate of the core temperature? 1) Add 1°F to 100.8°F to obtain an oral equivalent. 2) Add 2°F to 100.8°F to obtain a rectal equivalent. 3) Obtain a rectal temperature reading. 4) Obtain a tympanic membrane reading.

*3) Obtain a rectal temperature reading.*

Which expected outcome is best for the patient with a nursing diagnosis of Acute Pain related to movement and secondary to surgical resection of a ruptured spleen and possible inadequate analgesia? 1) The patient will verbalize a reduction in pain after receiving pain medication and repositioning. 2) The patient will rest quietly when undisturbed. 3) On a scale of 0 to 10, the patient will rate pain as a 3 while in bed or as a 4 during ambulation. 4) The patient will receive pain medication every 2 hours as prescribed.

*3) On a scale of 0 to 10, the patient will rate pain as a 3 while in bed or as a 4 during ambulation.*

A patient with tuberculosis is scheduled for computed tomography (CT). How should the nurse proceed? (Select all that apply.) 1) Question the order because the patient must remain in isolation. 2) Place an N-95 respirator mask on the patient and transport him to the test. 3) Place a surgical mask on the patient and transport him to CT lab. 4) Notify the computed tomography department about precautions prior to transport.

*3) Place a surgical mask on the patient and transport him to CT lab.* *4) Notify the computed tomography department about precautions prior to transport.*

Which of the following statements made by a nurse reflects the best understanding of the role of the bath in the nursing assessment process? 1. "I work with my ancillary staff to be able to determine what is abnormal." 2. "The skin is easy to observe for abnormalities when you are giving the bath." 3. "I use the time to really look at my clients and determine what's normal and what's not." 4. "Bath time is an excellent time to get to know your clients and form that nurse-client relationship."

*3. "I use the time to really look at my clients and determine what's normal and what's not."* Take this time to identify abnormalities and initiate appropriate actions to prevent further injury to sensitive tissues. It also provides an opportunity to assess other systems (e.g., circulatory, respiratory) and client behaviors as well. While the nurse is responsible for determining abnormalities, the ancillary staff should be instructed to report any suspicious factors they note. Answer 3 is the most thorough statement regarding the question.

The nurse should describe pain that is causing the client a "burning sensation in the epigastric region" as: 1. Referred 2. Radiating 3. Deep or visceral 4. Superficial or cutaneous

*3. Deep or visceral* Deep or visceral pain is diffuse and may radiate in several directions. Visceral pain may be described as a burning sensation. Referred pain is felt in a part of the body separate from the source of pain, such as with a myocardial infarction, in which pain may be referred to the jaw, left arm, and left shoulder. Radiating pain feels as though it travels down or along a body part, such as low back pain that is accompanied by pain radiating down the leg from sciatic nerve irritation. Superficial or cutaneous pain is of short duration and is localized as in a small cut.

Which of the following symptoms would the nurse expect with a client who is experiencing acute pain? 1. Bradycardia 2. Bradypnea 3. Diaphoresis 4. Decreased muscle tension

*3. Diaphoresis* An expected assessment finding of a client experiencing acute pain would be diaphoresis resulting from sympathetic nerve stimulation. Additional assessment findings of a client experiencing acute pain would be an increased heart rate, respiratory rate, and muscle tension.

Following an assessment of the client, the nurse identifies the nursing diagnosis activity intolerance related to increased weight gain and inactivity. An outcome identified by the nurse should be: 1. Resting heart rate will be 90 to 100 beats/minute 2. Blood pressure will be maintained between 140/80 and 160/90 mm Hg 3. Exercise will be performed 3 to 4 times over the next 2 weeks 4. Achievement of a rating of 3 for activity endurance

*3. Exercise will be performed 3 to 4 times over the next 2 weeks* An appropriate outcome for activity intolerance related to increased weight gain and inactivity is that the client will perform exercise 3 to 4 times over the next 2 weeks. This outcome is realistic, measurable, and addresses the problem. A resting heart rate of 90-100 beats/minute is too high, and it does not address the need to increase activity. This outcome does not state whether this blood pressure is at rest or after exercising. It also does not address the need to increase activity. A more appropriate outcome is that the client will increase his or her activity (over the next 2 weeks).

While ambulating in the hallway of a hospital, the client complains of extreme dizziness. The nurse, alert to a syncopal episode, should first: 1. Support the client and walk quickly back to the room 2. Lean the client against the wall until the episode passes 3. Lower the client gently to the floor 4. Go for help

*3. Lower the client gently to the floor* If the client has a syncopal episode or begins to fall, the nurse should assume a wide base of support with one foot in front of the other, supporting the client's weight, and then extend the leg, allowing the client to slide against the leg while gently lowering the client to the floor and protecting the client's head. The nurse should not attempt to walk the client quickly back to the room. The nurse should not lean the client against a wall as the client may fall. The nurse should not leave the client alone and go for help.

A priority nursing intervention when caring for a client who is receiving an epidural infusion for pain relief is to: 1. Use aseptic technique 2. Label the port as an epidural catheter 3. Monitor vital signs every 15 minutes 4. Avoid supplemental doses of sedatives

*3. Monitor vital signs every 15 minutes* When clients are receiving epidural analgesia, monitoring occurs as often as every 15 minutes, including assessment of respiratory rate, respiratory effort, and skin color. Complications of epidural opioid use include nausea and vomiting, urinary retention, constipation, respiratory depression, and pruritus. A common complication of epidural anesthesia is hypotension. Assessing vital signs is the priority nursing intervention. Because of the catheter location, strict surgical asepsis is needed to prevent a serious and potentially fatal infection. To reduce the risk for accidental epidural injection of drugs intended for IV use, the catheter should be clearly labeled "epidural catheter." Supplemental doses of opioids or sedative/hypnotics are avoided because of possible additive central nervous system adverse effects.

Which of the following is most appropriate when the nurse assesses the intensity of the client's pain? 1. Ask about what precipitates the pain. 2. Question the client about the location of the pain. 3. Offer the client a pain scale to objectify the information. 4. Use open-ended questions to find out about the sensation.

*3. Offer the client a pain scale to objectify the information.* Descriptive scales are a more objective means of measuring pain intensity. Asking the client what precipitates the pain does not assess intensity, but rather it is an assessment of the pain pattern. Asking the client about the location of pain does not assess the intensity of the client's pain. To determine the quality of the client's pain, the nurse may ask open-ended questions to find out about the sensation experienced.

A nurse who is caring for postoperative clients on a surgical unit knows that for 24 to 48 hours postoperatively, clients who have undergone general anesthesia may experience: 1. Colitis 2. Stomatitis 3. Paralytic ileus 4. Gastrocolic reflex

*3. Paralytic ileus*

Discussing the client's follow-up dietary needs immediately after the surgery when the client is experiencing discomfort is an error in: 1. Pacing 2. Intonation 3. Timing and relevance 4. Denotative meaning

*3. Timing and relevance* Discussing follow-up dietary needs immediately after surgery when the client is experiencing discomfort is an error in timing and relevance. The client is less likely to be able to pay attention and comprehend instruction when in pain, and immediately after surgery, discussing follow-up dietary needs would seem irrelevant. Pacing has to do with the speed of conversation. This is not an example of an error in pacing. Intonation is the tone of voice used. This is not an example of an error in intonation. Denotative meaning is when a single word can have several meanings. This is not an example of an error in denotative meaning.

Which patient teaching would be most therapeutic for someone with sleep disturbance? 1) Give yourself at least 60 minutes to fall asleep. 2) Avoid eating carbohydrates before going to sleep. 3) Catch up on sleep by napping or sleeping in when possible. 4) Do not go to bed feeling upset about a conflict.

*4) Do not go to bed feeling upset about a conflict.*

The nurse is removing personal protective equipment (PPE). Which item should be removed first? 1) Gown 2) Gloves 3) Face shield 4) Hair covering

*4) Hair covering*

The nurse assists a surgeon with central venous catheter insertion. Which action is necessary to help maintain sterile technique? 1) Closing the patient's door to limit room traffic while preparing the sterile field 2) Using clean procedure gloves to handle sterile equipment 3) Placing the nonsterile syringes containing flush solution on the sterile field 4) Remaining 6 inches away from the sterile field during the procedure

*4) Remaining 6 inches away from the sterile field during the procedure*

A patient with a colostomy complains to the nurse, "I am having really bad odors coming from my pouch." To help control odor, which foods should the nurse advise him to consume? 1) White rice and toast 2) Tomatoes and dried fruit 3) Asparagus and melons 4) Yogurt and parsley

*4) Yogurt and parsley* Yogurt, cranberry juice, parsley, and buttermilk may help control odor. White rice and toast (also bananas and applesauce) help control diarrhea. Asparagus, peas, melons, and fish are known to cause odor. Tomatoes, pears, and dried fruit are high-fiber foods that might cause blockage in a patient with an ostomy.

The nurse is completing an assessment of the client's sleep patterns. A specific question that the nurse should ask to determine the potential presence of sleep apnea is: 1. "How easily do you fall asleep?" 2. "Do you have vivid, lifelike dreams?" 3. "Do you ever experience loss of muscle control or falling?" 4. "Do you snore loudly or experience headaches?"

*4. "Do you snore loudly or experience headaches?"* To assess for sleep apnea (unlike assessing for narcolepsy or insomnia), the nurse may ask, "Do you snore loudly?" and "Do you experience headaches after awakening?" A positive response may indicate the client experiences sleep apnea.

An adult client reports to the nurse that she has been experiencing constipation recently and is interested in any suggestions regarding dietary changes she might make. Which of the following suggestions provided by the nurse is most likely to minimize the client's complaint? 1. "Have you tried foods like prunes and bran?" 2. "You might find the new flavored bulk laxatives helpful." 3. "What have you tried in the past that hasn't been helpful?" 4. "Increase your fluid intake; have some juice with breakfast."

*4. "Increase your fluid intake; have some juice with breakfast."*

The nurse is visiting the client who has a nursing diagnosis of urinary retention. Upon assessment the nurse anticipates that this client will exhibit: 1. Severe flank pain and hematuria 2. Pain and burning on urination 3. A loss of the urge to void 4. A feeling of pressure and voiding of small amounts

*4. A feeling of pressure and voiding of small amounts*

The client needs to use crutches at home, and will have to manage going up and down a short flight of stairs. The nurse evaluates the use of an appropriate technique if the client: 1. Uses a banister or wall for support when descending 2. Uses one crutch for support while going up and down 3. Advances the crutches first to ascend the stairs 4. Advances the affected leg after moving the crutches to descend the stairs

*4. Advances the affected leg after moving the crutches to descend the stairs* To descend stairs, the crutches are placed on the stairs and the client moves the affected leg, then the unaffected leg to the stairs with the crutches. The client should continue to use the crutches for support, not the banister or wall. The client should continue to use both crutches for support when going up or down stairs. When ascending stairs, the client moves the unaffected leg up the stair, then the crutches and affected leg.

The unit manager is evaluating the care of a new nursing staff member. Which of the following is an appropriate technique for the nurse to implement in order to obtain a clean-voided urine specimen? 1. Apply sterile gloves for the procedure. 2. Restrict fluids before the specimen collection. 3. Place the specimen in a clean urinalysis container. 4. Collect the specimen after the initial stream of urine has passed.

*4. Collect the specimen after the initial stream of urine has passed.*

Which of the following information provided by the client's bed partner is most associated with sleep apnea? 1. Restlessness 2. Talking during sleep 3. Somnambulism 4. Excessive snoring

*4. Excessive snoring* Partners of clients with sleep apnea often complain that the client's snoring disturbs their sleep. Restlessness is not most associated with sleep apnea. Sleep talking is associated with sleep-wake transition disorders; somnambulism is associated with parasomnias (specifically, arousal disorders and sleep-wake transition disorders).

An ambulatory client is admitted to the extended care facility with a diagnosis of Alzheimer's disease. In using a falls assessment tool, the nurse knows that the greatest indicator of risk is: 1. Confusion 2. Impaired judgment 3. Sensory deficits 4. History of falls

*4. History of falls* According to the falls assessment tool, the greatest indicator of risk is a history of falls. According to the falls assessment tool, the second leading risk factor for falls is confusion. According to the falls assessment tool, impaired judgment is the fourth leading risk factor for falls. According to the falls assessment tool, sensory deficit is the fifth leading risk factor for falls.

The nurse determines that the nursing diagnosis stress urinary incontinence related to decreased pelvic muscle tone is the most appropriate for an oriented adult female client. A therapeutic nursing intervention based on this diagnosis is to: 1. Apply adult diapers 2. Catheterize the client 3. Administer Urecholine 4. Teach Kegel exercises

*4. Teach Kegel exercises*

The nurse recognizes that a client recovering from anesthesia required for surgical repair of a fractured ulna is likely to experience difficulty urinating primarily because of: 1. The impaired cognitive state the client will experience as the effects of the anesthesia wear off 2. The decreased volume of orally ingested fluids before, during, and after the surgical procedure 3. The length of time the client was under the effects of general anesthesia required for the surgical procedure 4. The effects of the anesthetic on the nerves and muscles controlling the relaxation of the urinary bladder

*4. The effects of the anesthetic on the nerves and muscles controlling the relaxation of the urinary bladder*

When obtaining a sterile urine specimen from an indwelling urinary catheter the nurse should: 1. Disconnect the catheter from the drainage tubing 2. Withdraw urine from a urinometer 3. Open the drainage bag and removing urine 4. Use a needle to withdraw urine from the catheter port

*4. Use a needle to withdraw urine from the catheter port*

DO NOT delegate what you can...EAT (Acronym)

*E*valuate, *A*ssess, *T*each

AIRBORN Transmission Precautions: MTV

*M* = Measles *T* = Tuberculosis *V* = Varicella (Chicken Pox)/Herpes Zoster (Shingles) Remember: Private room w/ negative pressure w/ 6 - 12 air exchange/hr AND MASK. *N95* mask for TB.

CONTACT Precautions: MRS WEE

*M* = Multidrug Resistant Organism *R* = Respiratory Infection *S* = Skin Infections *W* = Wound Infections *E* = Enteric Infection (Clostridium Difficile) *E* = Eye Infections (Conjunctivitis)

DROPLET Precautions: SPIDERMAN (Some letters are repeated consecutively)

*S* = Sepsis *S* = Scarlet Fever *S* = Streptococcal Pharyngitis *P* = Paravirus B19 *P* = Pneumonia *P* = Pertussis *I* = Influenza *D* = Diptheria (Pharyngeal) *E* = Epiglottitis *R* = Rubella *M* = Mumps *M* = Meningitis *M* = Mycoplasma or Meningeal Pneumonia *AN* = Adenovirus Remember: Private room or cohort and MASK

SKIN Infections: V CHIPS

*V* = Varicella *C* = Cutaneous Diptheria *H* = Herepes Zoster *I* = Impetigo *P* = Pediculosis *S* = Scabies

In caring for a client with a tracheostomy, the nurse would give priority to the nursing diagnosis of a. Risk for ineffective airway clearance b. Anxiety related to suctioning c. Social isolation related to altered body image d. Impaired tissue integrity

*a. Risk for ineffective airway clearance* While other diagnoses may be applicable, maintaining a patent airway by tending to excessive secretions is a priority.

The client has a draining abdominal wound that has become infected. In caring for the client, the nurse will implement a. contact precautions b. droplet precautions c. no precautions d. airborne precautions

*a. contact precautions* Contact precautions are used when "contact" with the infected drainage could lead to transmission of the infection.

The female client states to the nurse, "I'm so distressed. It seems like every time I laugh hard, I wet myself." The nurse knows that this condition is known as a. stress incontinence b. urge incontinence c. functional incontinence d. unconscious incontinence

*a. stress incontinence* Stress incontinence results from increased pressure within the abdominal cavity.

Mr. Zenobia's chronic cancer pain has recently increased, and he asks the home health nurse what can be done. In relationship to his long-acting morphine, which of the following is an appropriate response by the nurse? a. "If you take more morphine, it will not change your pain relief." b. "I'll call the physician and ask for an increased dose." c. "The amount you are taking now is all I can give you." d. "I'm worried if we increase your dose that you will stop breathing."

*b. "I'll call the physician and ask for an increased dose."* There is no ceiling on the analgesic effect of opioid narcotics. Patients develop a tolerance to the effects, which often necessitates an increase in the dose.

When gathering admission assessment data the nurse obtains a weight of 200 pounds. The client states, "I've never weighed that much!" The nurse should a. Explain to the client how weight gain occurs b. Check the calibration and re-weigh the client c. Document the weight as 200 pounds d. Instruct the UAP to re-weigh the client in 2 hours

*b. Check the calibration and re-weigh the client* It is important to FIRST validate data when there is a mismatch between what the client states as history and the data obtained. Validating data often includes ensuring that equipment is functioning properly first.

The nurse is teaching the client about his upcoming procedure and the client is very stressed. It would be most important for the nurse to a. Use humor first to decrease the client's stress level b. Determine if the teaching should take place at a different time c. Introduce himself as the RN to give credibility to his message d. Speak to the client when family members are there so they can teach the client

*b. Determine if the teaching should take place at a different time* Clients who are stressed may be unable to listen fully and will not receive/understand the intended message.

When administering a drug via a parenteral routes, the drug would be absorbed fastest if given per the IM route. a. True b. False

*b. False* Absorption refers to the "movement" of the drug from the site of administration into the blood stream. Therefore, the intravenous, parenteral route leads to "instant" absorption.

Light sleep and slowing brain and body processes are associated with which stage of NREM sleep? a. I b. II c. III d. IV

*b. II* These are characteristics of a person in Stage II of NREM sleep.

The client calls the nurse to the room and states, "Look, my incision is popping open where they did my hip surgery!" The nurse notes that the wound edges have separated 1 cm at the center and there is straw-colored fluid leaking from one end. The nurse's best action is to a. Notify the surgeon STAT. b. Place a clean, sterile 4 x 4 over the incision and monitor the drainage. c. Wrap an ace bandage firmly around the area and have the client maintain bedrest. d. Immediately cover the wound with sterile towels soaked in normal saline and call the surgeon.

*b. Place a clean, sterile 4 x 4 over the incision and monitor the drainage.* A 1 cm separation of wound edges only in the center of a surgical incision on the hip is too small to truly be termed dehiscence. Even if there were a large separation, there are no "internal viscera" to protrude.

Which of the following actions violates a principle that is key to proper hand washing at the bedside? a. Washing your hands for 1 minute b. Shaking your hands dry over the sink c. Using warm, not very hot water d. Using the soap provided by the agency

*b. Shaking your hands dry over the sink* Shaking your hands will not completely remove the excess moisture, allowing for the reacquisition of bacteria on the area.

During the communication process, "decoding" is a. The selection of words by the sender b. The interpretation of the message by the receiver c. The method by which the message is given d. The way in which feedback is interpreted

*b. The interpretation of the message by the receiver*

To maintain proper posture, it is important to a. sleep on the softest mattress possible b. avoid arching shoulders forward when sitting c. keep your knees locked when standing upright d. keep your stomach muscles relaxed to prevent back spasms

*b. avoid arching shoulders forward when sitting* Arching shoulders forward when sitting alters the curvature of the spine and contributes to poor body alignment.

The term "Kussmaul" refers to a high-pitched, harsh, crowing inspiratory sound that occurs due to partial obstruction of the larynx. a. true b. false

*b. false* The term for this sound of respiratory distress is "stridor."

The nurse is assessing the confused client. In trying to determine the client's level of pain, the nurse should a. be aware that confused clients don't feel as much pain due to their confusion b. observe the client carefully for changes in behavior or vital signs c. ask the client's family how much pain the client normally has d. use only pain scales that feature numbers or "faces" the client can point to

*b. observe the client carefully for changes in behavior or vital signs* The nurse should observe the confused client for nonverbal cues to pain.

Four nurses are inserting catheters in their clients. Which nurse's statement, related to this intervention, is incorrect? I am inserting this catheter to a. empty your bladder prior to your procedure b. treat your problem of leaking urine c. obtain a sterile urine specimen d. measure the amount of urine left after you emptied your bladder

*b. treat your problem of leaking urine* Insertion of a urinary catheter is not a "treatment" for incontinence. "Never event" by CMS - CAUTI

The nurse is assisting the client in caring for her ostomy. The client states, "Oh, this is so disgusting. I'll never be able to touch this thing." The nurse's best response is a. "I'm sure you will get used to taking care of it eventually." b. "Yes, it is pretty messy, so I'll take care of it for you today." c. "It sounds like you are really upset." d. "You sound very angry. Should I call the chaplain for you?"

*c. "It sounds like you are really upset."* This statement reflects the principles of therapeutic communication.

Which diagnostic test/exam would best measure a client's level of hypoxemia? a. chest x-ray b. pulse oximeter reading c. ABG d. peak expiratory flow rate

*c. ABG* The term "hypoxemia" means low blood oxygen level. Arterial blood gas sampling is the most direct way in which the level of oxygen in the blood can be measured.

The client has been on a low-protein diet. This will most likely affect which pharmacokinetic process? a. Absorption b. Excretion c. Distribution d. Metabolism

*c. Distribution* A low-protein diet may lead to an inadequate level of plasma proteins, which will affect availability of "free" drug.

For which sleep disorder would the nurse most likely need to include safety measures in the client's plan of care? a. Snoring b. Enuresis c. Narcolepsy d. Hypersomnia

*c. Narcolepsy* Narcolepsy can occur suddenly during the daytime hours when a person is involved in any type of activity. This could put the person at risk for harm depending on the activity in which he is engaged.

The nurse is completing a head-to-toe assessment on her client at the beginning of the shift for the hospital unit. This would be considered a/an a. Focused assessment b. Initial assessment c. Ongoing assessment d. Special needs assessment

*c. Ongoing assessment* This type of assessment can be completed at any time after the initial assessment. Gathering data at the beginning of a shift will enable the nurse to more effectively evaluate how to proceed with the plan of care for the shift.

Of the following factors, which would put a client at greatest risk for impaired skin integrity? a. the medication digoxin b. moisture c. decreased sensation d. dehydration

*c. decreased sensation* Decreased sensation would greatly increase the risk for injury with a tear or break in the skin. This could lead to a delay in seeking treatment due to lack of awareness.

Of the following interventions for the client who is immobile, the nurse will give priority to a. encouraging a diet high in fiber and extra fluids b. administering the PRN medication for sleep c. having the client use his incentive spirometer q2hrs d. massaging the client's legs every hour

*c. having the client use his incentive spirometer q2hrs* Use of the incentive spirometer helps to prevent atelectasis, which improves oxygenation - a priority need.

It is most important for the nurse to understand the various ways in which pain is classified a. so that he can document the client's pain using accurate terms b. so that he can be clear in his communication with the physician c. so that he can develop an effective pain management plan d. so that he can educate the client thoroughly

*c. so that he can develop an effective pain management plan* ANS: C Different modalities are used in the treatment/ management of pain and are often based on how the pain is classified (e.g., acute vs. chronic).

The primary provider has written a medication prescription. The nurse is having difficulty deciphering what has been written. The best strategy to clarify the information is a. Ask the patient what medication the provider prescribed. b. Call the pharmacist and ask her to read the prescription. c. Ask the nurse who knows the provider's handwriting to read the prescription. d. Call the provider and ask him to clarify the prescription.

*d. Call the provider and ask him to clarify the prescription.* All other answers increase the risk of a medication error.

There is a 24-hour urine collection in process for a client. The NAP inadvertently empties one specimen into the toilet instead of the collection "hat." The nurse should a. Continue with the collection of urine until the 24-hour time period is finished. b. Make a note to the lab to inform them that one specimen was missed during the collection. c. Begin filling a new collection container and take both containers to the lab at the end of the collection period. d. Dispose of the urine already collected and begin an entirely new 24-hour collection.

*d. Dispose of the urine already collected and begin an entirely new 24-hour collection.* Once one specimen is "missed" during a 24-hour urine collection, the results of the lab test will be inaccurate and the collection must be restarted.

Rationale: Individuals who are allergic to kiwis, bananas, pineapples, tropical fruits, grapes, avocados, potatoes, hazelnuts, and water chestnuts are at risk for developing a latex allergy. This is thought to be due to a possible cross-reaction between the food and the latex allergen. The incorrect options are unrelated to latex allergy.

...

Identify the true statement about devices used when assisting clients to ambulate. a. The client should stand a foot back from the back legs of a walker. b. A cane should be used by the client to support the weakest side of the body. c. A transfer belt should be placed around the client's chest for maximum " lift." d. Each crutch-walking "gait" begins with the client in the tripod position.

*d. Each crutch-walking "gait" begins with the client in the The tripod position is the basic crutch standing position from which the client then moves forward.

Use of the statements "Tell me more about..." or "I see" encourage clients to continue talking and expressing themselves. This is called: a. Summarizing b. Open-ended questions c. Focusing d. Encouraging elaboration

*d. Encouraging elaboration*

Mrs. Addie is 70 years old. While the nurse is gathering admission assessment data, the patient states, "I've taken a tablespoon of Milk of Magnesia every day for 3 years." Which nursing diagnosis is most appropriate for the nurse to use in her plan of care? a. Diarrhea b. Constipation c. Risk for Ineffective Therapeutic Regimen d. Perceived Constipation

*d. Perceived Constipation* Daily laxative use by the patient might suggest that she perceives she is constipated, and the nurse would gather further assessment data related to the client's bowel pattern. There is not enough data to infer actual constipation.

The nurse knows that the results of a fecal occult blood test can be inaccurate if a. the client has had an excessive intake of red meat b. the female client is menstruating c. the client takes high doses of vitamin C d. all of the above

*d. all of the above*

Autonomic Dysreflexia/Hyperflexia: S&S and Care

- Acutely, this is a reaction of the autonomic (involuntary) nervous system to overstimulation (due to lesions or injury of the spinal cord) S&S Include: Pounding headache, profuse sweating, nasal congestion, goose flesh, bradycardia, *Hypertension* Care Includes: Place the client in *Sitting Position* (elevate HOB) FIRST before any other implementation.

Focus on the data in the question. Recalling that nausea, vomiting, and abdominal pain are signs of pancreatitis and that pancreatitis is associated with the use of this medication should direct you to the correct option. Review the adverse effects of this medication if you had difficulty with this question.

...

If in a tie never pick magnesium. If the symptom involves nerve or skeletal pick calcemia. For any other symptom pick potassium.

...

If you're being questioned over a drag and drop and you're given an option that would require a doctors order assume that you have that order. But if an option is to call the doctor, you call first then give.

...

Knowing the location of a laminectomy is key to getting the ? right.

...

NPH is not so fast and not in the bag (IV)

...

Rationale: A test done for HIV should be repeated. There might be a lag period after the infection occurs and before antibodies appear in the blood. Therefore a negative HIV test is not considered accurate during the first 6 months after exposure.

...

Rationale: Acquired immunodeficiency syndrome decreases the body's immune response, making the infected person susceptible to infections. AIDS affects helper T lymphocytes, which are vital to the body's defense system. Opportunistic infections are a primary cause of death in people affected with AIDS. Therefore preventing infection is a priority of nursing care. Although the concerns in options 1, 3, and 4 may need to be addressed at some point in the care of the client, these are not the priority.

...

Rationale: All of the conditions identified in the options can occur in rheumatoid arthritis. Ulnar drift occurs when synovitis stretches and damages the tendons, and eventually the tendons become shortened and fixed. This damage causes subluxation (drift) of the joints.

...

Rationale: Cloudy synovial fluid is diagnostic of rheumatoid arthritis. Organisms present in the synovial fluid are characteristic of a septic joint condition. Bloody synovial fluid is seen with trauma. Urate crystals are found in gout.

...

Rationale: For clients with AIDS who experience night fever and night sweats, it is useful to offer an antipyretic at bedtime. It is also helpful to keep a change of bed linens and night clothes nearby for use. The pillow should have a plastic cover, and a towel may be placed over the pillowcase if there is profuse diaphoresis. The client should have liquids at the bedside to drink. Options 1, 2, and 4 are important interventions but they are unrelated to the subject of fever and night sweats.

...

Rationale: Hematological monitoring should be done every 2 weeks in the client taking zidovudine. If severe anemia or severe neutropenia develops, treatment should be discontinued until there is evidence of bone marrow recovery. If anemia or neutropenia is mild, a reduction in dosage may be sufficient. The administration of prednisone may further alter the immune function. Epoetin alfa is given to clients experiencing anemia.

...

Rationale: Individuals at risk for developing a latex allergy include health care workers; individuals who work in the rubber industry; individuals having multiple surgeries; individuals with spina bifida; individuals who wear gloves frequently such as food handlers, hairdressers, and auto mechanics; and individuals allergic to kiwis, bananas, pineapples, tropical fruits, grapes, avocados, potatoes, hazelnuts, and water chestnuts.

...

Rationale: Pancreatitis, which can be fatal, is the major dose-limiting toxicity associated with the administration of didanosine. Clients should be monitored for indications of developing pancreatitis, which include increased serum amylase in association with increased serum triglycerides, decreased serum calcium, and nausea, vomiting, or abdominal pain. If evolving pancreatitis is diagnosed, the medication should be discontinued. The client should be seen by the health care provider.

...

Rationale: Peripheral neuropathy is an adverse effect associated with the use of zalcitabine, which manifests initially as numbness and burning sensations in the extremities. They may progress to sharp shooting pains or severe continuous burning if the medication is not withdrawn. The other options are not associated with use of this medication.

...

Rationale: Providing supportive care with hygiene needs as needed reduces the client's physical and emotional energy demands and conserves energy resources for other functions such as breathing. Options 2, 3, and 4 are important interventions for the client with AIDS but do not address the subject of activities of daily living. Option 2 will assist the client in maintaining appropriate weight and proper nutrition. Option 3 will assist the client in tolerating meals better. Option 4 will decrease the client's risk of infection.

...

Rationale: SLE is a chronic inflammatory disease that affects multiple body systems. A butterfly rash on the cheeks and on the bridge of the nose is a classic sign of SLE. Option 3 is found in sickle cell anemia. Options 1 and 2 may be found in many conditions but are not associated with SLE.

...

Rationale: Skin lesions or rash on the face across the bridge of the nose and on the cheeks is an initial characteristic sign of systemic lupus erythematosus (SLE). Fever and weight loss may also occur. Anemia is most likely to occur later in SLE.

...

Rationale: The client is instructed to avoid spicy, sticky, or excessively hot or cold foods. The client also is instructed to avoid foods that are rough, such as uncooked fruits or vegetables. The client is encouraged to take in foods that are mild, nonabrasive, and easy to swallow. Examples of these include baked fish, noodle dishes, well-cooked eggs, and desserts such as ice cream or pudding. Dry grain foods such as crackers, bread, or cookies may be softened in milk or another beverage before eating.

...

Rationale: The client with AIDS experiencing nausea should avoid fatty products, such as dairy products and red meat. Meals should be small and frequent to lessen the chance of vomiting. Spices and odorous foods should be avoided because they aggravate nausea. Foods are best tolerated either cold or at room temperature.

...

Rationale: The client with HIV is considered to have positive results on Mantoux skin testing with an area of 5 mm of induration or greater. The client without HIV is positive with induration greater than 10 or 15 mm if the client is at low risk. The client with HIV is immunosuppressed, making a smaller area of induration positive for this type of client. It is also possible for the client infected with HIV to have false negative readings because of the immunosuppression factor.

...

Rationale: The client with immunodeficiency has inadequate or an absence of immune bodies and is at risk for infection. The priority problem is infection. The question presents no data indicating that options 2, 3, or 4 are a problem.

...

Rationale: The client with tuberculosis who is coinfected with HIV requires that antitubercular therapy last longer than usual. The prescription is usually for a total of 9 months and at least 6 months after sputum cultures convert to negative.

...

Rationale: The rubber pads used on crutches may contain latex. If the client requires the use of crutches, the nurse can cover the pads with a cloth to prevent cutaneous contact. Option 4 is inappropriate and may alarm the client. The nurse cannot prescribe a cane for a client. In addition, this type of assistive device may not be appropriate, considering this client's injury. No reason exists to contact the HCP at this time.

...

Rationale: Treatment of SLE is based on the systems involved and symptoms. Treatment normally consists of anti-inflammatory drugs, corticosteroids, and immunosuppressants. The incorrect options are not standard components of medication therapy for this disorder.

...

Rationale: When a bee sting occurs and is painful, it is best to treat the site locally rather than systemically. Pain may be alleviated by the application of an ice pack and elevating the site. A heating pad will increase discomfort at the site. Acetaminophen may be taken by the client to assist in alleviating discomfort, but this would not treat the injury at a local level. Lying down and elevating the arm may have some effect on reducing edema at the site but will not directly assist in alleviating the pain at the site of injury.

...

Read pg. 67 for a list of Tyramine

...

Regualr insulin is rapid and run

...

Remember RUN yourself in the GRAVE

...

Review Psych section. Pages 60-64

...

Study Mode Question 53 of 64 A client with acquired immunodeficiency syndrome has a respiratory infection from Pneumocystis jiroveci and a nursing diagnosis of Impaired Gas Exchange written in the plan of care. Which of the following indicates that the expected outcome of care has not yet been achieved? Rationale: The status of the client with a nursing diagnosis of Impaired Gas Exchange would be evaluated against the standard outcome criteria for this nursing diagnosis. These would include that the client breathes easier, coughs up secretions effectively, and has clear breath sounds. The client should not limit fluid intake because fluids are needed to decrease the viscosity of secretions for expectoration.

...

Test-Taking Strategy: Focus on the subject, performing activities of daily living. Options 2, 3, and 4 are important interventions for the client with AIDS but do not address the subject. Option 1 is the only option that addresses the subject of the question. Also note that options 2, 3, and 4 are comparable or alike and relate to nutrition. Review care to the client with AIDS if you had difficulty with this question.

...

Test-Taking Strategy: Knowledge regarding the adverse effects related to the administration of zidovudine is required to answer this question. Focus on the strategic words, "severe neutropenia," to assist in directing you to the correct option. Review the adverse effects of this medication if you had difficulty with this question.

...

Test-Taking Strategy: Note the strategic word, "priority." Use Maslow's Hierarchy of Needs theory to eliminate options 1, 3, and 4. Also, recalling that AIDS affects the body's immune system will direct you to option 2. Review the priority concerns related to the client with AIDS if you had difficulty with this question.

...

Test-Taking Strategy: Recall the classic symptoms of herpes zoster: clear vesicles in groups, pain, and lesions that occur on one side of the body. This will assist in determining that the client has shingles. From this point, recall that this disorder is associated with the chicken pox virus. Review the manifestations of shingles if you had difficulty with this question.

...

Test-Taking Strategy: Use Maslow's Hierarchy of Needs theory to answer the question. Recall that physiological needs are the priority. This will easily direct you to option 1. Review the care of a client with immunodeficiency if you had difficulty with this question.

...

Test-Taking Strategy: Use knowledge related to the effects of AIDS on the gastrointestinal system to assist in answering the question. Additionally, general principles related to nutrition in a client with an immunosuppressive disorder will assist in directing you to option 4. Review dietary measures for the client with AIDS if you had difficulty with this question.

...

Test-Taking Strategy: Use the process of elimination and knowledge regarding the alternative resources for a client with an allergy to latex. No data in the question support the need to contact the HCP. The nurse should not prescribe assistive devices for the client. Option 4 is not a therapeutic action. Review care to the client with a latex allergy if you had difficulty with this question.

...

Test-Taking Strategy: Use the process of elimination and knowledge regarding the food items related to a latex allergy. Eliminate the incorrect options because they are comparable or alike and relate to dairy products. Review the food items that are associated with a risk for latex allergy if you had difficulty with this question.

...

Test-Taking Strategy: Use the process of elimination and knowledge regarding the products that contain latex to answer this question. Eliminate options 1 and 3 first because they are comparable or alike. Noting the strategic words, "cotton" and "silk," in option 4 will assist you in answering correctly from the remaining options. Review the list of products that contain latex if you had difficulty with this question.

...

Test-Taking Strategy: Use the process of elimination and note the strategic words "characteristic sign." Recalling the characteristic butterfly rash associated with SLE will direct you to the correct option. If you are unfamiliar with this disorder, review this content.

...

Test-Taking Strategy: Use the process of elimination to answer the question. Begin by eliminating options 2 and 3 because they are comparable or alike. Remembering that the client with HIV is immunosuppressed will assist in directing you to option 4, the correct option. Review the procedures for interpreting the results of this test if you had difficulty with this question.

...

Test-Taking Strategy: Use the process of elimination, and focus on the subject of difficulty swallowing. Evaluate each of the foods listed in terms of how easily they are swallowed. The rough, hot, and sticky foods in options 1, 2, and 3, respectively, help you to choose option 4 as the correct option. Review nutritional concepts for the AIDS client if you had difficulty with this question.

...

Test-Taking Strategy: Use the process of elimination, and note the strategic words, "expected outcome" and "has not yet been achieved." These words indicate a negative event query and ask you to select an option that is an incorrect statement. This will direct you to option 1. Review care to the client with acquired immunodeficiency syndrome if you had difficulty with this question.

...

Test-Taking Strategy: Use the process of elimination. Eliminate option 1 because SLE is a systemic disorder, not a local one. Next eliminate option 2 because of its similarity to option 1. From the remaining options, select option 3 because of its systemic characteristic. If you are unfamiliar with this disorder, review its characteristics.

...

Test-Taking Strategy: Use the process of elimination. Eliminate options 1 and 2 first because they are comparable or alike. From the remaining options, recalling that antibodies do not appear immediately in the blood will assist in directing you to option 3. Review HIV testing procedures if you had difficulty with this question.

...

Test-Taking Strategy: Use the process of elimination. Knowing that the client with HIV requires longer therapy helps you eliminate options 1 and 2 first. To select between the remaining options, it is necessary to recall that sputum cultures must be negative for 6 months before terminating medication therapy because of the immunosuppressed status of the client. Review the guidelines related to medication therapy in the client with TB if you had difficulty with this question.

...

ationale: The client has the symptoms of herpes zoster, or shingles, which is caused by the same organism as chicken pox. Asking about sexual partners is inappropriate for this disorder. An electric blanket use does not cause this type of lesions. Abreva is used on herpes simplex I (cold sores).

...

est-Taking Strategy: Focus on the subject of the question, which is night fever and night sweats. Options 1, 2, and 4 are helpful and important interventions but do not address the subject of the question. Because night fever and sweats occur serially, it is helpful to give the antipyretic before sleep as a prophylactic measure. Review nursing measures for the client with AIDS if you had difficulty with this question.

...

est-Taking Strategy: Use the process of elimination, noting the strategic word, "first." Focus on the subject, that the site of the bee sting is painful. Eliminate option 1 because this measure produces a systemic and not a local effect. Eliminate option 4 next because this measure will not directly assist in alleviating the pain at the site of injury. From the remaining options, recalling the effects of heat will assist in eliminating option 2 and direct you to option 3. Review the initial measures that will alleviate pain from a bee sting if you had difficulty with this question.

...

se the process of elimination. Remember that organisms indicate infection, blood indicates trauma, and urates indicate gout. Review the characteristics of rheumatoid arthritis if you had difficulty with this question.

...

Latent phase is?

0-4 cm dilated Contractions are 5-30min apart Lasting 15-30 sec. They are mild

conjugated (direct) bilirubin

0.1-0.4

unconjugated (indirect) bilirubin

0.1-0.4

total bilirubin

0.3-1

Hypotonic fluids

0.45% NS

Creatnine norm?

0.6-1.2

Lithium therapeutic level?

0.6-1.2

Creatinine

0.7-1.5

1 fingerwidth is how many cm's?

1

For a preschooler ____ meal a day is ok!!!

1

How often do you have to draw a WBC for a patient taking a second generation tranquilizer?

1 a week for 1 month 1 a month for 6 months 1Q6 months for life

Beta Blockers B1 vs B2

1 heart 2 lungs

Regular insulin onset?

1 hour

1st trimester weight gain?

1 lb a month

"Which of the following is accurate pertaining to physical exercise and type 1 diabetes mellitus? "1. Physical exercise can slow the progression of diabetes mellitus. 2. Strenuous exercise is beneficial when the blood glucose is high. 3. Patients who take insulin and engage in strenuous physical exercise might experience hyperglycemia. 4. Adjusting insulin regimen allows for safe participation in all forms of exercise."

1) physical exercise can slow the progression of diabetes mellitusRationale: Physical exercise slows the progression of diabetes mellitus, because exercise has beneficial effects on carbohydrate metabolism and insulin sensitivity. Strenuous exercise can cause retinal damage, and can cause hypoglycemia. Insulin and foods both must be adjusted to allow safe participation in exercise.

A nurse is caring for a cient with type 1 diabetes mellitus. which client complaint would alert the nurse to the presence of a possible hypoglycemic reaction? "1. Tremors 2. Anorexia 3. Hot, dry skin 4. Muscle cramps

1) tremorsdecreased blood glucose levels produce autonomic nervous system symptoms, which are manifested classically as nervousness, irritability, and tremors. option 3 is more likely for hyperglycemia, and options 2 and 4 are unrelated to the signs of hypoglycemia.

"A client is admitted to the hospital with signs and symptoms of diabetes mellitus. Which findings is the nurse most likely to observe in this client? Select all that apply: "1. Excessive thirst 2. Weight gain 3. Constipation 4. Excessive hunger 5. Urine retention 6. Frequent, high-volume urination

1, 4, 6 Rationale: Classic signs of diabetes mellitus include polydipsia (excessive thirst), polyphagia (excessive hunger), and polyuria (excessive urination). Because the body is starving from the lack of glucose the cells are using for energy, the client has weight loss, not weight gain. Clients with diabetes mellitus usually don't present with constipation. Urine retention is only a problem is the patient has another renal-related condition.

Type II diabetics need how many calories a day?

1,200-1,800

Lanoxin (digoxin) therapeutic level?

1-2

The daughter of an 88-year-old female client tells the nurse that her mother has recently quit going on walks in the neighborhood because of pain in her legs. Which of the following is the best response from the nurse? 1. "I would like to speak with your mother to get more information." 2. "Older people frequently suffer from arthritis that can cause leg pain." 3. "Your mother probably has poor circulation in her legs, which is causing the pain." 4. "She is lucky to be as healthy as she is at her age."

1. " I would like to speak with your mother to get information."

The nursery nurse is explaining postcircumcision care to a new mother. Which of the following statements by the new mother indicates that additional teaching needs to occur? 1. "Babies don't experience pain, so I don't need to worry about hurting him when I touch the penis." 2. "I need to be careful not to put his diaper on too tight to avoid discomfort." 3. "I can comfort my baby following the procedure by holding him." 4. "The health care provider will numb the area before performing the procedure."

1. "Babies don't experience pain, so i don't need to worry about hurting him when i touch the penis."

Which of the following statements by the nurse reflects a need for immediate follow-up regarding the physical effects of chronic pain on body function? 1. "His pulse and blood pressure are within his normal baseline limits, so I'm sure the pain medication is working." 2. "Please take his pulse and blood pressure, and let me know if they are elevated above his normal baselines." 3. "If his pulse and blood pressure are above his normal baseline, let me know, and I will medicate him for pain." 4. "Unmanaged pain usually manifests itself in both an elevated pulse and blood pressure."

1. "His pulse and blood pressure are within his normal baseline limits, so i'm sure the pain medication is working"

Benzo's can also be used for what 5 other reasons?

1. Anesthesia induction 2. Muscle relaxant 3. Alcohol withdrawal 4. Seizures 5. Facilitates mechanical ventilation

"What will the nurse teach the client with diabetes regarding exercise in his or her treatment program? 1. During exercise the body will use carbohydrates for energy production, which in turn will decrease the need for insulin. 2. With an increase in activity, the body will use more carbohydrates; therefore more insulin will be required. 3. The increase in activity results in an increase in the use of insulin; therefore the client should decrease his or her carbohydrate intake. 4. Exercise will improve pancreatic circulation and stimulate the islets of Langerhans to increase the production of intrinsic insulin.

1. During exercise the body will use carbohydrates for energy production, which in turn will decrease the need for insuli"Rationale: As carbohydrates are used for energy, insulin needs decrease. Therefore during exercise, carbohydrate intake should be increased to cover the increased energy requirements. The beneficial effects of regular exercise may result in a decreased need for diabetic medications in order to reach target blood glucose levels. Furthermore, it may help to reduce triglycerides, LDL cholesterol levels, increase HDLs, reduce blood pressure, and improve circulation."

4. Which of the following factors has an impact on the severity of physical impairment a client will experience from a period of immobility? (Select all that apply.) 1. The client's age 2. Prior overall health 3. Length of immobility 4. The degree of immobility 5. Situation requiring the inactivity 6. Client's mental attitude about the limitations

1. The client's age 2. Prior overall health 3. Length of immobility 4. The degree of immobility

5. A client who experienced a myocardial infarction has been placed on bed rest. The nurse caring for the client recognizes that the inactivity will result in certain assessment findings that include: (Select all that apply.) 1. Lethargy 2. Confusion 3. Depression 4. Poor appetite 5. Hypoactive bowel sounds 6. Decrease in baseline respiratory rate

1. Lethargy 4. Poor appetite 5. Hypoactive bowel sounds 6. Decrease in baseline respiratory rate

3. A 16-year-old has had a full leg cast in place for 2 months, and it is being removed today. Which of the following assessment findings would be expected following the removal of the cast? (Select all that apply.) 1. Popliteal pulse equal in both legs 2. Slight footdrop noted on affected leg 3. Swelling noted at ankle on affected leg 4. Weight bearing less stable on affected leg 5. Calf circumference greater in unaffected leg 6. Greater range of motion of knee of unaffected leg

1. Popliteal pulse equal in both legs 4. Weight bearing less stable on affected leg 5. Calf circumference greater in unaffected leg 6. Greater range of motion of knee of unaffected leg

Nurses working with clients in pain need to recognize and avoid common misconceptions and myths about pain. In regard to the pain experience, which of the following is correct? 1. The client is the best authority on the pain experience. 2. Chronic pain is mostly psychological in nature. 3. Regular use of analgesics leads to drug addiction. 4. The amount of tissue damage is accurately reflected in the degree of pain perceived.

1. The client is the best authority on the pain experience

A 44-year-old client shares with the admitting nurse that the client is having epigastric pain that the client identifies as a 7 on a 0 to 10 scale. In order to plan for the pain management of this client, which is the most appropriate response from the nurse? 1. "What would be a satisfactory level of pain control for us to achieve?" 2. "You don't look like you're in that much pain." 3. "You'll be pain-free following your surgery." 4. "I've cared for a client with a nail in his head who only rated his pain as a 5; are you sure your pain is a 7?"

1."What would be a satisfactory level of pain control for us to achieve?"

specific gravity

1.010-1.030

Magnesium (Mg)

1.3-2.3

Bleeding time: used in preop screening

1.5-9.5 min

creatinine clearance 24 hr urine collection - creatinine level after end of urine collection

1.67-2.5

The greatest barrier to a 3-year-old client's ability to self-assess her pain is: 1. A limited vocabulary 2. Increased separation anxiety 3. Reluctance to talk to strangers 4. Inability to grasp the concept of pain

1.A limited vocabulary

When using the even gaits what one is for severe and what one for mild problems?

2 point gait for mild 4 point gait for severe.

Graves disease treatment options are? (3)

1.Radiation with I131 (radioactive iodine) 1st 24hrs be alone and flush the toilet alot. 2. PTU(drug) worry about immunosuppresion. 3. Surgical removal.

A client who had knee replacement surgery the previous day refuses to take any pain medication, even though he rates his pain as an 8 on a 0 to 10 scale. Upon questioning the client the nurse learns that the reason for refusing pain medication is because he is concerned about injuring the knee and not feeling it. The best information that the nurse can provide this client is to explain that: 1. The pain medication will help speed his recovery time 2. He need not worry about becoming addicted to the pain medication 3. He will not be perceived as weak for taking the pain medication 4. He is being a difficult client and needs to comply with the health care provider's orders

1.The pain medication will help speed his recovery time

Macrodrips

10, 12, or 15 drops per milliliter used for rapid administration

Hypertonic fluids

10-15% dextrose in water 3% NS Sodium Bicarb 5%

Aminophylline therapeutic level?

10-20

Dilantin therapeutic level?

10-20

Elevated bilirubin level?

10-20 neonate only

ALT: alanine aminotransferase

10-40

AST: aspartate aminotransferase

10-40

ANS: D The most important intervention is to avoid prolonged pressure on bony prominences by frequent repositioning. The other interventions also may be included in family teaching, but the most important instruction is to change the patient's position at least every 2 hours.

10. A patient who is confined to bed and who has a stage II pressure ulcer is being cared for in the home by family members. To prevent further tissue damage, the home care nurse instructs the family members that it is most important to a. change the patient's bedding frequently. b. use a hydrocolloid dressing over the ulcer. c. record the size and appearance of the ulcer weekly. d. change the patient's position at least every 2 hours.

ANS: A Mechanical debridement with wet-to-dry dressings is painful, and patients should receive pain medications before the dressing change begins. The new dressings are moistened with saline before being applied to the wound. Soaking the old dressings before removing them will eliminate the wound debridement that is the purpose of this type of dressing. Application of antimicrobial ointments is not indicated for a wet-to-dry dressing.

11. Which nursing action will be included when the nurse is doing a wet-to-dry dressing change for a patient's stage III sacral pressure ulcer? a. Administer the ordered PRN oral opioid 30 minutes before the dressing change. b. Soak the old dressings with sterile saline a few minutes before removing them. c. Pour sterile saline onto the new dry dressings after the wound has been packed. d. Apply antimicrobial ointment before repacking the wound with moist dressings.

Fundal height is not palpable until week?

12

NPH duration?

12 hours

Bartholemew's Rule of forths: fundus height by weeks

12 wks: symphisis pubis 16 wks: midway between symphisis pubis and umbilicus 20 wks: umbilicus 36 wks: xiphoid process

Normal Hgb level in women is?

12-16

Hgb norm?

12-18

ANS: D Pressure ulcers should not be cleaned with solutions that are cytotoxic, such as hydrogen peroxide. The other actions by the new graduate are appropriate.

12. The charge nurse observes a new graduate performing a dressing change on a stage II left heel pressure ulcer. Which action by the new graduate indicates a need for further education about pressure ulcer care? a. The new graduate uses a hydrocolloid dressing (DuoDerm) to cover the ulcer. b. The new graduate inserts a sterile cotton-tipped applicator into the pressure ulcer. c. The new graduate irrigates the pressure ulcer with a 30-ml syringe using sterile saline. d. The new graduate cleans the ulcer with a sterile dressing soaked in half-strength peroxide.

ANS: A Soft tissue injuries are treated with rest, ice, compression, and elevation (RICE). Elevation of the ankle will decrease tissue swelling. Moving the ankle through the ROM will increase swelling and risk further injury. Cold packs should be applied the first 24 hours to reduce swelling. The soccer shoe does not need to be removed immediately and will help to compress the injury if it is left in place.

13. A patient arrives in the emergency department with a swollen ankle after an injury incurred while playing soccer. Which action by the nurse is appropriate? a. Elevate the ankle above heart level. b. Remove the patient's shoe and sock. c. Apply a warm moist pack to the ankle. d. Assess the ankle's range of motion (ROM).

Sodium norm?

135-145

sodium (Na)

135-145

ANS: D Chronic corticosteroid use will interfere with wound healing. The persistence of the ulcers over the last 6 months is a concern, but changes in care may be effective in promoting healing. Keloids are not disabling or painful, although the cosmetic effects may be distressing for some patients. Actions to reduce the patient's pain will be implemented, but pain does not impact directly on wound healing.

14. When admitting a patient with stage III pressure ulcers on both heels, which information obtained by the nurse will have the most impact on wound healing? a. The patient states that the ulcers are very painful. b. The patient has had the heel ulcers for the last 6 months. c. The patient has several old incisions that have formed keloids. d. The patient takes corticosteroids daily for rheumatoid arthritis.

Humalog onset?

15 minutes

How long do you wash for handwashing?

15 seconds

When do you draw a IV peak level?

15-30 minutes after dose is finished.

blood ammonia

15-45

ANS: D Even a low fever in an immunosuppressed patient is a sign of serious infection and should be treated immediately with cultures and rapid initiation of antibiotic therapy. The nurse should assess the other patients as soon as possible after assessing and implementing appropriate care for the immunosuppressed patient.

15. The nurse has just received change-of-shift report about the following four patients. Which patient will the nurse assess first? a. The patient who has multiple black wounds on the feet and ankles. b. The newly admitted patient with a stage IV pressure ulcer on the coccyx. c. The patient who needs to be medicated with multiple analgesics before a scheduled dressing change. d. The patient who has been receiving immunosuppressant medications and has a temperature of 102° F.

Platelets norm?

150,000-400,000

platelet ct

150,000-450,000

total cholesterol

150-200

ANS: D LPN education and scope of practice include sterile dressing changes for stable patients. Initial wound assessments, patient teaching, and evaluation for possible poor wound healing or infection should be done by the RN.

16. Which of these four patients should the medical-surgical unit charge nurse assign to an LPN team member? a. The patient who has increased tenderness and swelling around a leg wound. b. The patient who has just arrived after suturing of a full-thickness arm wound. c. The patient who needs teaching about home care for a draining abdominal wound. d. The patient who requires a hydrocolloid dressing change for a Stage III sacral ulcer.

ANS: D Wound separation at a week postoperatively indicates possible wound dehiscence and should be immediately reported to the health care provider. The other findings also will be reported, but do not require intervention as rapidly.

17. When caring for a diabetic patient who had abdominal surgery one week ago, the nurse obtains these data. Which finding should be reported immediately to the health care provider? a. Blood glucose 136 mg/dl b. Oral temperature 101° F (38.3° C) c. Patient complaint of increased incisional pain d. New 5-cm separation of the proximal wound edges

ANS: A Elevated blood glucose will have an impact on multiple factors involved in wound healing. Ensuring adequate nutrition also is important for the postoperative patient, but a higher priority is blood glucose control. A temperature of 102° F will not impact adversely on wound healing, although the nurse may administer antipyretics if the patient is uncomfortable. Application of a dry, sterile dressing daily may be ordered, but frequent dressing changes for a wound healing by primary intention is not necessary to promote wound healing.

18. A diabetic patient is admitted for a laparotomy and possible release of adhesions. When planning interventions to promote wound healing, the nurse's highest priority will be a. maintaining the patient's blood glucose within a normal range. b. ensuring that the patient has an adequate dietary protein intake. c. giving antipyretics to keep the temperature less than 102° F (38.9° C). d. redressing the surgical incision with a dry, sterile dressing twice daily.

"The nurse administered 28 units of Humulin N, an intermediate-acting insulin, to a client diagnosed with Type 1 diabetes at 1600. Which action should the nurse implement? "1. Ensure the client eats the bedtime snack. 2. Determine how much food the client ate at lunch. 3. Perform a glucometer reading at 0700. 4. Offer the client protein after administering insulin.

1: ensure the client eats the bedtime snack"1. Humulin N peaks in 6-8 hours, making the client at risk for hypoglycemia around midnight, which is why the client should receive a bedtime snack. This snack will prevent nighttime hypoglycemia. (Correct) 2. The food intake at lunch will not affect the client's blood glucose level at midnight. 3. The client's glucometer reading should be done around 2100 to assess the effectiveness of insulin at 1600. 4. Humulin N is an intermediate-acting insulin that has an onset in 2-4 hours but does not peak until 6-8 hours."

Pre-op cervical laminectomy's most important assessment is?

1st breathing (rate and rhythm) 2nd arm and motor sensory

Pre-op thoracic laminectomy's most important assessment is?

1st cough (uses abdominal muscles) 2nd bowel sounds

Nageles rule for calculating a due date is?

1st day of the last menstrual period Add 7 days Subtract 3 months

Pre-op lumbar laminectomy's most important assessment is?

1st voiding ( when was last time, can they) 2nd leg motor and sensory.

The nurse knows that a PCA pump would be most appropriate for the client who: 1. Has psychogenic discomfort 2. Is recovering after a total hip replacement 3. Experiences renal dysfunction 4. Recently experienced a cerebrovascular accident (stroke)

2. Is recovering after a total hip replacement

Regular insulin peak?

2 hours

INR therapeutic level?

2-3

ANS: A The shift to the left indicates that the patient probably has a bacterial infection, and the nurse will plan to obtain wound cultures. Antibiotic therapy and/or dressing changes may be started, but cultures should be done first. The nurse will continue to monitor the wound, but additional actions are needed as well.

2. A patient with an open abdominal wound has a complete blood cell (CBC) count and differential, which indicate an increase in white blood cells (WBCs) and a shift to the left. The nurse anticipates that the next action will be to a. obtain wound cultures. b. start antibiotic therapy. c. redress the wound with wet-to-dry dressings. d. continue to monitor the wound for purulent drainage.

A nurse is preparing a teaching plan for a client with diabetes Mellitus regarding proper foot care. Which instruction is included in the plan? 1. Soak feet in hot water 2. apply a moisturizing lotion to dry feet but not between the toes 3. Always have a podiatrist cut your toenails, never cut them yourself 4. avoid using mild soap on the feet

2. The client is instructed to use a moisturizing lotion on the feet and to avoid applying the lotion between the toes.

The nurse caring for a terminally ill client with liver cancer understands which of the following goals would be most appropriate? 1. Increasingly administer narcotics to oversedate the client and thereby decrease the pain. 2. Continue to change the analgesics until the right narcotic is found that completely alleviates the pain. 3. Adapt the analgesics as the nursing assessment reveals the need for specific medications. 4. Withhold analgesics because they are not being effective in relieving discomfort.

3. Adapt the analgesics as the nursing assessment reveals the need for specific medications.

When caring for a client who is experiencing continuous severe pain, the nurse should expect that the pain management plan would include: 1. Focusing on intramuscular administration of analgesics 2. Waiting for pain to become more intense before administering opioids 3. Administering opioids with nonopioid analgesics for severe pain experiences 4. Administering large doses of opioids initially to clients who have not taken the medications before

3. Administering opioids with nonopioid analgesics for severe pain experiences

Which of the following is the most appropriate nursing intervention for a client who is receiving epidural analgesia? 1. Change the tubing every 48 to 72 hours. 2. Change the dressing every shift. 3. Secure the catheter to the outside skin. 4. Use a bulky occlusive dressing over the site.

3. Secure the catheter to the outside skin

A client with type I diabetes is placed on an insulin pump. The most appropriate short-term goal when teaching this client to control the diabetes is: "1) adhere to the medical regimen 2) remain normoglycemic for 3 weeks 3) demonstrate the correct use of the administration equipment. 4) list 3 self care activities that are necessary to control the diabetes"

3.) is correct "1) this is not a short-term goal 2) this is measurable, but it's a long-term goal 3) this is a short-term goal, client oriented, necessary for the client to control the diabetes, and measurable when the client performs a return demonstration for the nurse 4) although this is measurable and a short-term goal, it is not the one with the greatest priority when a client has an insulin pump that must be mastered before discharge"

"The nurse is teaching a class on atherosclerosis. Which statement describes the scien-tific rationale as to why diabetes is a risk factor for developing atherosclerosis? (1.Glucose combines with carbon monoxide, instead of with oxygen, and this leads tooxygen deprivation of tissues.2.Diabetes stimulates the sympathetic nervous system, resulting in peripheralconstriction that increases the development of atherosclerosis.3.Diabetes speeds the atherosclerotic process by thickening the basement membraneof both large and small vessels.4.The increased glucose combines with the hemoglobin, which causes deposits of plaque in the lining of the vessels.

3.Diabetes speeds the atherosclerotic process by thickening the basement membraneof both large and small vessels."1.Glucose does not combine with carbonmonoxide.2.Vasoconstriction is not a risk factor for devel-oping atherosclerosis. 3.This is the scientific rationale why diabetesmellitus is a modifiable risk factor for atherosclerosis. 4.When glucose combines with the hemoglobinin a laboratory test called glycosylated hemo-globin, the result can determine the client'saverage glucose level over the past three (3)months"

A client with chronic pain presents in the emergency department of the local hospital stating "I just can't take this anymore." On questioning the client, the nurse discovers that the client have experienced chronic pain since being involved in an accident 2 years previously. The client states that he has been labeled a "drug seeker" because he is looking for relief for the pain and feels hopeless, angry, and powerless to do anything about the situation. The nurse understands that this client is at risk for: 1. Criminal activity 2. Opioid abuse 3. Suicide 4. Drug addiction

3.Suicide

When do you draw a IM peak level?

30-60 minutes after given

The nurse is discussing the effects of pain with an older adult client diagnosed with osteoarthritis. The most therapeutic response to the client's comment of, "I wonder whether it would hurt if I took a nap in the afternoon?" would be: 1. "As long as it did not interfere with your getting a good night's sleep." 2. "I'd suggest taking your nap right after you take your pain medication." 3. "If it helps you cope better with the pain, I don't see any harm in taking a nap." 4. "I think a nap is a good idea because we seem to feel pain more when we are tired."

4. " I think a nap is a good idea because we seem to feel pain more when we are tired."

Which of the following statements is the most appropriate response to a client's statement, "I thought you could tell I was in pain"? 1. "How do you express a need for pain medication if not by asking?" 2. "I'm so very sorry; may I get you your pain medication right now? 3. "I don't think it's wise to assume I can effectively read your mind regarding the need for pain medication." 4. "I will make a point of asking you to rate your pain at least every 2 hours, so this miscommunication won't happen again."

4. " I will make a point of asking you to rate your pain at least every 2 hours, so this miscommunication won't happen again."

Which of the following statements made by a nurse caring for a client reporting severe pain expresses the most insight into how pain impacts a client's energy reserves? 1. "If I can't get his pain under control, his recovery will take a lot longer." 2. "Pain certainly interferes with the client's ability to rest and recuperate." 3. "I'm going to call for another pain prescription so he can get some rest." 4. "Trying to cope with pain is using up the energy that his recovery requires."

4. " Trying to cope with pain is using up the energy so he can get some rest."

Which of the following statements made by the nurse regarding the client's self-assessment of pain requires immediate follow-up regarding the personal nature of pain? 1. "The medication should be providing enough relief; try to ambulate her." 2. "I've never known anyone to have such pain after that procedure." 3. "He should be able to ambulate with only minimal pain by now." 4. "She says she's in pain, but she doesn't act like she is in pain."

4. "She says she's in pain, but she doesn't act like she is in pain."

Taking into consideration the hospice client's chronic pain from bone cancer, the most appropriate person to collaborate with regarding management of pain is: 1. Occupational therapist to devise a splint for the client's leg 2. Physical therapist to determine exercises to strengthen the leg muscles 3. Art therapist to provide creative therapy as a diversion 4. An oncology nurse

4. An oncology nurse

The nurse is attempting to ambulate an older adult client who recently experienced a fall at the assisted living facility where he resides. The client is reluctant to walk and consents to move only to the chair, reporting that "it hurts too much to walk." Which of the following nursing interventions is most therapeutic regarding this client? 1. Allow the client to remain in bed in order to conserve his energy. 2. Transfer him to the chair, realizing some activity is preferable to none. 3. Call his health care provider to discuss the apparent ineffectiveness of his pain medications. 4. Assess the client for other factors that may be affecting his ability and motivation to ambulate.

4. Assess the client for other factors that may be affecting his ability and motivation to ambulate

A client with diabetes mellitus demonstratees acute anxiety when first admitted for the treatment of hyperglycemia. The most appropriate intervention to decrease the client's anxiety would be to 1. administer a sedative 2. make sure the client knows all the correct medical terms to understand what is happening 3. ignore the signs and symptoms of anxiety so that they will soon disappear 4. convey empathy, trust, and respect toward the client

4. The most appropriate intervention is to address the client's feelings related to the anxiety

A 38-year-old client presents to the pain clinic with complaints of phantom pain. The client was involved in a farming accident 3 years previously that resulted in a below-the-elbow amputation of his right arm. The nurse knows that phantom pain is categorized as: 1. Painful polyneuropathy 2. Somatic pain 3. Sympathetically maintained pain 4. Deafferentation pain

4.Deafferentation pain

"The nurse is discussing the importance of exercising to a client diagnosed with Type 2diabetes whose diabetes is well controlled with diet and exercise. Which informationshould the nurse include in the teaching about diabetes? "1.Eat a simple carbohydrate snack before exercising. 2.Carry peanut butter crackers when exercising. 3.Encourage the client to walk 20 minutes three (3) times a week. 4.Perform warmup and cooldown exercises

4.Perform warmup and cooldown exercises "The client diagnosed with Type 2 diabetes whois not taking insulin or oral agents does notneed extra food before exercise.2.The client with diabetes who is at risk forhypoglycemia when exercising should carry asimple carbohydrate, but this client is not atrisk for hypoglycemia.3.Clients with diabetes that is controlled by dietand exercise must exercise daily at the sametime and in the same amount to control theglucose level. 4. [correct] All clients who exercise should perform warmup and cooldown exercises to helpprevent muscle strain and injury"

How much K+ can you have per liter of IV fluid?

40 mEq

ANS: C The wound requires debridement of the necrotic areas and absorption of the yellow-green slough. A hydrocolloid dressing such as DuoDerm would accomplish these goals. Transparent film dressings are used for red wounds or approximated surgical incisions. Dry dressings will not debride the necrotic areas. Nonadherent dressings will not absorb wound drainage or debride the wound.

5. A patient's 6 3-cm leg wound has a 2-mm black area surrounded by yellow-green semiliquid material. Which dressing will the nurse use for wound care? a. Dry gauze dressing (Kerlix) b. Nonadherent dressing (Xeroform) c. Hydrocolloid dressing (DuoDerm) d. Transparent film dressing (Tegaderm)

For a laminectomy what incision site has the highest risk for infection?

50/50 equal

Type II diabetics need how many feedings a day?

6

NPH onset?

6 hours

If being asked about a surgery and the length of time for restrictions your default answer should be?

6 weeks

Must not take Benzo's for longer than?

6 weeks

amylase

6-160

ANS: B The description is consistent with a yellow wound. A stage III pressure wound would expose subcutaneous fat. A red wound would not have any creamy colored exudate. A full-thickness wound involves subcutaneous tissue, which is not indicated in the wound description.

6. A 76-year-old patient has an open surgical wound on the abdomen that contains a creamy exudate and small areas of deep pink granulation tissue. The nurse documents the wound as a a. red wound. b. yellow wound. c. full-thickness wound. d. stage III pressure wound.

Microdrips

60 drops per millilter more controlled smaller fluid volumes (compromised renal or cardiac, keep-open rates, or pediatrics)

fasting blood glucose

60-110

PO2 norm?

78-100

32. The nurse caring for a 73-year-old female client who has been hospitalized with a stroke instructs the client's daughter to continue to do passive range-of-motion exercises with her mother on her affected side to prevent contractures. The nurse explains to the daughter that this is very important in an immobile older adult client because contractures can form in as little as: 1. 8 hours 2. 24 hours 3. 1 week 4. 1 month

8 hours

Transition phase is?

8-10 cm dilated Contractions are 2-3 min apart Lasting 60-90 seconds They are strong

NPH peak?

8-10 hours

BUN norm?

8-30

ANS: D Wet-to-dry dressings are used when there is minimal eschar to be removed. A full-thickness wound filled with eschar will require interventions such as surgical debridement to remove the necrotic tissue. Wet-to-dry dressings are not needed on approximated surgical incisions. Wet-to-dry dressings are not used on uninfected granulating wounds because of the damage to the granulation tissue.

8. The nurse will plan to use wet-to-dry dressings when providing care for a patient with a a. pressure ulcer with pink granulation tissue. b. surgical incision with pink, approximated edges. c. full-thickness burn filled with dry, black material. d. wound with purulent drainage and dry brown areas.

PaO2

85-95

Study Mode Question 23 of 64 A client with human immunodeficiency virus (HIV) who has contracted tuberculosis (TB) asks the nurse how long the medication therapy lasts. The nurse responds that the duration of therapy would likely be for at least: 6 total months and at least 1 month after cultures convert to negative 6 total months and at least 3 months after cultures convert to negative 9 total months and at least 3 months after cultures convert to negative 9 total months and at least 6 months after cultures convert to negative

9 total months and at least 6 months after cultures convert to negative

ANS: C A stage III pressure ulcer has full-thickness skin damage and extends into the subcutaneous tissue. A stage I pressure ulcer has intact skin with some observable damage such as redness or a boggy feel. Stage II pressure ulcers have partial-thickness skin loss. Stage IV pressure ulcers have full-thickness damage with tissue necrosis, extensive damage, or damage to bone, muscle, or supporting tissues.

9. A patient is admitted to the hospital with a pressure ulcer on the left buttock. The base of the wound is yellow and involves subcutaneous tissue. The nurse classifies the pressure ulcer as stage a. I. b. II. c. III. d. IV.

PT: prothrombin time used in coumadin therapy

9.5-12 sec

Contractions should be no longer then ____ and no closer then_____.

90 sec and 2 minutes

O2 sat norm?

93-100

SaO2

95-99%

CD4 norm?

< 200 is AIDS

BNP norm?

<100

For injections an IM needle must be?

A 1 in both gauge and length.

17. It has been determined that all of the following clients are at risk for falling. Which one requires the nurse's priority for ambulation? 1. A 16-year-old with a sprained ankle being discharged from the emergency department 2. A 54-year-old who has taken the initial dose of an antihypertensive medication 3. A 45-year-old postoperative client up for the first time since knee surgery 4. An 81-year-old who is asthmatic and had a hip replaced 18 months ago

A 45-year-old postoperative client up for the first time since knee surgery

For injections SUBQ needles must have?

A 5 in both gauge and length.

What is a delusion?

A false fixed belief, idea or thought. This has no sensory component.

Asystole is?

A lack of QRS repolarizations

A client reports to the health care clinic to obtain testing regarding human immunodeficiency virus (HIV) status after being exposed to an individual who is HIV positive. The test results are reported as negative, and the client tells the nurse that he feels so much better knowing that he had not contracted HIV. The nurse explains the test results to the client, telling the client that: There is no further need for testing. A negative HIV test is considered accurate. A negative HIV test is not considered accurate during the first 6 months after exposure. The test should be repeated in 1 week.

A negative HIV test is not considered accurate during the first 6 months after exposure.

What are the 2 lethal arrythmias?

A-systole and V-Fib

What are the 6 arrythmias for NCLEX in order for prioritization?

A-systole, V-fib, V-tach, A-fib, A-flutter and PVC

which are symptoms of hypoglycemia? A. irritability, B. drowsiness c. Abdominal pain D. nausea and vomiting

A. Irritability: signs of hypoglycemia include irritability, shaky feeling, hunger, headache, dizziness. Other symptoms are hyperglycemia.

Hypertension treatment: ABCD

A: ACE inhibitors/ARBs B: beta blockers C: calcium channel blockers D: diuretics

"The risk factors for type 1 diabetes include all of the following except: "a. Diet b. Genetic c. Autoimmune d. Environmental"

A: Type 1 diabetes is a primary failure of pancreatic beta cells to produce insulin. It primarily affects children and young adults and is unrelated to diet.

Management of ASTHMA

A: adrenergics (albuterol) S: steroids T: theophylline H: hydration (IV) M:mask (oxygen) A: antibiotics (infection)

Secondary causes of hypertension: ABCDE

A: aldosterone/apnea B: bad kidney/bruits C: catecholamines/cushings D: drugs/diet E: endocrine

Lead Poisoning symptoms: ABCDEFG

A: anemia B: basophilic stripping C: colicky pain, constipation D: difficulty concentrating E: encephalopathy F: foot drop G: gum (lead line)

Anticholinergic side effects: ABCDs

A: anorexia B: blurry vision C: consitpation/confusion D: dry mouth S: stasis of urine

Aspirin side effects: ASPIRIN

A: asthma S: salicyalism P: peptic ulcer disease I: intestinal blood loss R: reye's syndrome I:idiosyncracy N: noise (tinnitus)

Melanoma characteristics: ABCDE

A: asymetrical B: borders irregular C: color dark and variation D: diameter is large (>6mm) E: evolving

Prozac has the same side effects as Elavil?

ABCDE

Haldol has the SE?

ABCDEFG

"pril" - benazepril, captopril, enalapril, lisinopril, moexipril, ramipril

ACE inhibitor - antihypertensive

With Addison's Disease you ADD what?

ADD-A-SONE

Cells attacked by AIDS/HIV

AIDS is 4 letters long - attack CD4 T cells

If you don't know what a drug is and you are being asked which lab is important.... you need to remember?

ALT A L-iver T-est

What is the best indicator of CHF?

ANF

For prioritization the question will give what 4 pieces of information?

Age Gender Disease Modifying phrase

For a prioritization question what 2 pieces of information do not matter?

Age and gender

Clozaril(Clozapine) has what SE?

Agranulocytosis

What two words mean the same thing as immunosuppresion?

Agranulocytosis and neutropenia.

What is the first thing you do in a Med-Surg situation?

Airway

Aminophylline is an?

Airway antispasm

Vitamin B1 helps breakdown?

Alcohol

If the pH is up it is?

Alkalosis

Droplet precautions are for what 2 diseases?

All meningitis and all influenza

Probable/presumptive signs of pregnancy are?

All urine and blood tests

The nurse plans to assess the client with type I hypersensitivity for which clinical manifestation? A. Poison ivy B. Autoimmune hemolytic anemia C. Allergic asthma D. Rheumatoid arthritis

Allergic asthma

Teach a patient taking Antabuse to avoid what?

Alochol

Type I: Rapid Hypersensitivity Reactions

Also called atopic allergy; most common type Reaction of IgE antibody on mast cells with antigen, which results in release of mediators, especially histamine Ex. Hay fever, Allergic asthma, Anaphylaxis

Allergic Rhinitis

Also known as hay fever Triggered by reaction to airborne allergens: Plant pollens; Molds; Dust; Animal dander; Wool, food ; Air pollutants

36. The nurse understands that using metabolic functioning, measures of height, weight, and skinfold thickness, to evaluate muscle atrophy in an immobilized client is known as: 1. Anthropometric measurements 2. Anhydrous measurements 3. Balke test 4. Calorimetry

Anthropometric measurements

Benzodiazepines are ?

Antianxiety meds.

Dilantin is an?

Anticonvulsant

Small doses of phenothiazines are?

Antiemetics

Calcium Channel Blockers treat what? (the 6 A's)

Antihypertensive, Anti-Anginal, Anti Atrial Arrythmia and SVTS

What is the second best toy for a 6-9 month old?

Anything large that they can't swallow.

What is the 2nd best toy for a 0-6 month old?

Anything soft and large

What is the second best toy for a 9-12 month old?

Anything that is purposeful (rolling a ball back and forth)

APGAR: A (1st A)

Appearance - color pink/pink and blue/blue [pale]

11. Antiembolic stockings (thromboembolic device [TED] hose) are ordered for the client on bed rest following surgery. The nurse explains to the client that the primary purpose for the TEDs is to: 1. Keep the skin warm and dry 2. Prevent abnormal joint flexion 3. Apply external pressure 4. Prevent bleeding

Apply external pressure

A client calls the emergency department and tells the nurse that he received a bee sting to the arm while weeding a garden. The client states that he has received bee stings in the past and is not allergic to bees. The client states that the site is painful and asks the nurse for advice to alleviate the pain. The nurse tells the client to first: Take two acetaminophen (Tylenol). Place a heating pad to the site. Apply ice and elevate the site. Lie down and elevate the arm.

Apply ice and elevate the site.

The middle-aged client, who is alert, is admitted to the emergency department (ED) with wheezing, difficulty breathing, angioedema, blood pressure (BP) of 70/52, and apical pulse of 122 and irregular. The nurse makes an immediate assessment using the ABCs for any client experiencing anaphylaxis. What nursing intervention is the immediate priority? A. Raise the lower extremities. B. Start intravenous (IV) administration of normal saline. C. Reassure the client that appropriate interventions are being instituted. D. Apply oxygen using a high-flow non-rebreather mask at 40% to 60%.

Apply oxygen using a high-flow non-rebreather mask at 40% to 60%.

The nurse is assessing a client with suspected serum sickness. Which symptoms will be consistent with serum sickness? Select all that apply. A. Arthralgia B. Blurred vision C. Lymphadenopathy D. Malaise E. Ptosis

Arthralgia Lymphadenopathy Malaise

A home health nurse is at the home of a client with diabetes and arthritis. The client has difficulty drawing up insulin. It would be most appropriate for the nurse to refer the client to: "A) A social worker from the local hospital B) An occupational therapist from the community center C) A physical therapist from the rehabilitation agency D) Another client with diabetes mellitus and takes insulin"

B) An occupational therapist can assist a client to improve the fine motor skills needed to prepare an insulin injection.

CO2 in the 60's (respiratory failure) you?

Assess respiratory status Do pursed lip breathing Prepare for intubation Call respiratory therapy Call the doctor

CO2 is the 50's you?

Assess respiratory status Do pursed lip breathing ^ exhale time DON't give O2 if the above isn't working call the doctor!

O2 sat below 93 you?

Assess respiratory status Give O2

PO2 70-77 you?

Assess respiratory status Give O2

PO2 below 60 you?

Assess respiratory status Give O2 Prepare for intubation Call respiratory therapy Call the doctor

2. A 61-year-old client recently suffered left-sided paralysis from a cerebrovascular accident (stroke). In planning care for this client, the nurse implements which one of the following as an appropriate intervention? 1. Encourage an even gait when walking in place. 2. Assess the extremities for unilateral swelling and muscle atrophy. 3. Encourage holding the breath frequently to hyperinflate the client's lungs. 4. Teach the use of a two-point crutch technique for ambulation.

Assess the extremities for unilateral swelling and muscle atrophy.

If the potassium is below 3.5 you?

Assess the heart Prepare to give potassium Call the doctor

4. A client is leaving for surgery and because of preoperative sedation needs complete assistance to transfer from the bed to the stretcher. Which of the following should the nurse do first? 1. Elevate the head of the bed. 2. Explain the procedure to the client. 3. Place the client in the prone position. 4. Assess the situation for any potentially unsafe complications.

Assess the situation for any potentially unsafe complications.

If the pH is under 6 you?

Assess vital signs Call the doctor ASAP

One of the benefits of Glargine (Lantus) insulin is its ability to: "a.Release insulin rapidly throughout the day to help control basal glucose.b. Release insulin evenly throughout the day and control basal glucose levels.c. Simplify the dosing and better control blood glucose levels during the day.d. Cause hypoglycemia with other manifestation of other adverse reactions.

B)Release insulin evenly throughout the day and control basal glucose levels"Glargine (Lantus) insulin is designed to release insulin evenly throughout the day and control basal glucose levels.

The nurse is making an occupied bed. Arrange the following steps in the order the nurse should perform them. A. Position the patient laterally near the side rail farthest from you (that side rail is up); roll the soiled linens under him. B. Lower the side rail on the side of the bed you are working on. C. Raise the side rail on the side of the bed you are working on. D. After placing clean linens and tucking them under the soiled linens, roll the patient over the "hump" and position him facing you on the near side of the bed.

B. Lower the side rail on the side of the bed you are working on. A. Position the patient laterally near the side rail farthest from you (that side rail is up); roll the soiled linens under him. D. After placing clean linens and tucking them under the soiled linens, roll the patient over the "hump" and position him facing you on the near side of the bed. C. Raise the side rail on the side of the bed you are working on. First lower the side rail on your side of the bed. This allows you to maintain good body mechanics while positioning the patient (in step 1). Position patient laterally near far side rail, and roll soiled linens under him. Then place clean linens on the side nearest you, and tuck them under soiled linens. Next, roll the patient over the "hump," and position him on his other side, facing you. Do this before raising the near side rail so you do not have to reach across the side rail to help the patient roll and turn to his other side.

When the nurse walks into the patient's room, she notices fire coming from the patient's trash can. Rank the following actions in the order they should be performed by the nurse. A. Activate the fire alarm. B. Move the patient out of the room. C. Close all doors and windows. D. Put out the fire using the proper extinguisher.

B. Move the patient out of the room. A. Activate the fire alarm. C. Close all doors and windows. D. Put out the fire using the proper extinguisher. R.A.C.E. - rescue, alarm, contain, and exstinquish or evacuate

"The guidelines for Carbohydrate Counting as medical nutrition therapy for diabetes mellitus includes all of the following EXCEPT: a. Flexibility in types and amounts of foods consumed b. Unlimited intake of total fat, saturated fat and cholesterol c. Including adequate servings of fruits, vegetables and the dairy group d. Applicable to with either Type 1 or Type 2 diabetes mellitusb. Unlimited intake of total fat, saturated fat and cholesterol"

B. You want to be careful of how much you eat in any food group.

Causes of Stevens-Johnson syndrome: BAPS H&M

B: barbiturates A: antibiotics P: phenytoin and other anticonvulsants S: sulfonamides H: herpes simplex M: mycoplasms

ADLs: BATTED

B: bathing A: ambulation T: toileting T: transfers E: eating D: dressing

SNRI side effects: BAD SNRI

B: body wt increase A: anorexia D: dizziness S: suicidal thoughts N: nausea/vomiting R: reproductive/sexual dysfunction I: insomnia

Low fetal heart rate is ?

Bad you LION under 110

What do you do if the patients disconnected tube is on the floor?

Bag them, (call for help) get new tube and then reconnect.

The home care nurse is collecting data from a client who has been diagnosed with an allergy to latex. In determining the client's risk factors associated with the allergy, the nurse questions the client about an allergy to which food item? Eggs Milk Yogurt Bananas

Bananas

How should you prioritize labs?

Based on what the level will do to the body and not the disease it's associated with.

Hemangiomas is?

Benign tumor of the capillaries.

What are the two neonatal lung medications?

Betamethasone Survanta

The nurse prepares to administer zafirlukast (Accolate) to a client with allergic rhinitis. The nurse understands that zafirlukast works by which mechanism? A. Blocking histamine from binding to receptors B. Preventing synthesis of mediators C. Preventing mast cell membranes from opening D. Blocking the leukotriene receptor

Blocking the leukotriene receptor

A nurse is collecting data on a client who complains of fatigue, weakness, malaise, muscle pain, joint pain at multiple sites, anorexia, and photosensitivity. Systematic lupus erythematosus (SLE) is suspected. The nurse further checks for which of the following that is also indicative of the presence of SLE? Emboli Ascites Two hemoglobin S genes Butterfly rash on cheeks and bridge of nose

Butterfly rash on cheeks and bridge of nose

Common Cold is caused by CREAR

C: coronavirus R: rhinoviruses E: enterovirus A: adenovirus R: RSV

A client is brought to the emergency department in an unresponsive state, and a diagnosis of hyperglycemic hyperosmolar nonketotic syndrome is made. The nurse would immediately prepare to initiate which of the following anticipated physician's prescriptions? 1. Endotracheal intubation 2. 100 units of NPH insulin 3. Intravenous infusion of normal saline 4. Intravenous infusion of sodium bicarbonate

CORRECT ANSWER: 3. Intravenous infusion of normal saline Rationale: The primary goal of treatment is hyperglycemic hyperosmolar nonketotic syndrome (HHNS) is to rehydrate the client to restore the fluid volume and to correct electrolyte deficiency. Intravenous fluid replacement is similar to that administered in diabetic keto acidosis (DKA) and begins with IV infusion of normal saline. Regular insulin, not NPH insulin, would be administered. The use of sodium bicarbonate to correct acidosis is avoided because it can precipitate a further drop in serum potassium levels. Intubation and mechanical ventilation are not required to treat HHNS.

If kernicterus and opisthotonos are occuring you?

Call doctor, drae bilirubin level, increase the IV rate and start billi lights.

SE of Pitocin?

Can cause hyperstimulation (contractions longer than 90 sec. and closer then 2 min.)If FHR is less then 110 stop it, if FHR is normal you slow it. Used also for PP Hemorrhage.

Hgb level in the third trimester?

Can fall to 10 and is normal.

Hgb level in second trimester?

Can fall to 10.5 and is normal.

Hgb level in first trimester?

Can fall to 11 and it is normal.

After Thyroidectomy: Care

Care Includes: Position *LOW* or *semi - fowlers*, support the head, neck and shoulders.

Infant w/ Cleft Lip: Care

Care Includes: Position on the back or in an infant seat to prevent trauma to the suture line. While feeding, hold in the upright position.

Ace of Spades answer for OB?

Check the FHR.

What causes angina?

Chest pain due to O2 supply and demand issues.

The ACE of spades answer for nutrition is to either pick ____ or _____.

Chicken Fish

What do you use for hand scrubbing?

Cleaning agent must have the prefix "chlor"

A health care provider aspirates synovial fluid from a knee joint of a client with rheumatoid arthritis. The nurse reviews the laboratory analysis of the specimen and would expect the results to indicate which finding? Cloudy synovial fluid Presence of organisms Bloody synovial fluid Presence of urate crystals

Cloudy synovial fluid

What is preschooler (3-6 yr.) play characterized by?

Co-operative play (together)

A client with acquired immunodeficiency syndrome (AIDS) is taking didanosine (Videx). The client calls the nurse at the health care provider's office and reports nausea, vomiting, and abdominal pain. Which of the following instructions would the nurse provide to the client? This is an expected side effect of the medication. Come to the office to be seen by the health care provider. Take crackers and milk with the administration of the medication. Decrease the dose of the medication until the next health care provider's visit.

Come to the office to be seen by the health care provider.

Assessment findings reveal that the client admitted to the hospital has a contact type I hypersensitivity to latex. Which preventive nursing intervention is best in planning care for this client? A. Report the need for desensitization therapy. B. Convey the need for pharmacologic therapy to the health care provider. C. Communicate the need for avoidance therapy to the health care team. D. Discuss symptomatic therapy with the health care provider.

Communicate the need for avoidance therapy to the health care team.

A 7-11 yr old is in what Piaget stage?

Concrete operation (think of a 7-11 with concrete around it)

At what Piaget stage can you teach a skill like how to draw up insulin?

Concrete operations

To treat denial you need to?

Confront

Allow adolescents to be in each others rooms unless?

Contagious Immunosupressed Fresh post-op

6. The nurse assesses that the client has torticollis and that this may adversely influence the client's mobility. This individual has a(n): 1. Exaggeration of the lumbar spine curvature 2. Increased convexity of the thoracic spine 3. Abnormal anteroposterior and lateral curvature of the spine 4. Contracture of the sternocleidomastoid muscle with a head incline

Contracture of the sternocleidomastoid muscle with a head incline

"Which of the following factors are risks for the development of diabetes mellitus? (Select all that apply.) "a) Age over 45 years b) Overweight with a waist/hip ratio >1 c) Having a consistent HDL level above 40 mg/dl d) Maintaining a sedentary lifestyle

Correct: a,b,d"Rationale: Aging results in reduced ability of beta cells to respond with insulin effectively. Overweight with waist/hip ratio increase is part of the metabolic syndrome of DM II. There is an increase in atherosclerosis with DM due to the metabolic syndrome and sedentary lifestyle.

The home care nurse is ordering dressing supplies for a client who has an allergy to latex. The nurse asks the medical supply personnel to deliver which of the following? Elastic bandages Adhesive bandages Brown Ace bandages Cotton pads and silk tape

Cotton pads and silk tape

INR monitors?

Coumadin/Warafin therapy

What do you do if a staff members behavior is legal, not harmful but just inappropriate?

Counsel them later at a better time.

What 4 things do you do if a chest tube comes out?

Cover hole with a gloved hand, put on a vaseline gauze dressing, put on sterile dressing ands then tape on 3 sides.

A nurse is assigned to care for a client who returned home from the emergency department following treatment for a sprained ankle. The nurse notes that the client was sent home with crutches that have rubber axillary pads and needs instructions regarding crutch walking. On data collection, the nurse discovers that the client has an allergy to latex. Before providing instructions regarding crutch walking, the nurse should: Contact the health care provider (HCP). Cover the crutch pads with cloth. Call the local medical supply store, and ask for a cane to be delivered. Tell the client that the crutches must be removed immediately from the house.

Cover the crutch pads with cloth.

What is the best toy for a 6-9 month old?

Cover/uncover toy

School age (7-11 yr.) is characterized by the 3 C's, what are they?

Creative (no coloring book, use blank paper) Collecting Competitive

Best indicator of kidney function?

Creatnine

To treat V-fib you ?

Defibrillate ... For V-fib you D-fib

How do you treat type I diabetes? (DIE)

Diet 3 Insulin 1 Exercise 2

A client with acquired immunodeficiency syndrome (AIDS) is taking zidovudine (Retrovir) 200 mg orally three times daily. The client reports to the health care clinic for follow-up blood studies, and the results of the blood studies indicate severe neutropenia. Which of the following would the nurse anticipate to be prescribed for the client? Reduction in the medication dosage Discontinuation of the medication The administration of prednisone concurrent with the therapy Administration of epoetin alfa (Epogen)

Discontinuation of the medication

A client who is receiving an intravenous antibiotic begins to cough and states, "My throat feels like it is swelling." Which action will the nurse take next? A. Infuse normal saline at 200 mL/hr. B. Administer epinephrine (Adrenalin) 1:1000, 0.3 mL subcutaneously. C. Discontinue infusing the antibiotic. D. Give diphenhydramine (Benadryl) 100 mg IV.

Discontinue infusing the antibiotic.

Milia is?

Distended sebacious glands which appear as tiny white spots on babys face.

Activity requirements/restrictions Q8H after laminectomy surgery?

Do not dangle (AKA sit on the side of bed) May stand, walk and ly without restrictions Don't sit longer than 30 minutes

Permanent restrictions for a laminectomy?

Do not lift by bending at the waist No crazy activities... jerky rides, horseback riding

If my pH is up my potassium (K+) is ?

Down

If a patient is on drug A for 10 yrs. and the doctor adds drug B which drug dose needs to be lowered?

Drug A

How do you treat morning sickness? (1st trimester)

Dry carb

Neuroleptic Malignant Syndomre: FEVER

F: fever E: encephalopathy V: vitals unstable E: elevated enzymes (CK) R: rigidity of muscles

s/s hypernatremia: FRIED

F: fever R: restless I: increased BP E: edema D: decreased urinary output

Endocarditis: FROM JANE

F: fever R: roth's spots O: osler's nodes M: murmur J: Janeway lesions A: anemia N: nail hemorrhage E: emboli

"statin" - atorvastatin, lovastatin, pitavastatin, pravastatin, rosuvastatin, simvastatin

HMG-CoA reductase inhibitor -lowers cholesterol

The community health nurse is conducting a research study and is identifying clients in the community who are at risk for latex allergy. Which client population is at most risk for developing this type of allergy? Hairdressers The homeless Children in day care centers Individuals living in a group home

Hairdressers

Elderly patients need to take what amount of Haldol?

Half the regular amount.

ARB side effects: Halt Dangerous Hypertension

Halt: headache Dangerous: dizziness Hypertension: hyperkalemia

What position are your hands for handwashing?

Hands below elbow

23. A staff member experienced a shoulder injury while assisting with a client transfer. The nurse manager's most therapeutic response to this situation is to: 1. Thoroughly review the accident report filed by the injured personnel to determine the factors that contributed to the injury 2. Have a nonpunitive meeting with all the involved staff to discuss correcting the factors that resulted in the injury 3. Require that mechanical lifts be used in the transfer of all clients incapable of assisting with the transfer 4. Implement new policies and procedures to correct the factors that resulted in the injury

Have a nonpunitive meeting with all the involved staff to discuss correcting the factors that resulted in the injury

3 treatments for hiatal hernia is?

High fowlers during and 1hr after meals Increase fluids with meals Increased carbs

NMS has what associated with it?

Hyperprexia

No PEE, No ??

K+; DO NOT give Potassium without adequate urine output.

Potassium increasing agents: K-Bank

K: k-sparing diuretic B: beta blocker A: ACE inhibitor N: NSAID K: K supplement

What is the only insulin safe to give at bedtime?

Lantus/Glargine

What are the three phases of labor?

Latent, Active and Transition

3 treatments for dumping syndrome?

Lay flat on side during and 1hr after meals Decrease fluids during meals(drink between meals) Decrease carbs (aka ^ protein diet)

If client exercises more they need?

Less insulin

s/s hypokalemia: 6Ls

Lethargy Lethal cardiac arrhythmia Leg cramps Limp muscles Low shallow resps Less stool (constipation)

If the WBC count is high this is called?

Leukocytosis

If the WBC is low it is called?

Leukopenia Neutropenia Agranulocytosis Immunosupression Bone marrow supression

To treat PVC's you give?

Lidocaine/Amnioderone

To treat V-tach you give? (If it start with a V you use..)

Lidocaine/Amnioderone

For a cervical laminectomy they can never?

Lift objects above the head.

21. An infant born via cesarean section because of a breech presentation is diagnosed with bilateral congenital hip dysplasia. The primary nursing intervention directed toward this diagnosis is: 1. Assessing the infant frequently to determine abduction of the thighs 2. Maintaining the infant in the position of continuous abduction of both hips 3. Educating the parents about the importance of positioning the infant so that the head of the femurs are in alignment with the hip sockets 4. Providing pain management so that the infant is comfortable in the therapeutic position required

Maintaining the infant in the position of continuous abduction of both hips

During the third stage of labor and delivery (placental delivery) you do what two things?

Make sure it's intact( if left in hemorrhage 1st infection 2nd) Check for three vessels (2 arteries, 1 vein)AVA

What is neologisms?

Make up new words.

Lithium decreases?

Mania

MAOI's are the?

Mar-plan Nar-dil Par-nate

In airborne precautions it is ok to remove what piece of PPE outside of the room?

Mask

SE for Terbutaline?

Maternal tachycardia This drug is not good if a heart issue is already present.

8. The best approach for the nurse to use to assess the presence of thrombosis in an immobilized client is to: 1. Measure the calf and thigh circumferences 2. Attempt to elicit Homans' sign 3. Palpate the temperature of the feet 4. Observe for a loss of hair and skin turgor in the lower legs

Measure the calf and thigh circumferences

Adolescents (12-18 yr.) "play" is?

Peer associated

A two point restraint is?

One arm and the opposite leg.

If water is present in the dependant loops you?

Open system and empty water.

Dilation is?

Opening of the cervix (0-10)

How can Coumadin be given?

Oral only

Pseudomonas Aeruginosa: PSEU AERUGINOSA

P: pneumonia S: sepsis E: external otits media U: UTI A: aerobic E: exotoxin A R: rod U: UTI burns and infection G: green blue pigment I: iron containing lesions N: negative (gram) non lactose fermenting O: oxidase positive S: sepsis A: adherence pili

Liver Functions: PUSH DoG

P: protein synthesis U: ureas synthesis S: storage H: hormone synthesis D: detoxification G: glucose and fat metabolism

Nephritic Syndrome: PHARAOH

P: proteinuria & edema H: hematuria A: azotemia R: RBC casts A: anti-strep titres (if post-strep) O: oliguria H: hypertension

If you have no idea what a drug is check to see if it is ___. If it is pick a _______ SE.

PO GI

What do you monitor if on Heparin?

PTT

respiratory acidosis

PaCO2 greater than 45

respiratory alkalosis

PaCO2 less than 35

3 P's of nerve root compression (S/S) ?

Pain Paresis= muscle weakness Parasthesia

Lithium SE? (The 3 P's)

Peeing Pooping Parasthesia

What is the best toy for a toddler (1-3 yr.)?

Push/pull toy

If asked about the best thing to di if the water seal breaks and not asking the first thing to do you?

Put it in water (NS).

How often do you monitor during labor?

Q15min until the 2hr mark after placental delivery. Then Q1H.

Always teach a pregnant women to pee how often from day of pregnancy to 6 weeks post partum?

Q2H

Frequency of administration for aminoglycosides?

Q8H

Regualr insulins all have what in them?

R

A client is suspected of having systemic lupus erythematous. The nurse monitors the client, knowing that which of the following is one of the initial characteristic sign of systemic lupus erythematous? Weight gain Subnormal temperature Elevated red blood cell count Rash on the face across the bridge of the nose and on the cheeks

Rash on the face across the bridge of the nose and on the cheeks

Type V: Stimulatory Reaction

Reaction of autoantibodies with normal cell-surface receptors, which stimulates a continual overreaction of the target cell Ex. Graves' disease, B-cell gammopathies

Type IV: Delayed Hypersensitivity Reactions

Reaction of sensitized T-cells with antigen and release of lymphokines, which activates macrophages and induces inflammation Ex. Poison ivy, Graft rejection, Positive TB skin tests, Sarcoidosis

Epinepherine - NC

Stimulates alpha and beta adrenergic receptors Monitor BP Give IM not IV ensure adequate hydration

Tocolytics do what to labor?

Stop it

Hypothalamus Functions "Hypothalamus wears TAN HATS"

T: thirst and water balance A: adrenohypophysis N: neurohypophysis H: hunger and satiety A: autonomic regulation T: temp regulation S: sexual urges and emotions

Post Op thyroidectomy risks 12-48 hrs for Total Thyroidectomy?

Tetany

What is the rule for Natremias?

The one with the E id dehydration the one with the O is overload HypErnatremia has s/s of dehydration and HypOnatremia has s/s of fluid overload.

Use the odd numbered gait when?

The problem is affecting one leg (unilateral)

What is Addison's Disease?

Under secretion of the adrenal cortex.

If you are asked about chest tubes after a surgery or trauma you can assumes it's a?

Unilateral Pneumohemothorax

If kinking in the tube is present you?

Unkink

Overdose of an upper causes everything to go?

Up

What is the antidote for Coumadin?

Vitamin K

How do you treat urninary incontinence during pregnancy? (1st and 3rd trimester)

Void Q2H

20. A client recovering from hip surgery tells the nurse that she wants to get better so she can walk down the aisle to her seat at her granddaughter's wedding. Which of the following nursing interventions will have the greatest impact on achieving that goal? 1. Informing physical therapists that the client has expressed that goal 2. Reminding the ancillary staff to offer to walk with the client after her bath 3. Regularly praising the client for the efforts she is making to reach her goal 4. Walking with the client to and from the dining room where she eats her meals

Walking with the client to and from the dining room where she eats her meals

Pain medication for Pancreantitis

With this, give *DEMEROL* not Morphine Sulfate.

If 24 hours after birth assume the baby is in?

Withdrawal

Every alcoholic goes through what withing 24 hours after cessation?

Withdrawal syndrome

What pregnancy class is Coumadin?

X

All K+ wasting diuretics end in?

X... If it ends in X its X's out K+ everything else it K+ sparing diuretics.

Are patients with Delirium Tremens a danger to themselves or others?

Yes

Can Delirium Tremens kill you?

Yes

Can Zoloft be taken in the evening?

Yes

Can the faucet have handles for handwashing?

Yes

Does Zoloft cause insomnia?

Yes

Is Wernickes arrestable?

Yes

Is Wernickes preventable?

Yes

Is bubbling in the suction control chanber continuously good?

Yes

Is it normal to be slightly anemic during pregnancy?

Yes

Is bubbling in the water seal intermittently good?

Yes it should tidal on inhale.

Situation: The client with a history of asthma is prescribed a leukotriene receptor antagonist to prevent allergic rhinitis. The nurse anticipates that which drug will be prescribed? A. Cromolyn sodium (Nasalcrom) B. Desloratadine (Clarinex) C. Fexofenadine (Allegra) D. Zafirlukast (Accolate)

Zafirlukast (Accolate)

SE of Betamethasone?

^ in glucose so monitor blood sugar

The wife of a 67-year-old client who has been taking imipramine (Tofranil) for 3 days asks the nurse why her husband isn't better. The nurse should tell the wife: a) "It takes 2 to 4 weeks before the full therapeutic effects are experienced." b) "Your husband may need an increase in dosage." c) "A different antidepressant may be necessary." d) "It can take 6 weeks to see if the medication will help your husband."

a - Imipramine, a tricyclic antidepressant, typically requires 2 to 4 weeks of therapy before the full therapeutic effects are experienced. Because the client has been taking the drug for only 3 days, it is too soon to determine if the current dosage of imipramine is effective. It is also too soon to consider taking another antidepressant.

A client with a bleeding ulcer is vomiting bright red blood. The nurse should assess the client for which of the following indicators of early shock? a) Tachycardia. b) Dry, flushed skin. c) Increased urine output. d) Loss of consciousness.

a - In early shock, the body attempts to meet its perfusion needs through tachycardia, vasoconstriction, and fluid conservation. The skin becomes cool and clammy. Urine output in early shock may be normal or slightly decreased. The client may experience increased restlessness and anxiety from hypoxia, but loss of consciousness is a late sign of shock.

Total parenteral nutrition (TPN) is prescribed for a client who has recently had a significant small and large bowel resection and is currently not taking anything by mouth. The nurse should: a. administer TPN through a nasogastric or gastrostomy tube b. handle TPN using strict aseptic technique c. auscultate for bowel sounds prior to administering TPN d. designate a peripheral intravenous (IV) site for TPN administration

a - TPN is hypertonic, high-calorie, high-protein, intravenous (IV) fluid that should be provided to clients without functional gastrointestinal tract motility, to better meet their metabolic needs and to support optimal nutrition and healing. TPN is ordered once daily, based on the client's current electrolyte and fluid balance, and must be handled with strict aseptic technique (because of its high glucose content, it is a perfect medium for bacterial growth). Also, because of the high tonicity, TPN must be administered through a central venous access, not a peripheral IV line. There is no specific need to auscultate for bowel sounds to determine whether TPN can safely be administered

A nurse is caring for a client with poorly managed diabetes mellitus who has a serious foot ulcer. When she informs him that the physician has ordered a wound care nurse to examine his foot, the client asks why he should see anyone other than this nurse. He states, "It's no big deal. I'll keep it covered and put antibiotic ointment on it." What is the nurse's best response? a) "We're very concerned about your foot and we want to provide the best possible care for you." b) "This is a big deal and you need to recognize how serious it is." c) "This is the physician's recommendation. The wound care nurse will see you today." d) "You could lose your foot if you don't see the wound care nurse."

a - The client's response indicates that he's in denial and needs further insight and education about his condition. Letting the client know that the nurse has his best interests in mind helps him accept the wound-care nurse. Although telling the client that his condition is serious and that the wound care nurse will see him that day are true statements, they're much too direct and may increase client resistance. Telling the client he could lose his foot is inappropriate and isn't therapeutic communication.

A client is scheduled for surgery under general anesthesia. The night before surgery, the client tells the nurse, "I can't wait to have breakfast tomorrow." Based on this statement, which nursing diagnosis should be the nurse's priority? a) Deficient knowledge related to food restrictions associated with anesthesia b) Fear related to surgery c) Risk for impaired skin integrity related to upcoming surgery d) Ineffective coping related to the stress of surgery

a - The client's statement reveals a Deficient knowledge related to food restrictions associated with general anesthesia. Fear related to surgery, Risk for impaired skin integrity related to upcoming surgery, and Ineffective coping related to the stress of surgery may be applicable nursing diagnoses but they aren't related to the client's statement.

A child, age 3, is brought to the emergency department in respiratory distress caused by acute epiglottiditis. Which clinical manifestations should the nurse expect to assess? a. severe sore throat, drooling, and inspiratory strider b. low grade fever, stridor, and a barking cough c. pulmonary congestion, a productive cough, and a fever d. sore throat, a fever, and general malaise

a - a child with acute epiglottiditis appears acutely ill and clinical manifestations may include drooping (because of difficulty swallowing), severe sore throat, hoarseness, a high temperature, and severe inspiratory stridor. A low grade fever, stridor, and barking cough that worsens at night are suggestive of croup. Pulmonary congestion, productive cough, and fever along with nasal flaring, retractions, chest pain, dyspnea, decreased breath sounds, and crackles indicate pneumococcal pneumonia. A sore throat, fever, and general malaise point to viral pharyngitis.

A nurse is caring for a client with lower back pain who is scheduled for myelography using metrizamide (a water-soluble contrast dye). After the test, the nurse should place the client in which position? a. head of the bed elevated 45 degrees b. prone c. supine with feet raised d. supine with the head lower than the trunk

a - after a myelogram, positioning depends on the dye injected. When a water-soluble dye such as metrizamide in injected, the head of the bed is elevated to a 45-degree angle to slow the upward dispersion of the dye. The prone and supine positions are contraindicated when a water-soluble contrast dye is used. The client should be positioned supine with the head lower than the trunk after an air-contrast study.

when caring for a client after a closed renal biopsy, the nurse should: a. maintain the client on strict bed rest in a supine position for 6 hours b. insert an indwelling catheter to monitor urine output c. apply a sandbag to the biopsy site to prevent bleeding d. administer IV opioid medications to promote comfort

a - after a renal biopsy, the client is maintained on strict bed rest in a supine position for at least 6 hours to prevent bleeding. If no bleeding occurs, the client typically resumes general activity after 24 hours. Urine output is monitored, but an indwelling catheter is not typically inserted. A pressure dressing is applied over the site, but a sandbag is not necessary. Opioids to control pain would not be anticipated; local discomfort at a biopsy site can be controlled with analgesics.

A nurse is documenting a variance that has occurred during the shift, and this report will be used for quality improvement to identify high-risk patterns and potentially initiate in-services programs. This is an example of which type of report? a. incident report b. nurse's shift report c. transfer report d. telemedicine report.

a - an incident report, also termed a variance report or occurrence report, is a tool healthcare agencies use to document anything out of the ordinary that results in or has the potential to result in harm to a client, employee, or visitor. These reports are used for quality improvement and not for disciplinary action. They are a means of identifying risks and high-risk patterns and initiating in-service programs to prevent further problems. A nurse's shift report is given by a primary nurse to the nurse replacing him or her by the charge nurse to the nurse who assumes responsibility for continuing client care. A transfer report is a summary of a client's condition and care when transferring clients from one unit or institution to another. A telemedicine report can link healthcare professional immediately and enable nurses to receive and give critical information about clients in a timely fashion.

the comatose victim of the car accident is to have a gastric lavage. Which of the following positions would be most appropriate for the client during this procedure? a. lateral b. supine c. trendelenburg's d. lithotomy

a - an unconscious client is best positioned in a lateral or semiprone position because these positions allow the jaw and tongue to fall forward, facilitate drainage of secretions, and prevent aspiration. Positioning the client supine carries a major risk of airway obstruction from the tongue, vomit, or nasopharyngeal secretions. Trendeleburg's position, with the head lower than the heart, decreases effective lung volume and increases the risk of cerebral edema. The lithotomy position has no purpose in this situation.

The nurse is assessing a client at her postpartum checkup 6 weeks after a vaginal delivery. The mother is bottle feeding her baby. Which client finding indicates a problem at this time? a. firm fundus at the symphysis b. white, thick vaginal discharge c. striae that are silver in color d. soft breasts without milk

a - by 4 to 6 weeks postpartum, the fundus should be deep in the pelvis and the size of a non-pregnant uterus. Subinvolution, caused by infection or retained placental fragments, is a problem associated with a uterus that is larger than expected at this time. Normal expectations include a white, thick vaginal discharge, striae that are beginning to fade to silver, and breasts that are soft without evidence of milk production (in a bottle feeding mother).

A client with type 1 diabetes must undergo bowel resection in the morning. How should the nurse proceed while caring for him on the morning of surgery? a. administer half of the client's typical morning insulin dose as ordered b. administer an oral antidiabeteic agent as ordered c. administer an I.V. insulin infusion as ordered d. administer the client's normal daily dose of insulin as ordered

a - if the nurse administers the client's normal daily dose of insulin while he's on nothing-by-mouth status before surgery, he'll experience hypoglycemia. Therefore, the nurse should administer half the daily insulin dose as ordered. Oral antidiabetic agents aren't effective for type 1 diabetes I.V. insulin infusions aren't necessary to manage blood glucose levels in clients undergoing routine surgery.

The best indicator that the client has learned how to give an insulin self-injection correctly is when the client can: a. perform the procedure safely and correctly b. critique the nurse's performance of the procedure c. explain all of the steps of the procedure correctly d. correctly answer a post-test about the procedure

a - the nurse should judge that learning has occurred from evidence of a change I the client's behavior. A client who performs a procedure safely and correctly demonstrates that he has acquired a skill. Evaluation of this skill acquisition requires performance of that skill by the client with observation by the nurse. The client must also demonstrate cognitive understanding, as shown by the ability to critique the nurse's performance. Explaining the steps demonstrates of knowledge at the cognitive level only. A post-test does not indicate the degree to which the client has learned a psychomotor skill.

Which of the following client statements indicates that the client with hepatitis B understands discharge teaching? a) "I will not drink alcohol for at least 1 year." b) "I must avoid sexual intercourse." c) "I should be able to resume normal activity in a week or two. d) "Because hepatitis B is a chronic disease, I know I will always be jaundiced."

a) CORRECT ANSWER "I will not drink alcohol for at least 1 year." Reason: It is important that the client understand that alcohol should be avoided for at least 1 year after an episode of hepatitis. Sexual intercourse does not need to be avoided, but the client should be instructed to use condoms until the hepatitis B surface antigen measurement is negative. The client will need to restrict activity until liver function test results are normal; this will not occur within 1 to 2 weeks. Jaundice will subside as the client recovers; it is not a permanent condition.

A nurse preceptor is working with a student nurse who is administering medications. Which statement by the student indicates an understanding of the action of an antacid? a) "The action occurs in the stomach by increasing the pH of the stomach contents and decreasing pepsin activity." b) "The action occurs in the small intestine, where the drug coats the lining and prevents further ulceration." c) "The action occurs in the esophagus by increasing peristalsis and improving movement of food into the stomach." d) "The action occurs in the large intestine by increasing electrolyte absorption into the system that decreases pepsin absorption."

a) CORRECT ANSWER "The action occurs in the stomach by increasing the pH of the stomach contents and decreasing pepsin activity." Reason: The action of an antacid occurs in the stomach. The anions of an antacid combine with the acidic hydrogen cations secreted by the stomach to form water, thereby increasing the pH of the stomach contents. Increasing the pH and decreasing the pepsin activity provide symptomatic relief from peptic ulcer disease. Antacids don't work in the large or small intestine or in the esophagus.

A client is scheduled for an excretory urography at 10 a.m. An order directs the nurse to insert a saline lock I.V. device at 9:30 a.m.. The client requests a local anesthetic for the I.V. procedure and the physician orders lidocaine-prilocaine cream (EMLA cream). The nurse should apply the cream at: a) 7:30 a.m. b) 8:30 a.m. c) 9 a.m. d) 9:30 a.m.

a) CORRECT ANSWER 7:30 a.m. Reason: It takes up to 2 hours for lidocaine-prilocaine cream (EMLA cream) to anesthetize an insertion site. Therefore, if the insertion is scheduled for 9:30 a.m., EMLA cream should be applied at 7:30 a.m. The local anesthetic wouldn't be effective if the nurse administered it at the later times.

Which of the following statements would provide the best guide for activity during the rehabilitation period for a client who has been treated for retinal detachment? a) Activity is resumed gradually, and the client can resume her usual activities in 5 to 6 weeks. b) Activity level is determined by the client's tolerance; she can be as active as she wishes. c) Activity level will be restricted for several months, so she should plan on being sedentary. d) Activity level can return to normal and may include regular aerobic exercises.

a) CORRECT ANSWER Activity is resumed gradually, and the client can resume her usual activities in 5 to 6 weeks. Reason: The scarring of the retinal tear needs time to heal completely. Therefore, resumption of activity should be gradual; the client may resume her usual activities in 5 to 6 weeks. Successful healing should allow the client to return to her previous level of functioning.

A nurse is caring for a client who has a tracheostomy and temperature of 103° F (39.4° C). Which intervention will most likely lower the client's arterial blood oxygen saturation? a) Endotracheal suctioning b) Encouragement of coughing c) Use of a cooling blanket d) Incentive spirometry

a) CORRECT ANSWER Endotracheal suctioning Reason: Endotracheal suctioning removes secretions as well as gases from the airway and lowers the arterial oxygen saturation (SaO2) level. Coughing and using an incentive spirometer improve oxygenation and should raise or maintain oxygen saturation. Because of superficial vasoconstriction, using a cooling blanket can lower peripheral oxygen saturation readings, but SaO2 levels wouldn't be affected.

The nurse is serving on the hospital ethics committee which is considering the ethics of a proposal for the nursing staff to search the room of a client diagnosed with substance abuse while he is off the unit and without his knowledge. Which of the following should be considered concerning the relationship of ethical and legal standards of behavior? a) Ethical standards are generally higher than those required by law. b) Ethical standards are equal to those required by law. c) Ethical standards bear no relationship to legal standards for behavior. d) Ethical standards are irrelevant when the health of a client is at risk.

a) CORRECT ANSWER Ethical standards are generally higher than those required by law. Reason: Some behavior that is legally allowed might not be considered ethically appropriate. Legal and ethical standards are often linked, such as in the commandment "Thou shalt not kill." Ethical standards are never irrelevant, though a client's safety or the safety of others may pose an ethical dilemma for health care personnel. Searching a client's room when they are not there is a violation of their privacy. Room searches can be done with a primary health care provider's order and generally are done with the client present.

Which nursing action is required before a client in labor receives epidural anesthesia? a) Give a fluid bolus of 500 ml. b) Check for maternal pupil dilation. c) Assess maternal reflexes. d) Assess maternal gait.

a) CORRECT ANSWER Give a fluid bolus of 500 ml. Reason: One of the major adverse effects of epidural administration is hypotension. Therefore, a 500-ml fluid bolus is usually administered to prevent hypotension in the client who wishes to receive an epidural for pain relief. Assessing maternal reflexes, pupil response, and gait isn't necessary.

A client is prescribed metaproterenol (Alupent) via a metered-dose inhaler, two puffs every 4 hours. The nurse instructs the client to report adverse effects. Which of the following are potential adverse effects of metaproterenol? a) Irregular heartbeat. b) Constipation. c) Pedal edema. d) Decreased pulse rate.

a) CORRECT ANSWER Irregular heartbeat. Reason: Irregular heartbeats should be reported promptly to the care provider. Metaproterenol (Alupent) may cause irregular heartbeat, tachycardia, or anginal pain because of its adrenergic effect on beta-adrenergic receptors in the heart. It is not recommended for use in clients with known cardiac disorders. Metaproterenol does not cause constipation, pedal edema, or bradycardia.

Which of the following should the nurse use to determine achievement of the expected outcome for an infant with severe diarrhea and a nursing diagnosis of Deficient fluid volume related to passage of profuse amounts of watery diarrhea? a) Moist mucous membranes. b) Passage of a soft, formed stool. c) Absence of diarrhea for a 4-hour period. d) Ability to tolerate intravenous fluids well.

a) CORRECT ANSWER Moist mucous membranes. Reason: The outcome of moist mucous membranes indicates adequate hydration and fluid balance, showing that the problem of fluid volume deficit has been corrected. Although a normal bowel movement, ability to tolerate intravenous fluids, and an increasing time interval between bowel movements are all positive signs, they do not specifically address the problem of deficient fluid volume.

Prochlorperazine (Compazine) is prescribed postoperatively. The nurse should evaluate the drug's therapeutic effect when the client expresses relief from which of the following? a) Nausea. b) Dizziness. c) Abdominal spasms. d) Abdominal distention.

a) CORRECT ANSWER Nausea. Reason: Prochlorperazine is administered postoperatively to control nausea and vomiting. Prochlorperazine is also used in psychotherapy because of its effects on mood and behavior. It is not used to treat dizziness, abdominal spasms, or abdominal distention.

A nurse takes informed consent from a client scheduled for abdominal surgery. Which of the following is the most appropriate principle behind informed consent? a) Protects the client's right to self-determination in health care decision making. b) Helps the client refuse treatment that he or she does not wish to undergo. c) Helps the client to make a living will regarding future health care required. d) Provides the client with in-depth knowledge about the treatment options available.

a) CORRECT ANSWER Protects the client's right to self-determination in health care decision making. Reason: Informed consent protects the client's right to self-determination in health care decision making. Informed consent helps the client to refuse a treatment that the client does not wish to undergo and helps the client to gain in-depth knowledge about the treatment options available, but the most important function is to encourage shared decision making. Informed consent does not help the client to make a living will.

A client has the following arterial blood gas values: pH, 7.30; PaO2, 89 mm Hg; PaCO2, 50 mm Hg; and HCO3-, 26 mEq/L. Based on these values, the nurse should suspect which condition? a) Respiratory acidosis b) Respiratory alkalosis c) Metabolic acidosis d) Metabolic alkalosis

a) CORRECT ANSWER Respiratory acidosis Reason: This client has a below-normal (acidic) blood pH value and an above-normal partial pressure of arterial carbon dioxide (PaCO2) value, indicating respiratory acidosis. In respiratory alkalosis, the pH value is above normal and the PaCO2 value is below normal. In metabolic acidosis, the pH and bicarbonate (HCO3-) values are below normal. In metabolic alkalosis, the pH and HCO3- values are above normal.

A 10-year-old child diagnosed with acute glomerulonephritis is admitted to the pediatric unit. The nurse should ensure that which action is a part of the child's care? a) Taking vital signs every 4 hours and obtaining daily weight b) Obtaining a blood sample for electrolyte analysis every morning c) Checking every urine specimen for protein and specific gravity d) Ensuring that the child has accurate intake and output and eats a high-protein diet

a) CORRECT ANSWER Taking vital signs every 4 hours and obtaining daily weight Reason: Because major complications — such as hypertensive encephalopathy, acute renal failure, and cardiac decompensation — can occur, monitoring vital signs (including blood pressure) is an important measure for a child with acute glomerulonephritis. Obtaining daily weight and monitoring intake and output also provide evidence of the child's fluid balance status. Sodium and water restrictions may be ordered depending on the severity of the edema and the extent of impaired renal function. Typically, protein intake remains normal for the child's age and is only increased if the child is losing large amounts of protein in the urine. Checking urine specimens for protein and specific gravity and daily monitoring of serum electrolyte levels may be done, but their frequency is determined by the child's status. These actions are less important nursing measures in this situation.

A nurse has received change-of-shift-report and is briefly reviewing the documentation about a client in the client's medical record. A recent entry reads, "Client was upset throughout the morning." How could the charting entry be best improved? a) The entry should include clearer descriptions of the client's mood and behavior. b) The entry should avoid mentioning cognitive or psychosocial issues. c) The entry should list the specific reasons that the client was upset. d) The entry should specify the subsequent interventions that were performed.

a) CORRECT ANSWER The entry should include clearer descriptions of the client's mood and behavior. Reason: Entries in the medical record should be precise, descriptive, and objective. An adjective such as "upset" is unclear and open to many interpretations. As such, the nurse should elaborate on this description so a reader has a clearer understanding of the client's state of mind. Stating the apparent reasons that the client was "upset" does not resolve the ambiguity of this descriptor. Cognitive and psychosocial issues are valid components of the medical record. Responses and interventions should normally follow assessment data but the data themselves must first be recorded accurately.

A nurse is facilitating mandated group therapy for clients who have sexually abused children. Children who are victims of sexual abuse are typically: a) from any segment of the population. b) of low socioeconomic background. c) strangers to the abuser. d) willing to engage in sexual acts with adults.

a) CORRECT ANSWER from any segment of the population. Reason: Victims of childhood sexual abuse come from all segments of the population and from all socioeconomic backgrounds. Most victims know their abuser. Children rarely willingly engage in sexual acts with adults because they don't have full decision-making capacities.

After staying several hours with her 9-year-old daughter who is admitted to the hospital with an asthma attack, the mother leaves to attend to her other children. The child exhibits continued signs and symptoms of respiratory distress. Which of the following findings should lead the nurse to believe the child is experiencing anxiety? a) Not able to get comfortable. b) Frequent requests for someone to stay in the room. c) Inability to remember her exact address. d) Verbalization of a feeling of tightness in her chest.

b - A 9-year-old child should be able to tolerate being alone. Frequently asking for someone to be in the room indicates a degree of psychological distress that, at this age, suggests anxiety. The inability to get comfortable is more characteristic of a child in pain. Inability to answer questions correctly may reflect a state of anoxia or a lack of knowledge. Tightness in the chest occurs as a result of bronchial spasms.

A client with chronic heart failure is receiving digoxin (Lanoxin), 0.25 mg by mouth daily, and furosemide (Lasix), 20 mg by mouth twice daily. The nurse instructs the client to notify the physician if nausea, vomiting, diarrhea, or abdominal cramps occur because these signs and symptoms may signal digoxin toxicity. Digoxin toxicity may also cause: a) visual disturbances. b) taste and smell alterations. c) dry mouth and urine retention. d) nocturia and sleep disturbances.

a) CORRECT ANSWER visual disturbances. Reason: Digoxin toxicity may cause visual disturbances (such as, flickering flashes of light, colored or halo vision, photophobia, blurring, diplopia, and scotomata), central nervous system abnormalities (such as headache, fatigue, lethargy, depression, irritability and, if profound, seizures, delusions, hallucinations, and memory loss), and cardiovascular abnormalities (abnormal heart rate and arrhythmias). Digoxin toxicity doesn't cause taste and smell alterations. Dry mouth and urine retention typically occur with anticholinergic agents, not inotropic agents such as digoxin. Nocturia and sleep disturbances are adverse effects of furosemide — especially if the client takes the second daily dose in the evening, which may cause diuresis at night.

The nurse assisting in the admission of a client with diabetic ketoacidosis will anticipate the physician ordering which of the following types of intravenous solution if the client cannot take any fluids orally? " a. 0.45% normal saline solution b. Lactated Ringer's solution c. 0.9 normal saline solution d. 5% dextrose in water (D5W)"

a. 0.45% normal saline solution Helps to hydrate patient and keep electrolyte levels balanced

Which of the following is an early symptom of glaucoma? a) Hazy vision. b) Loss of central vision. c) Blurred or "sooty" vision. d) Impaired peripheral vision.

d - In glaucoma, peripheral vision is impaired long before central vision is impaired. Hazy, blurred, or distorted vision is consistent with a diagnosis of cataracts. Loss of central vision is consistent with senile macular degeneration but it occurs late in glaucoma. Blurred or "sooty" vision is consistent with a diagnosis of detached retina.

After surgery for an ileal conduit, the nurse should closely assess the client for the occurrence of which of the following complications related to pelvic surgery? a) Peritonitis. b) Thrombophlebitis. c) Ascites. d) Inguinal hernia.

b - After pelvic surgery, there is an increased chance of thrombophlebitis owing to the pelvic manipulation that can interfere with circulation and promote venous stasis. Peritonitis is a potential complication of any abdominal surgery, not just pelvic surgery. Ascites is most frequently an indication of liver disease. Inguinal hernia may be caused by an increase in intra-abdominal pressure or a congenital weakness of the abdominal wall; ventral hernia occurs at the site of a previous abdominal incision.

The nurse has administered aminophylline to a client with emphysema. The medication is effective when there is: a) Relief from spasms of the diaphragm. b) Relaxation of smooth muscles in the bronchioles. c) Efficient pulmonary circulation. d) Stimulation of the medullary respiratory center.

b - Aminophylline, a bronchodilator that relaxes smooth muscles in the bronchioles, is used in the treatment of emphysema to improve ventilation by dilating the bronchioles. Aminophylline does not have an effect on the diaphragm or the medullary respiratory center and does not promote pulmonary circulation.

A client complains of severe abdominal pain. To elicit as much information as possible about the pain, the nurse should ask: a) "Do you have the pain all the time?" b) "Can you describe the pain?" c) "Where does it hurt the most?" d) "Is the pain stabbing like a knife?"

b - Asking an open-ended question such as "Can you describe the pain?" encourages the client to describe any and all aspects of the pain in his own words. The other options are likely to elicit less information because they're more specific and would limit the client's response.

A pregnant client in her third trimester is started on chlorpromazine (Thorazine) 25 mg four times daily. Which of the following instructions is most important for the nurse to include in the client's teaching plan? a) "Don't drive because there's a possibility of seizures occurring." b) "Avoid going out in the sun without a sunscreen with a sun protection factor of 25." c) "Stop the medication immediately if constipation occurs." d) "Tell your doctor if you experience an increase in blood pressure."

b - Chlorpromazine is a low-potency antipsychotic that is likely to cause sun-sensitive skin. Therefore the client needs instructions about using sunscreen with a sun protection factor of 25 or higher. Typically, chlorpromazine is not associated with an increased risk of seizures. Although constipation is a common adverse effect of this drug, it can be managed with diet, fluids, and exercise. The drug does not need to be discontinued. Chlorpromazine is associated with postural hypotension, not hypertension. Additionally, if postural hypotension occurs, safety measures, such as changing positions slowly and dangling the feet before arising, not stopping the drug, are instituted.

A 7 year old with a history of tonic-clonic seizures has been actively seizing for 10 minutes. The child weighs 22 kg and currently has an intravenous (IV) line of D5 1/2 NS + 20 meq KCL/L running at 60 ml/hr. Vital signs are a temperature of 38 degrees C, heart rate of 120, respiratory rate of 28, and oxygen saturation of 92%. Using the SBAR (Situation-Background-Assessment-Recommendation) technique for communication, the nurse calls the primary healthcare provider with a recommendation for: a) Rectal diazepam (Diastat). b) IV lorazepam (Ativan). c) Rectal acetaminophen (Tylenol). d) IV fosphenytoin.

b - IV ativan is the benzodiazepine of choice for treating prolonged seizure activity. IV benzodiazepines potentiate the action of the gamma-aminobutyric acid (GABA) neurotransmitter, stopping seizure activity. If an IV line is not available, rectal Diastat is the benzodiazepine of choice. The child does have a low-grade fever; however, this is likely caused by the excessive motor activity. The primary goal for the child is to stop the seizure in order to reduce neurologic damage. Benzodiazepines are used for the initial treatment of prolonged seizures. Once the seizure has ended, a loading dose of fosphenytoin or phenobarbital is given.

A 10-month-old child with recurrent otitis media is brought to the clinic for evaluation. To help determine the cause of the child's condition, the nurse should ask the parents: a) "Does water ever get into the baby's ears during shampooing?" b) "Do you give the baby a bottle to take to bed?" c) "Have you noticed a lot of wax in the baby's ears?" d) "Can the baby combine two words when speaking?"

b - In a young child, the eustachian tube is relatively short, wide, and horizontal, promoting drainage of secretions from the nasopharynx into the middle ear. Therefore, asking if the child takes a bottle to bed is appropriate because drinking while lying down may cause fluids to pool in the pharyngeal cavity, increasing the risk of otitis media. Asking if the parent noticed earwax, or cerumen, in the external ear canal is incorrect because wax doesn't promote the development of otitis media. During shampooing, water may become trapped in the external ear canal by large amounts of cerumen, possibly causing otitis external (external ear inflammation) as opposed to internal ear inflammation. Asking if the infant can combine two words is incorrect because a 10-month-old child isn't expected to do so.

The mother of a client with chronic undifferentiated schizophrenia calls the visiting nurse in the outpatient clinic to report that her daughter has not answered the phone in 10 days. "She was doing so well for months. I don't know what's wrong. I'm worried." Which of the following responses by the nurse is most appropriate? a) "Maybe she's just mad at you. Did you have an argument?" b) "She may have stopped taking her medications. I'll check on her." c) "Don't worry about this. It happens sometimes." d) "Go over to her apartment and see what's going on."

b - Noncompliance with medications is common in the client with chronic undifferentiated schizophrenia. The nurse has the responsibility to assess this situation. Asking the mother if they've argued or if the client is mad at the mother or telling the mother to go over to the apartment and see what's going on places the blame and responsibility on the mother and therefore is inappropriate. Telling the mother not to worry ignores the seriousness of the client's symptoms.

A client with inflammatory bowel disease is receiving total parenteral nutrition (TPN). The basic component of the client's TPN solution is most likely to be: a) An isotonic dextrose solution. b) A hypertonic dextrose solution. c) A hypotonic dextrose solution. d) A colloidal dextrose solution.

b - The TPN solution is usually a hypertonic dextrose solution. The greater the concentration of dextrose in solution, the greater the tonicity. Hypertonic dextrose solutions are used to meet the body's calorie demands in a volume of fluid that will not overload the cardiovascular system. An isotonic dextrose solution (e.g., 5% dextrose in water) or a hypotonic dextrose solution will not provide enough calories to meet metabolic needs. Colloids are plasma expanders and blood products and are not used in TPN.

A client with bipolar disorder, manic phase, just sat down to watch television in the lounge. As the nurse approaches the lounge area, the client states, "The sun is shining. Where is my son? I love Lucy. Let's play ball." The client is displaying: a) Concreteness. b) Flight of ideas. c) Depersonalization. d) Use of neologisms.

b - The client is demonstrating flight of ideas, or the rapid, unconnected, and often illogical progression from one topic to another. Concreteness involves interpreting another person's words literally. Depersonalization refers to feelings of strangeness concerning the environment or the self. A neologism is a word made up by a client.

The nurse is caring for several mother-baby couplets. In planning the care for each of the couplets, which mother would the nurse expect to have the most severe afterbirth pains? a) G 4, P 1 client who is breastfeeding her infant. b) G 3, P 3 client who is breastfeeding her infant. c) G 2, P 2 cesarean client who is bottle-feeding her infant. d) G 3, P 3 client who is bottle-feeding her infant.

b - The major reasons for afterbirth pains are breast-feeding, high parity, overdistended uterus during pregnancy, and a uterus filled with blood clots. Physiologically, afterbirth pains are caused by intermittent contraction and relaxation of the uterus. These contractions are stronger in multigravidas in order to maintain a contracted uterus. The release of oxytocin when breast-feeding also stimulates uterine contractions. There is no data to suggest any of these clients has had an overdistended uterus or currently has clots within the uterus. The G 3, P 3 client who is breast-feeding has the highest parity of the clients listed, which—in addition to breast-feeding—places her most at risk for afterbirth pains. The G 2, P 2 postcesarean client may have cramping but it should be less than the G 3, P 3 client. The G 3, P 3 client who is bottle-feeding would be at risk for afterbirth pains because she has delivered several children, but her choice to bottle-feed reduces her risk of pain.

A nurse is monitoring a client receiving tranylcypromine sulfate (Parnate). Which serious adverse reaction can occur with high dosages of this monoamine oxidase (MAO) inhibitor? a) Hypotensive episodes b) Hypertensive crisis c) Muscle flaccidity d) Hypoglycemia

b - The most serious adverse reaction associated with high doses of MAO inhibitors is hypertensive crisis, which can lead to death. Although not a crisis, orthostatic hypotension is also common and may lead to syncope with high doses. Muscle spasticity (not flaccidity) is associated with MAO inhibitor therapy. Hypoglycemia isn't an adverse reaction of MAO inhibitors.

Which scenario complies with Health Insurance Portability and Accountability Act of 1996 (HIPAA) regulations? a) Two nurses in the cafeteria are discussing a client's condition. b) The health care team is discussing a client's care during a formal care conference. c) A nurse checks the computer for the laboratory results of a neighbor who has been admitted to another floor. d) A nurse talks with her spouse about a client's condition.

b - To provide interdisciplinary continuity of care, nurses must share relevant information during client care conferences. Nurses discussing information in the cafeteria may be overheard; this indiscretion violates HIPAA regulations. Looking up laboratory results for a neighbor is a HIPAA violation, as is discussing a client's condition with one's spouse.

A nurse is instructing a client with bipolar disorder on proper use of lithium carbonate (Eskalith), the drug's adverse effects, and symptoms of lithium toxicity. Which client statement indicates that additional teaching is required? a. "I can still eat my favorite salty foods." b. "when my moods fluctuate, I'll increase my dose of lithium." c. "a good blood level of the drug means the drug concentration has stabilized." d. "eating too much watermelon will affect my lithium level."

b - a client who states that he'll increase his dose of lithium if his mood fluctuates requires additional teaching because increasing the dose of lithium without evaluating the client's laboratory values can cause serious health problems, such as lithium toxicity, overdose, and renal failure. Clients taking lithium don't need to limit their sodium intake. A low sodium diet causes lithium retention. A therapeutic lithium blood level indicates that the drug concentration has stabilized. The client demonstrates effective teaching by stating his lithium levels will be affected by foods that have a diuretic effect, such as watermelon, cantaloupe, grapefruit juice and cranberry juice.

A 10-month-old child has cold symptoms. The mother asks how she can clear infant's nose. Which of the following would be the nurse's best recommendation? a. use a cool air vaporizer with plain water b. use saline nose drops and then a bulb syringe c. blow into the child's mouth to clear the nose d. administer a nonprescription vasoconstrictive nose spray.

b - although a cool air vaporizer may be recommended to humidify the environment, using saline nose drops and then a bulb syringe before meals and at nap and bed times will allow the child to breathe more easily. Saline helps to loosen secretions and keep the mucous membranes moist. The bulb syringe then gently aids in removing the loosened secretions. Blowing into the child's mouth to clear the nose introduces more organisms to the child. A nonprescription vasoconstrictive nasal spray is not recommended for infants because if the spray in used for longer than 3 days a rebound effect with increased inflammation occurs.

A client with Rh isoimmunization gives birth to a neonate with an enlarged heart and severe, generalized edema. The neonate is immediately transferred to the neonatal intensive care unit. Which nursing diagnosis is most appropriate for the client? a. ineffective parenting related to the neonate's transfer to the intensive care unit b. impaired parenting related to the neonate's transfer to the intensive care unit c. deficient fluid volume related to severe edema d. fear related to removal and loss of the neonate by statute

b - because the neonate is severely ill and needs to be placed in the neonatal intensive care unit, the client may have a nursing diagnosis of 'impaired parenting related to the neonate's transfer to the neonatal intensive care unit.' (another pertinent nursing diagnosis may be 'compromised family coping related to lack the opportunity for bonding.) Rh isoimmunization isn't a socially unacceptable infection. This condition causes an excess fluid volume (not deficient) related to cardiac problems. Rh isoimmunization doesn't lead to loss of the neonate by statute.

A client who has a history of Crohn's disease is admitted to the hospital with fever, diarrhea, cramping, abdominal pain, and weight loss. The nurse should monitor the client for: a) Hyperalbuminemia. b) Thrombocytopenia. c) Hypokalemia. d) Hypercalcemia.

c) CORRECT ANSWER Hypokalemia. Reason: Hypokalemia is the most expected laboratory finding owing to the diarrhea. Hypoalbuminemia can also occur in Crohn's disease; however, the client's potassium level is of greater importance at this time because a low potassium level can cause cardiac arrest. Anemia is an expected development, but thrombocytopenia is not. Calcium levels are not affected.

The major goal of therapy in crisis intervention is to: a. withdraw from the stress b. resolve the immediate problem c. decrease anxiety d. provide documentation of events

b - during a period of crisis, the major goal is to resolve the immediate problem, with hopes of getting the individual to the level of functioning that existed before the crisis or to a higher level of functioning. Withdrawing from stress doesn't address the immediate problem and isn't therapeutic. The client's anxiety will decrease after the immediate problem is resolved. Providing support and safety are necessary interventions while working toward accomplishing the goal. Documentation is necessary for maintaining accurate records of treatment, but isn't a major goal.

a client comes to the outpatient department complaining of vaginal discharge, dysuria, and genital irritation. Suspecting a sexually transmitted disease (STD), the physician orders diagnostic testing of the vaginal discharge. Which STD must be reported to the public health department. a. bacterial vaginitis b. gonorrhea c. genital herpes d. human papillomavirus (HPV)

b - gonorrhea must be reported to the public health department. Bacterial vaginitis, genital herpes, and HPV aren't reportable diseases

A client has a herniated disk in the region of the third and fourth lumbar vertebrae. Which nursing assessment finding most supports this diagnosis? a. hypoactive bowel sounds b. severe lower back pain c. sensory deficits in one arm d. weakness and atrophy of the arm muscles

b - the most common finding in a client with a herniated lumbar disk is severe lower back pain, which radiates to the buttocks, legs, and feet - usually unilaterally. A herniated disk also may cause sensory and motor loss (such as foot drop) in the area innervated by the compressed spinal nerve root. During later stages, it may cause weakness and atrophy of leg muscles. The condition doesn't affect bowel sounds or the arms.

Before discharge, which instruction should a nurse give to a client receiving digoxin (Lanoxin)? a. "take an extra dose of digoxin if you miss one dose." b. "call the physician if your heart rate is above 90 beats/minute c. "call the physician if your pulse drops below 80 beats/minute." d. "take digoxin with meals."

b - the nurse should instruct the client to notify the physician if his heart rate is greater than 90 beats/ minute because cardiac arrhythmias may occur with digoxin toxicity. To prevent toxicity, the nurse should instruct the client never to take an extra dose of digoxin if he misses a dose. The nurse should show the client how to take his pulse and tell him to call the physician if his pulse rate drops below 60 beats/minute - not 80 beats/minute, which is a normal pulse rate and doesn't warrant action. The client shouldn't take digoxin with meals; doing so slows the absorption rate.

The nurse observes that the right eye of an unconscious client does not close completely. Which nursing intervention is most appropriate a. have the client wear eyeglasses at all time b. lightly tape the eyelid shut c. instill artificial tears once every shift d. clean the eyelid with a washcloth every shift

b - when the blink reflex is absent or the eyes do not close completely, the cornea may become dry and irritated. Corneal abrasion can occur. Taping the eye closed will prevent injury. Having the client wears eyeglasses or cleaning the eyelid will not protect the cornea from dryness or irritation. Artificial tears instilled once per shift are not frequent enough for preventing dryness.

A home health nurse who sees a client with diverticulitis is evaluating teaching about dietary modifications necessary to prevent future episodes. Which statement by the client indicates effective teaching? a) "I'll increase my intake of protein during exacerbations." b) "I should increase my intake of fresh fruits and vegetables during remissions." c) "I'll snack on nuts, olives, and popcorn during flare-ups." d) "I'll incorporate foods rich in omega-3 fatty acids into my diet."

b) CORRECT ANSWER "I should increase my intake of fresh fruits and vegetables during remissions." Reason: A client with diverticulitis needs to modify fiber intake to effectively manage the disease. During episodes of diverticulitis, he should follow a low-fiber diet to help minimize bulk in the stools. A client with diverticulosis should follow a high-fiber diet. Clients with diverticular disease don't need to modify their intake of protein and omega-3 fatty acids.

The nurse has discussed sexuality issues during the prenatal period with a primigravida who is at 32 weeks' gestation. She has had one episode of preterm labor. The nurse determines that the client understands the instructions when she says: a) "I can resume sexual intercourse when the bleeding stops." b) "I should not get sexually aroused or have any nipple stimulation." c) "I can resume sexual intercourse in 1 to 2 weeks." d) "I should not have sexual intercourse until my next prenatal visit."

b) CORRECT ANSWER "I should not get sexually aroused or have any nipple stimulation." Reason: This client has already had one episode of preterm labor at 32 weeks' gestation. Sexual intercourse, arousal, and nipple stimulation may result in the release of oxytocin which can contribute to continued preterm labor and early delivery. The client should be advised to refrain from these activities until closer to term, which is 6 to 8 weeks later. Telling the client that intercourse is acceptable after the bleeding stops is incorrect and may lead to early delivery of a preterm neonate. The client should not have intercourse for at least 6 weeks because of the danger of inducing labor. There is no indication when the client's next prenatal visit is scheduled.

When planning care for a client with a head injury, which position should the nurse include in the care plan to enhance client outcomes? a) Trendelenburg's b) 30-degree head elevation c) Flat d) Side-lying

b) CORRECT ANSWER 30-degree head elevation Reason: For clients with increased intracranial pressure (ICP), the head of the bed should be elevated to 30 degrees to promote venous outflow. Trendelenburg's position is contraindicated because it can raise ICP. Flat or neutral positioning is indicated when elevating the head of the bed would increase the risk of neck injury or airway obstruction. A side-lying position isn't specifically a therapeutic treatment for increased ICP.

A child with a poor nutritional status and weight loss is at risk for a negative nitrogen balance. To help diagnose this problem, the nurse anticipates that the physician will order which laboratory test? a) Total iron-binding capacity b) Hemoglobin (Hb) c) Total protein d) Sweat test

c) CORRECT ANSWER Total protein Reason: The nurse anticipates the physician will order a total protein test because negative nitrogen balance may result from inadequate protein intake. Measuring total iron-binding capacity and Hb levels would help detect iron deficiency anemia, not a negative nitrogen balance. The sweat test helps diagnose cystic fibrosis, not a negative nitrogen balance.

A client with an incomplete small-bowel obstruction is to be treated with a Cantor tube. Which of the following measures would most likely be included in the client's care once the Cantor tube has passed into the duodenum? a) Maintain bed rest with bathroom privileges. b) Advance the tube 2 to 4 inches at specified times. c) Avoid frequent mouth care. d) Provide ice chips for the client to suck.

b) CORRECT ANSWER Advance the tube 2 to 4 inches at specified times. Reason: Once the intestinal tube has passed into the duodenum, it is usually advanced as ordered 2 to 4 inches every 30 to 60 minutes. This, along with gravity and peristalsis, enables passage of the tube forward. The client is encouraged to walk, which also facilitates tube progression. A client with an intestinal tube needs frequent mouth care to stimulate saliva secretion, to maintain a healthy oral cavity, and to promote comfort regardless of where the tube is placed in the intestine. Ice chips are contraindicated because hypotonic fluid will draw extra fluid into an already distended bowel.

A woman who has recently immigrated from Africa who delivered a term neonate a short time ago requests that a "special bracelet" be placed on the baby's wrist. The nurse should: a) Tell the mother that the bracelet is not recommended for cleanliness reasons. b) Apply the bracelet on the neonate's wrist as the mother requests. c) Place the bracelet on the neonate, limiting its use to when the neonate is with the mother. d) Recommend that the mother wait until she is discharged to apply the bracelet.

b) CORRECT ANSWER Apply the bracelet on the neonate's wrist as the mother requests. Reason: The nurse should abide by the mother's request and place the bracelet on the neonate. In some cultures, amulets and other special objects are viewed as good luck symbols. By allowing the bracelet, the nurse demonstrates culturally sensitive care, promoting trust. The neonate can wear the bracelet while with the mother or in the nursery. The bracelet can be used while the neonate is being bathed, or if necessary and acceptable to the client removed and replaced afterward.

When measuring the fundal height of a primigravid client at 20 weeks' gestation, the nurse will locate the fundal height at which of the following points? a) Halfway between the client's symphysis pubis and umbilicus. b) At about the level of the client's umbilicus. c) Between the client's umbilicus and xiphoid process. d) Near the client's xiphoid process and compressing the diaphragm.

b) CORRECT ANSWER At about the level of the client's umbilicus. Reason: Measurement of the client's fundal height is a gross estimate of fetal gestational age. At 20 weeks' gestation, the fundal height should be at about the level of the client's umbilicus. The fundus typically is over the symphysis pubis at 12 weeks. A fundal height measurement between these two areas would suggest a fetus with a gestational age between 12 and 20 weeks. The fundal height increases approximately 1 cm/week after 20 weeks' gestation. The fundus typically reaches the xiphoid process at approximately 36 weeks' gestation. A fundal height between the umbilicus and the xiphoid process would suggest a fetus with a gestational age between 20 and 36 weeks. The fundus then commonly returns to about 4 cm below the xiphoid owing to lightening at 40 weeks. Additionally, pressure on the diaphragm occurs late in pregnancy. Therefore, a fundal height measurement near the xiphoid process with diaphragmatic compression suggests a fetus near the gestational age of 36 weeks or older.

A nurse is assessing the legs of a client who's 36 weeks pregnant. Which finding should the nurse expect? a) Absent pedal pulses b) Bilateral dependent edema c) Sluggish capillary refill d) Unilateral calf enlargement

b) CORRECT ANSWER Bilateral dependent edema Reason: As the uterus grows heavier during pregnancy, femoral venous pressure rises, leading to bilateral dependent edema. Factors interfering with venous return, such as sitting or standing for long periods, contribute to edema. Absence of pedal pulses and sluggish capillary refill signal inadequate circulation to the legs — an unexpected finding during pregnancy. Unilateral calf enlargement, also an abnormal finding, may indicate thrombosis.

A parent brings a 5-year-old child to a vaccination clinic to prepare for school entry. The nurse notes that the child has not had any vaccinations since 4 months of age. To determine the current evidence for best practices for scheduling missed vaccinations the nurse should: a) Ask the primary care provider. b) Check the website at the Center for Disease Control and Prevention (CDC). c) Read the vaccine manufacturer's insert. d) Contact the pharmacist.

b) CORRECT ANSWER Check the website at the Center for Disease Control and Prevention (CDC). Reason: The CDC is the federal body that is ultimately responsible for vaccination recommendations for adults and children. A division of the CDC, the Advisory Committee on Immunization Practices, reviews vaccination evidence and updates recommendation on a yearly basis. The CDC publishes current vaccination catch-up schedules that are readily available on their website. The lack of vaccinations is a strong indicator that the child probably does not have a primary care provider. If consulted, the pharmacist would most likely have to review the CDC guidelines that are equally available to the nurse. Reading the manufacturer's inserts for multiple vaccines would be time consuming and synthesis of the information could possibly lead to errors.

A 10-year-old with glomerulonephritis reports a headache and blurred vision. The nurse should immediately: a) Put the client to bed. b) Obtain the child's blood pressure. c) Notify the physician. d) Administer acetaminophen (Tylenol).

b) CORRECT ANSWER Obtain the child's blood pressure. Reason: Hypertension occurs with acute glomerulonephritis. The symptoms of headache and blurred vision may indicate an elevated blood pressure. Hypertension in acute glomerulonephritis occurs due to the inability of the kidneys to remove fluid and sodium; the fluid is reabsorbed, causing fluid volume excess. The nurse must verify that these symptoms are due to hypertension. Calling the physician before confirming the cause of the symptoms would not assist the physician in his treatment. Putting the client to bed may help treat an elevated blood pressure, but first the nurse must establish that high blood pressure is the cause of the symptoms. Administering Tylenol for high blood pressure is not recommended.

Which of the following interventions would be most appropriate for the nurse to recommend to a client to decrease discomfort from hemorrhoids? a) Decrease fiber in the diet. b) Take laxatives to promote bowel movements. c) Use warm sitz baths. d) Decrease physical activity.

c) CORRECT ANSWER Use warm sitz baths. Reason: Use of warm sitz baths can help relieve the rectal discomfort of hemorrhoids. Fiber in the diet should be increased to promote regular bowel movements. Laxatives are irritating and should be avoided. Decreasing physical activity will not decrease discomfort.

A client who took an overdose of Tylenol in a suicide attempt is transferred overnight to the psychiatric inpatient unit from the intensive care unit. The night shift nurse called the primary health care provider on call to obtain initial prescriptions. The primary health care provider prescribes the typical routine medications for clients on this unit: Milk of Magnesia, Maalox and Tylenol as needed. Prior to implementing the prescriptions, the nurse should? a) Ask the primary health care provider about holding all the client's PM prescriptions. b) Question the primary health care provider about the Tylenol prescription. c) Request a prescription for a medication to relieve agitation. d) Suggest the primary health care provider write a prescription for intravenous fluids.

b) CORRECT ANSWER Question the primary health care provider about the Tylenol prescription. Reason: The nurse should question the Tylenol order because the client overdosed on Tylenol, and that analgesic would be contraindicated as putting further stress on the liver. There is no need to hold the PM Milk of Magnesia or Maalox. There is no indication that the client is agitated or needs medication for agitation. There is little likelihood that the client needs an IV after being transferred out of an intensive care unit, as the client will be able to take oral fluids.

A nurse assesses a client's respiratory status. Which observation indicates that the client is having difficulty breathing? a) Diaphragmatic breathing b) Use of accessory muscles c) Pursed-lip breathing d) Controlled breathing

b) CORRECT ANSWER Use of accessory muscles Reason: The use of accessory muscles for respiration indicates the client is having difficulty breathing. Diaphragmatic and pursed-lip breathing are two controlled breathing techniques that help the client conserve energy.

A primigravid client gives birth to a full-term girl. When teaching the client and her partner how to change their neonate's diaper, the nurse should instruct them to: a) fold a cloth diaper so that a double thickness covers the front. b) clean and dry the neonate's perineal area from front to back. c) place a disposable diaper over a cloth diaper to provide extra protection. d) position the neonate so that urine will fall to the back of the diaper.

b) CORRECT ANSWER clean and dry the neonate's perineal area from front to back. Reason: When changing a female neonate's diaper, the caregiver should clean the perineal area from front to back to prevent infection and then dry the area thoroughly to minimize skin breakdown. For a male, the caregiver should clean and dry under and around the scrotum. Because of anatomic factors, a female's diaper should have the double thickness toward the back. The diaper, not the neonate, should be positioned properly. Placing a disposable diaper over a cloth diaper isn't necessary. The direction of urine flow can't be ensured.

"Polydipsia and polyuria related to diabetes mellitus are primarily due to: "a. The release of ketones from cells during fat metabolism b. Fluid shifts resulting from the osmotic effect of hyperglycemia c. Damage to the kidneys from exposure to high levels of glucose d. Changes in RBCs resulting from attachment of excessive glucose to hemoglobin

b. Fluid shifts resulting from the osmotic effect of hyperglycemia Rationale: The osmotic effect of glucose produces the manifestations of polydipsia and polyuria.

"Polydipsia and polyuria related to diabetes mellitus are primarily due to: "a. The release of ketones from cells during fat metabolism b. Fluid shifts resulting from the osmotic effect of hyperglycemia c. Damage to the kidneys from exposure to high levels of glucose d. Changes in RBCs resulting from attachment of excessive glucose to hemoglobin"

b. Fluid shifts resulting from the osmotic effect of hyperglycemia Rationale: The osmotic effect of glucose produces the manifestations of polydipsia and polyuria.

"zepam" - clonazepam, diazepam, flurazepam, lorazepam, temazepam "zolam" - alprazolam, esazolam, midazolam, triazolam

benzodiazepine

"olol" - atenolol, metoprolol, nadolol, pindolol, propranolol, timolol

beta blocker - antihypertensive

A nurse is conducting an initial assessment on a client with possible tuberculosis. Which assessment finding indicates a risk factor for tuberculosis? a) The client sees his physician for a check-up yearly. b) The client has never traveled outside of the country. c) The client had a liver transplant 2 years ago. d) The client works in a health care insurance office.

c - A history of immunocompromised status, such as that which occurs with liver transplantation, places the client at a higher risk for contracting tuberculosis. Other risk factors include inadequate health care, traveling to countries with high rates of tuberculosis (such as southeastern Asia, Africa, and Latin America), being a health care worker who performs procedures in which exposure to respiratory secretions is likely, and being institutionalized.

A nurse is helping a physician insert a subclavian central line. After the physician has gained access to the subclavian vein, he connects a 10-ml syringe to the catheter and withdraws a sample of blood. He then disconnects the syringe from the port. Suddenly, the client becomes confused, disoriented, and pale. The nurse suspects an air embolus. She should: a) place the client in a supine position and prepare to perform cardiopulmonary resuscitation. b) place the client in high-Fowler's position and administer supplemental oxygen. c) turn the client on his left side and place the bed in Trendelenburg's position. d) position the client in the shock position with his legs elevated.

c - A nurse who suspects an air embolism should place the client on his left side and in Trendelenburg's position. Doing so allows the air to collect in the right atrium rather than enter the pulmonary system. The supine position, high-Fowler's position, and the shock position are therapeutic for other situations but not for air embolism.

When obtaining the vital signs of a client with multiple traumatic injuries, a nurse detects bradycardia, bradypnea, and systolic hypertension. The nurse must notify the physician immediately because these findings may reflect which complication? a) Shock b) Encephalitis c) Increased intracranial pressure (ICP) d) Status epilepticus

c - When ICP increases, Cushing's triad may develop, which involves decreased heart and respiratory rates and increased systolic blood pressure. Shock typically causes tachycardia, tachypnea, and hypotension. In encephalitis, the temperature rises and the heart and respiratory rates may increase from the effects of fever on the metabolic rate. (If the client doesn't maintain adequate hydration, hypotension may occur.) Status epilepticus causes unceasing seizures, not changes in vital signs.

A nurse, a licensed practical nurse (LPN), and a nursing assistant are caring for a group of clients. The nurse asks the nursing assistant to check the pulse oximetry level of a client who underwent laminectomy. The nursing assistant reports that the pulse oximetry reading is 89%. The client Kardex contains an order for oxygen application at 2 L/min should the pulse oximetry level fall below 92%. The nurse is currently assessing a postoperative client who just returned from the postanesthesia unit. How should the nurse proceed? a. immediately go the client's room and assess vital signs, administer oxygen at 2 L/minute, and notify the physician. b. ask the nursing assistant to notify the physician of the low pulse oximetry level c. ask the LPN to obtain vital signs and administer oxygen at 2 L/min to the client who underwent laminectomy d. complete the assessment of the new client before attending to the client who underwent laminectomy

c - because it's important to get more information about the client with a decreased pulse oximetry level, the nurse should ask the LPN to obtain vital signs and administer oxygen as ordered. The nurse must attend to the newly admitted client without delaying treatment to the client who is already in her care. The nurse can effectively do this by delegating tasks to an appropriate health team member such as an LPN. The nurse doesn't need to immediately attend to the client with a decreased pulse oximetry level; she may wait until she completes the assessment of the newly admitted client. The physician doesn't need to be notified at this time because an order for oxygen administration is already on record.

A client has an episiotomy to widen her birth canal. Birth extends the incision into the anal sphincter. This complication is called: a. a first-degree laceration b. a second-degree laceration c. a third-degree laceration d. a fourth-degree laceration

c - birth may extend an episiotomy incision to the anal sphincter (a third degree laceration) or the anal canal (a fourth degree laceration). A first degree laceration involves the fourchette, perineal skin, and vaginal mucous membranes. A second degree laceration extends to the fasciae and muscle of the perineal body.

The nurse should instruct the family of a child with newly diagnosed hyperthyroidism to: a. keep their home warmer than usual b. encourage plenty of outdoor activities c. promote interactions with one friend instead of groups d. limit bathing to prevent skin irritation

c - children with hyperthyroidism experience emotional labiality that may strain interpersonal relationships. Focusing on one friend's is easier than adapting to group dynamics until the child's condition improves. Because of their high metabolic rate, children with hyperthyroidism complain of being too warm. Bright sunshine may be irritating because of disease-related ophthalmopathy. Sweating is common and bathing should be encouraged.

A client with pneumonia has a temperature of 102.6F (39.2C), is diaphoretic, and has a productive cough. The nurse should include which of the following measures in the plan of care? a. position changes every 4 hours b. nasotracheal suctioning to clear secretions c. frequent linen changes d. frequent offering of a bedpan

c - frequent linen changes are appropriate for this client because of the diaphoresis. Diaphoresis produces general discomfort. The client should be kept dry to promote comfort. Position changes need to be done every 2 hours. Nasotracheal suctioning is not indicated with the client's productive cough. Frequent offering of a bedpan is not indicated by the data provided in this scenario.

The neonoate of a client with type 1 diabetes is at high risk for hypoglycemia. An initial sign the nurse should recognize as indicating hypoglycemia in a neonate is: a. peripheral acrocyanosis b. bradycardia c. lethargy d. jaundice

c - lethargy in the neonate may be seen with hypoglycemia because of a glucose in the nerve cells. Peripheral acrocyanosis is normal in the neonate because of immature capillary function. Tachycardia - not bradycardia - is seen with hypoglycemia. Jaundice isn't a sign of hypoglycemia.

The physician ordered IV naloxone (Narcan) to reverse the respiratory depression from morphine administration. After administration of the naloxone the nurse should: a. check respirations in 5 minutes because naxolone is immediately effective in relieving respiratory depression b. check respirations in 30 minutes because the effects of morphine will have worn off by then c monitor respirations frequently for 4 to 6 hours because the client may need repeated doses of naloxone d. monitor respirations each time the client receives morphine sulfate 10 mg IM

c - the nurse should monitor the client's respirations closely for 4 to 6 hours because naloxone has a shorter duration of action than opioids. The client may need repeated doses of naloxone to prevent or treat a recurrence of the respiratory depression. Naloxone is usually effective in a few minutes; however, its effects last only 1 to 2 hours and ongoing monitoring of the client's respiratory rate will be necessary. The client's dosage of morphine will be decreased or a new drug will be ordered to prevent another instance of respiratory depression.

After discussing asthma as a chronic condition, which of the following statements by the father of a child with asthma best reflects the family's positive adjustment to this aspect of the child's disease? a) "We try to keep him happy at all costs; otherwise, he has an asthma attack." b) "We keep our child away from other children to help cut down on infections." c) "Although our child's disease is serious, we try not to let it be the focus of our family." d) "I'm afraid that when my child gets older, he won't be able to care for himself like I do."

c) CORRECT ANSWER "Although our child's disease is serious, we try not to let it be the focus of our family." Reason: Positive adjustment to a chronic condition requires placing the child's illness in its proper perspective. Children with asthma need to be treated as normally as possible within the scope of the limitations imposed by the illness. They also need to learn how to manage exacerbations and then resume as normal a life as possible. Trying to keep the child happy at all costs is inappropriate and can lead to the child's never learning how to accept responsibility for behavior and get along with others. Although minimizing the child's risk for exposure to infections is important, the child needs to be with his or her peers to ensure appropriate growth and development. Children with a chronic illness need to be involved in their care so that they can learn to manage it. Some parents tend to overprotect their child with a chronic illness. This overprotectiveness may cause a child to have an exaggerated feeling of importance or later, as an adolescent, to rebel against the overprotectiveness and the parents.

The nurse meets with the client and his wife to discuss depression and the client's medication. Which of the following comments by the wife would indicate that the nurse's teaching about disease process and medications has been effective? a) "His depression is almost cured." b) "He's intelligent and won't need to depend on a pill much longer." c) "It's important for him to take his medication so that the depression will not return or get worse." d) "It's important to watch for physical dependency on Zoloft."

c) CORRECT ANSWER "It's important for him to take his medication so that the depression will not return or get worse." Reason: Improved balance of neurotransmitters is achieved with medication. Clients with endogenous depression must take antidepressants to prevent a return or worsening of depressive symptoms. Depression is a chronic disease characterized by periods of remission; however, it is not cured. Depression is not dependent on the client's intelligence to will the illness away. Zoloft is not physically addictive.

A worried mother confides in the nurse that she wants to change physicians because her infant is not getting better. The best response by the nurse is which of the following? a) "This doctor has been on our staff for 20 years." b) "I know you are worried, but the doctor has an excellent reputation." c) "You always have an option to change. Tell me about your concerns." d) "I take my own children to this doctor."

c) CORRECT ANSWER "You always have an option to change. Tell me about your concerns." Reason: Asking the mother to talk about her concerns acknowledges the mother's rights and encourages open discussion. The other responses negate the parent's concerns.

A client in the triage area who is at 19 weeks' gestation states that she has not felt her baby move in the past week and no fetal heart tones are found. While evaluating this client, the nurse identifies her as being at the highest risk for developing which problem? a) Abruptio placentae. b) Placenta previa. c) Disseminated intravascular coagulation. d) Threatened abortion.

c) CORRECT ANSWER Disseminated intravascular coagulation. Reason: A fetus that has died and is retained in utero places the mother at risk for disseminated intravascular coagulation (DIC) because the clotting factors within the maternal system are consumed when the nonviable fetus is retained. The longer the fetus is retained in utero, the greater the risk of DIC. This client has no risk factors, history, or signs and symptoms that put her at risk for either abruptio placentae or placenta previa, such as sharp pain and "woody," firm consistency of the abdomen (abruption) or painless bright red vaginal bleeding (previa). There is no evidence that she is threatening to abort as she has no complaints of cramping or vaginal bleeding.

Which of the following laboratory findings are expected when a client has diverticulitis? a) Elevated red blood cell count. b) Decreased platelet count. c) Elevated white blood cell count. d) Elevated serum blood urea nitrogen concentration.

c) CORRECT ANSWER Elevated white blood cell count. Reason: Because of the inflammatory nature of diverticulitis, the nurse would anticipate an elevated white blood cell count. The remaining laboratory findings are not associated with diverticulitis. Elevated red blood cell counts occur in clients with polycythemia vera or fluid volume deficit. Decreased platelet counts can occur as a result of aplastic anemias or malignant blood disorders, as an adverse effect of some drugs, and as a result of some heritable conditions. Elevated serum blood urea nitrogen concentration is usually associated with renal conditions.

A nurse is developing a nursing diagnosis for a client. Which information should she include? a) Actions to achieve goals b) Expected outcomes c) Factors influencing the client's problem d) Nursing history

c) CORRECT ANSWER Factors influencing the client's problem Reason: A nursing diagnosis is a written statement describing a client's actual or potential health problem. It includes a specified diagnostic label, factors that influence the client's problem, and any signs or symptoms that help define the diagnostic label. Actions to achieve goals are nursing interventions. Expected outcomes are measurable behavioral goals that the nurse develops during the evaluation step of the nursing process. The nurse obtains a nursing history during the assessment step of the nursing process.

A client who has been diagnosed with gastroesophageal reflux disease (GERD) complains of heartburn. To decrease the heartburn, the nurse should instruct the client to eliminate which of the following items from the diet? a) Lean beef. b) Air-popped popcorn. c) Hot chocolate. d) Raw vegetables.

c) CORRECT ANSWER Hot chocolate. Reason: With GERD, eating substances that decrease lower esophageal sphincter pressure causes heartburn. A decrease in the lower esophageal sphincter pressure allows gastric contents to reflux into the lower end of the esophagus. Foods that can cause a decrease in esophageal sphincter pressure include fatty foods, chocolate, caffeinated beverages, peppermint, and alcohol. A diet high in protein and low in fat is recommended for clients with GERD. Lean beef, popcorn, and raw vegetables would be acceptable.

The nurse is caring for a client with asthma. The nurse should conduct a focused assessment to detect which of the following? a) Increased forced expiratory volume. b) Normal breath sounds. c) Inspiratory and expiratory wheezing. d) Morning headaches.

c) CORRECT ANSWER Inspiratory and expiratory wheezing. Reason: The hallmark signs of asthma are chest tightness, audible wheezing, and coughing. Inspiratory and expiratory wheezing is the result of bronchoconstriction. Even between exacerbations, there may be some soft wheezing, so a finding of normal breath sounds would be expected in the absence of asthma. The expected finding is decreased forced expiratory volume [forced expiratory flow (FEF) is the flow (or speed) of air coming out of the lung during the middle portion of a forced expiration] due to bronchial constriction. Morning headaches are found with more advanced cases of COPD and signal nocturnal hypercapnia or hypoxemia.

Which of the following is a priority during the first 24 hours of hospitalization for a comatose client with suspected drug overdose? a) Educate regarding drug abuse. b) Minimize pain. c) Maintain intact skin. d) Increase caloric intake.

c) CORRECT ANSWER Maintain intact skin. Reason: Maintaining intact skin is a priority for the unconscious client. Unconscious clients need to be turned every hour to prevent complications of immobility, which include pressure ulcers and stasis pneumonia. The unconscious client cannot be educated at this time. Pain is not a concern. During the first 24 hours, the unconscious client will mostly likely be on nothing-by-mouth status.

A nurse is performing a psychosocial assessment on a 14-year-old adolescent. Which emotional response is typical during early adolescence? a) Frequent anger b) Cooperativeness c) Moodiness d) Combativeness

c) CORRECT ANSWER Moodiness Reason: Moodiness may occur often during early adolescence. Frequent anger and combativeness are more typical of middle adolescence. Cooperativeness typically occurs during late adolescence.

An anxious young adult is brought to the interviewing room of a crisis shelter, sobbing and saying that she thinks she is pregnant but does not know what to do. Which of the following nursing interventions is most appropriate at this time? a) Ask the client about the type of things that she had thought of doing. b) Give the client some ideas about what to expect to happen next. c) Recommend a pregnancy test after acknowledging the client's distress. d) Question the client about her feelings and possible parental reactions.

c) CORRECT ANSWER Recommend a pregnancy test after acknowledging the client's distress. Reason: Before any interventions can occur, knowing whether the client is pregnant is crucial in formulating a plan of care. Asking the client about what things she had thought about doing, giving the client some ideas about what to expect next, and questioning the client about her feelings and possible parental reactions would be appropriate after it is determined that the client is pregnant.

A man of Chinese descent is admitted to the hospital with multiple injuries after a motor vehicle accident. His pain is not under control. The client states, "If I could be with my people, I could receive acupuncture for this pain." The nurse should understand that acupuncture in the Asian culture is based on the theory that it: a) Purges evil spirits. b) Promotes tranquility. c) Restores the balance of energy. d) Blocks nerve pathways to the brain.

c) CORRECT ANSWER Restores the balance of energy. Reason: Acupuncture, like acumassage and acupressure, is performed in certain Asian cultures to restore the energy balance within the body. Pressure, massage, and fine needles are applied to energy pathways to help restore the body's balance. Acupuncture is not based on a belief in purging evil spirits. Although pain relief through acupuncture can promote tranquility, acupuncture is performed to restore energy balance. In the Western world, many researchers think that the gate-control theory of pain may explain the success of acupuncture, acumassage, and acupressure.

The nurse walks into a client's room to administer the 9:00 a.m. medications and notices that the client is in an awkward position in bed. What is the nurse's first action? a) Ask the client his name. b) Check the client's name band. c) Straighten the client's pillow behind his back. d) Give the client his medications.

c) CORRECT ANSWER Straighten the client's pillow behind his back. Reason: The nurse should first help the client into a position of comfort even though the primary purpose for entering the room was to administer medication. After attending to the client's basic care needs, the nurse can proceed with the proper identification of the client, such as asking the client his name and checking his armband, so that the medication can be administered.

A client was hospitalized for 1 week with major depression with suicidal ideation. He is taking venlafaxine (Effexor), 75 mg three times a day, and is planning to return to work. The nurse asks the client if he is experiencing thoughts of self-harm. The client responds, "I hardly think about it anymore and wouldn't do anything to hurt myself." The nurse should make which judgment about the client? a) The client is decompensating and in need of being readmitted to the hospital. b) The client needs an adjustment or increase in his dose of antidepressant. c) The depression is improving and the suicidal ideation is lessening. d) The presence of suicidal ideation warrants a telephone call to the client's primary care provider.

c) CORRECT ANSWER The depression is improving and the suicidal ideation is lessening. Reason: The client's statements about being in control of his behavior and his or her plans to return to work indicate an improvement in depression and that suicidal ideation, although present, is decreasing. Nothing in his comments or behavior indicate he is decompensating. There is no evidence to support an increase or adjustment in the dose of Effexor or a call to the primary care provider. Typically, the cognitive components of depression are the last symptoms eliminated. For the client to be experiencing some suicidal ideation in the second week of psychopharmacologic treatment is not unusual.

Before an incisional cholecystectomy is performed, the nurse instructs the client in the correct use of an incentive spirometer. Why is incentive spirometry essential after surgery in the upper abdominal area? a) The client will be maintained on bed rest for several days. b) Ambulation is restricted by the presence of drainage tubes. c) The operative incision is near the diaphragm. d) The presence of a nasogastric tube inhibits deep breathing.

c) CORRECT ANSWER The operative incision is near the diaphragm. Reason: The incisions made for upper abdominal surgeries, such as cholecystectomies, are near the diaphragm and make deep breathing painful. Incentive spirometry, which encourages deep breathing, is essential to prevent atelectasis after surgery. The client is not maintained on bed rest for several days. The client is encouraged to ambulate by the first postoperative day, even with drainage tubes in place. Nasogastric tubes do not inhibit deep breathing and coughing.

A client received chemotherapy 24 hours ago. Which precautions are necessary when caring for the client? a) Wear sterile gloves. b) Place incontinence pads in the regular trash container. c) Wear personal protective equipment when handling blood, body fluids, and feces. d) Provide a urinal or bedpan to decrease the likelihood of soiling linens.

c) CORRECT ANSWER Wear personal protective equipment when handling blood, body fluids, and feces. Reason: Chemotherapy drugs are present in the waste and body fluids of clients for 48 hours after administration. The nurse should wear personal protective equipment when handling blood, body fluids, or feces. Gloves offer minimal protection against exposure. The nurse should wear a face shield, gown, and gloves when exposure to blood or body fluid is likely. Placing incontinence pads in the regular trash container and providing a urinal or bedpan don't protect the nurse caring for the client.

A client has just been diagnosed with early glaucoma. During a teaching session, the nurse should: a) provide instructions on eye patching. b) assess the client's visual acuity. c) demonstrate eyedrop instillation. d) teach about intraocular lens cleaning.

c) CORRECT ANSWER demonstrate eyedrop instillation. Reason: Eyedrop instillation is a critical component of self-care for a client with glaucoma. After demonstrating eyedrop instillation to the client and family, the nurse should verify their ability to perform this measure properly. An eye patch isn't necessary unless the client has undergone surgery. Visual acuity assessment isn't necessary before discharge. Intraocular lenses aren't implanted in clients with glaucoma.

When developing a care plan for a client with a do-not-resuscitate (DNR) order, a nurse should: a) withhold food and fluids. b) discontinue pain medications. c) ensure access to spiritual care providers upon the client's request. d) always make the DNR client the last in prioritization of clients.

c) CORRECT ANSWER ensure access to spiritual care providers upon the client's request. Reason: Ensuring access to spiritual care, if requested by the client, is an appropriate nursing action. A nurse should continue to administer appropriate doses of pain medication as needed to promote the client's comfort. A health care provider may not withhold food and fluids unless the client has a living will that specifies this action. A DNR order does not mean that the client does not require nursing care.

"The nurse is working with an overweight client who has a high-stress job and smokes. This client has just received a diagnosis of Type II Diabetes and has just been started on an oral hypoglycemic agent. Which of the following goals for the client which if met, would be most likely to lead to an improvement in insulin efficiency to the point the client would no longer require oral hypoglycemic agents? "a. Comply with medication regimen 100% for 6 months b. Quit the use of any tobacco products by the end of three months c. Lose a pound a week until weight is in normal range for height and exercise 30 minutes daily d. Practice relaxation techniques for at least five minutes five times a day for at least five months"

c. Lose a pound a week until weight is in normal range for height and exercise 30 minutes daily When type II diabetics lose weight through diet and exercise they sometimes have an improvement in insulin efficiency sufficient to the degree they no longer require oral hypoglycemic agents.

"dipine" - amlodipine, elodipine, nifedipine, nimodipine, nisoldipine

calcium channel blocker - antihypertensive

The client is taking risperidone (Risperdal) to treat the positive and negative symptoms of schizophrenia. Improvement of which of the following negative symptoms indicate the drug is effective? a) Abnormal thought form. b) Hallucinations and delusions. c) Bizarre behavior. d) Asocial behavior and anergia.

d - Asocial behavior, anergia, alogia, and affective flattening are some of the negative symptoms of schizophrenia that may improve with risperidone therapy. Abnormal thought form is a positive symptom of schizophrenia. Hallucinations and delusions are positive symptoms of schizophrenia. Bizarre behavior is a positive symptom of schizophrenia.

A client was talking with her husband by telephone, and then she began swearing at him. The nurse interrupts the call and offers to talk with the client. She says, "I can't talk about that bastard right now. I just need to destroy something." Which of the following should the nurse do next? a) Tell her to write her feelings in her journal. b) Urge her to talk with the nurse now. c) Ask her to calm down or she will be restrained. d) Offer her a phone book to "destroy" while staying with her.

d - At this level of aggression, the client needs an appropriate physical outlet for the anger. She is beyond writing in a journal. Urging the client to talk to the nurse now or making threats, such as telling her that she will be restrained, is inappropriate and could lead to an escalation of her anger.

A 56-year-old client is receiving chemotherapy that has the potential to cause pulmonary toxicity. Which of the following symptoms indicates a toxic response to the chemotherapy? a) Decrease in appetite. b) Drowsiness. c) Spasms of the diaphragm. d) Cough and shortness of breath.

d - Cough and shortness of breath are significant symptoms because they may indicate decreasing pulmonary function secondary to drug toxicity. Decrease in appetite, difficulty in thinking clearly, and spasms of the diaphragm may occur as a result of chemotherapy; however, they are not indicative of pulmonary toxicity.

While assessing a male neonate whose mother desires him to be circumcised, the nurse observes that the neonate's urinary meatus appears to be located on the ventral surface of the penis. The primary health care provider is notified because the nurse suspects which of the following? a. phimosis b. hydrocele c. epispadias d. hydrospadias

d - The condition in which the urinary meatus is located on the ventral surface of the penis, termed hypospadias, occurs in 1 of every 500 male infants. Circumcision is delayed until the condition is corrected surgically, usually between 6 and 12 months of age. Phimosis is an inability to retract the prepuce at an age when it should be retractable or by age 3 years. Phimosis may necessitate circumcision or surgical intervention. Hydrocele is a painless swelling of the scrotum that is common in neonates. It is not a contraindication for circumcision. Epispadias occurs when the urinary meatus is located on the dorsal surface of the penis. It is extremely rare and is commonly associated with bladder extrophy.

A client with burns on his groin has developed blisters. As the client is bathing, a few blisters break. The best action for the nurse to take is to: a) remove the raised skin because the blister has already broken. b) wash the area with soap and water to disinfect it. c) apply a weakened alcohol solution to clean the area. d) clean the area with normal saline solution and cover it with a protective dressing.

d - The nurse should clean the area with a mild solution such as normal saline, and then cover it with a protective dressing. Soap and water and alcohol are too harsh. The body's first line of defense broke when the blisters opened; removing the skin exposes a larger area to the risk of infection.

A client diagnosed with pain disorder is talking with the nurse about fishing when he suddenly reverts to talking about the pain in his arm. Which of the following should the nurse do next? a) Allow the client to talk about his pain. b) Ask the client if he needs more pain medication. c) Get up and leave the client. d) Redirect the interaction back to fishing.

d - The nurse should redirect the interaction back to fishing or another focus whenever the client begins to ruminate about physical symptoms or impairment. Doing so helps the client talk about topics that are more therapeutic and beneficial to recovery. Allowing the client to talk about his pain or asking if he needs additional pain medication is not therapeutic because it reinforces the client's need for the symptom. Getting up and leaving the client is not appropriate unless the nurse has set limits previously by saying, "I will get up and leave if you continue to talk about your pain."

Nurses teach infant care and safety classes to assist parents in appropriately preparing to take their neonates home. Which statement about automobile restraints for infants is correct? a) An infant should ride in a front-facing car seat until he weighs 20 lb (9.1 kg) and is 1 year old. b) An infant should ride in a rear-facing car seat until he weighs 25 lb (11.3 kg) or is 1 year old. c) An infant should ride in a front-facing car seat until he weighs 30 lb (13.6 kg) or is 2 years old. d) An infant should ride in a rear-facing car seat until he weighs 20 lb and is 1 year old.

d - Until the infant weighs 20 lb and is 1 year old, he should ride in a rear-facing car seat.

A client has refused to take a shower since being admitted 4 days earlier. He tells a nurse, "there are poison crystals hidden in the showerhead. They'll kill me if I take a shower." Which nursing action is most appropriate? a. dismantling the showerhead and showing the client that there is nothing in it b. explaining that other clients are complaining about the client's body odor c. asking a security officer to assist in giving the client a shower d. accepting these fears and allowing the client to take a sponge bath

d - by acknowledging the client's fears, the nurse can arrange to meet the client's hygiene needs in another way. Because these fears are real to the client, providing a demonstration of reality by dismantling the shower head wouldn't be effective at this time.

a nurse is caring for a client who required chest tube insertion for a pneumothorax. To assess for pneumothorax resolution, the nurse can anticipate that the client will require: a. monitoring of arterial oxygen saturation (SaO2) b. arterial blood gas (ABG) studies c. chest auscultation d. chest x ray

d - chest x ray confirms diagnosis by revealing air or fluid in the pleural space. SaO2 values may initially decrease with a pneumothorax but typically return to normal within 24 hours. ABG studies may show hypoxemia, possibly with respiratory acidosis and hypercapnia but these are not necessarily related to a pneumothorax. Chest auscultation will determine overall lung status, but it's difficult to determine if the best has re-expanded sufficiently.

The client with a hearing aid does not seem to be able to hear the nurse. The nurse should do which of the following? a. contact the client's audiologist b. cleanse the hearing aid ear mold in normal saline c. irrigate the ear canal d. check the hearing aid's placement

d - inadequate amplification can occur when a hearing aid is not place properly. The certified audiologist is licensed to dispense hearing aids. The ear mold is the only part of the hearing aid that may be wash frequently; it should be washed daily with soap and water. Irrigation of the ear canal is done to remove impacted cerumen or a foreign body

To prevent development of peripheral neuropathies associated with isoniazid administration, the nurse should teach the client to: a. avoid excessive sun exposure b. follow a low-cholesterol diet c. obtain extra rest d. supplement the diet with pyridoxine (vitamin B6)

d - isoniazid competes for the available vitamin B6 in the body and leaves the client at risk for developing neuropathies related to vitamin deficiency. Supplemental vitamin B6 is routinely prescribed to address this issue. Avoiding sun exposure is a preventative measure to lower the risk of skin cancer. Following a low-cholesterol diet lowers the individual's risk of developing atherosclerotic plaque. Rest is important in maintaining homeostasis but has no real impact on neuropathies.

On the second postpartum day a gravida 6, para 5 complains of intermittent abdominal cramping. The nurse should assess for: a) endometritis. b) postpartum hemorrhage. c) subinvolution. d) afterpains.

d) CORRECT ANSWER afterpains. Reason: In a multiparous client, decreased uterine muscle tone causes alternating relaxation and contraction during uterine involution, which leads to afterpains. The client's symptoms don't suggest endometritis, hemorrhage, or subinvolution.

A 16-year-old academically gifted boy is about to graduate from high school early, because he has completed all courses needed to earn a diploma. Within the last 3 months, he has experienced panic attacks that have forced him to leave classes early and occasionally miss a day of school. He is concerned that these attacks may hinder his ability to pursue a college degree. What would be the best response by the school nurse who has been helping him deal with his panic attacks? a) "It is natural to be worried about going into a new environment. I am sure with your abilities you will do well once you get settled." b) "You are putting too much pressure on yourself. You just need to relax more and things will be alright." c) "It might be best for you to postpone going to college. You need to get these panic attacks controlled first." d) "It sounds like you have a real concern about transitioning to college. I can refer you to a health care provider for assessment and treatment."

d) CORRECT ANSWER "It sounds like you have a real concern about transitioning to college. I can refer you to a health care provider for assessment and treatment." Reason: The client's concerns are real and serious enough to warrant assessment by a physician rather than being dismissed as trivial. Though he is very intelligent, his intelligence cannot overcome his anxiety. In fact, his anxiety is likely to interfere with his ability to perform in college if no assessment and treatment are received. Just postponing college is likely to increase rather than lower the client's anxiety, because it does not address the panic he is experiencing.

A 16-year-old primigravida at 36 weeks' gestation who has had no prenatal care experienced a seizure at work and is being transported to the hospital by ambulance. Which of the following should the nurse do upon the client's arrival? a) Position the client in a supine position. b) Auscultate breath sounds every 4 hours. c) Monitor the vital signs every 4 hours. d) Admit the client to a quiet, darkened room.

d) CORRECT ANSWER Admit the client to a quiet, darkened room. Reason: Because of her age and report of a seizure, the client is probably experiencing eclampsia, a condition in which convulsions occur in the absence of any underlying cause. Although the actual cause is unknown, adolescents and women older than 35 years are at higher risk. The client's environment should be kept as free of stimuli as possible. Thus, the nurse should admit the client to a quiet, darkened room. Clients experiencing eclampsia should be kept on the left side to promote placental perfusion. In some cases, edema of the lungs develops after seizures and is a sign of cardiovascular failure. Because the client is at risk for pulmonary edema, breath sounds should be monitored every 2 hours. Vital signs should be monitored frequently, at least every hour.

A nurse is caring for a client diagnosed with a cerebral aneurysm who reports a severe headache. Which action should the nurse perform? a) Sit with the client for a few minutes. b) Administer an analgesic. c) Inform the nurse manager. d) Call the physician immediately.

d) CORRECT ANSWER Call the physician immediately. Reason: The nurse should notify the physician immediately because the headache may be an indication that the aneurysm is leaking. Sitting with the client is appropriate but only after the physician has been notified of the change in the client's condition. The physician will decide whether or not administration of an analgesic is indicated. Informing the nurse manager isn't necessary.

During the first feeding, the nurse observes that the neonate becomes cyanotic after gagging on mucus. Which of the following should the nurse do first? a) Start mouth-to-mouth resuscitation. b) Contact the neonatal resuscitation team. c) Raise the neonate's head and pat the back gently. d) Clear the neonate's airway with suction or gravity.

d) CORRECT ANSWER Clear the neonate's airway with suction or gravity. Reason: If a neonate gags on mucus and becomes cyanotic during the first feeding, the airway is most likely closed. The nurse should clear the airway by gravity (by lowering the infant's head) or suction. Starting mouth-to-mouth resuscitation is not indicated unless the neonate remains cyanotic and lowering his head or suctioning doesn't clear his airway. Contacting the neonatal resuscitation team is not warranted unless the infant remains cyanotic even after measures to clear the airway. Raising the neonate's head and patting the back are not appropriate actions for removing mucus. Doing so allows the mucus to remain lodged causing further breathing difficulties.

A nurse is performing a baseline assessment of a client's skin integrity. What is the priority assessment parameter? a) Family history of pressure ulcers b) Presence of pressure ulcers on the client c) Potential areas of pressure ulcer development d) Overall risk of developing pressure ulcers

d) CORRECT ANSWER Overall risk of developing pressure ulcers Reason: When assessing skin integrity, the overall risk potential of developing pressure ulcers takes priority. Overall risk encompasses existing pressure ulcers as well as potential areas for development of pressure ulcers. Family history isn't important when assessing skin integrity.

A nurse is caring for a client with a diagnosis of Impaired gas exchange. Based upon this nursing diagnosis, which outcome is most appropriate? a) The client maintains a reduced cough effort to lessen fatigue. b) The client restricts fluid intake to prevent overhydration. c) The client reduces daily activities to a minimum. d) The client has normal breath sounds in all lung fields.

d) CORRECT ANSWER The client has normal breath sounds in all lung fields. Reason: If the interventions are effective, the client's breath sounds should return to normal. The client should be able to cough effectively and should be encouraged to increase activity, as tolerated. Fluids should help thin secretions, so fluid intake should be encouraged.

Based on a client's history of violence toward others and her inability to cope with anger, which of the following should the nurse use as the most important indicator of goal achievement before discharge? a) Acknowledgment of her angry feelings. b) Ability to describe situations that provoke angry feelings. c) Development of a list of how she has handled her anger in the past. d) Verbalization of her feelings in an appropriate manner.

d) CORRECT ANSWER Verbalization of her feelings in an appropriate manner. Reason: Verbalizing feelings, especially feelings of anger, in an appropriate manner is an adaptive method of coping that reduces the chance that the client will act out these feelings toward others. The client's ability to verbalize her feelings indicates a change in behavior, a crucial indicator of goal achievement. Although acknowledging feelings of anger and describing situations that precipitate angry feelings are important in helping the client reach her goal, they are not appropriate indicators that she has changed her behavior. Asking the client to list how she has handled anger in the past is helpful if the nurse discusses coping methods with the client. However, based on this client's history, this would not be helpful because the nurse and client are already aware of the client's aggression toward others.

A nurse is providing care for a pregnant client in her second trimester. Glucose tolerance test results show a blood glucose level of 160 mg/dl. The nurse should anticipate that the client will need to: a) start using insulin. b) start taking an oral antidiabetic drug. c) monitor her urine for glucose. d) be taught about diet.

d) CORRECT ANSWER be taught about diet. Reason: The client will need to watch her overall diet intake to control her blood glucose level. The client's blood glucose level should be controlled initially by diet and exercise, rather than insulin. Oral antidiabetic drugs aren't used in pregnant clients. Urine glucose levels aren't an accurate indication of blood glucose levels.

When assessing an elderly client, the nurse expects to find various aging-related physiologic changes. These changes include: a) increased coronary artery blood flow. b) decreased posterior thoracic curve. c) decreased peripheral resistance. d) delayed gastric emptying.

d) CORRECT ANSWER delayed gastric emptying. Reason: Aging-related physiologic changes include delayed gastric emptying, decreased coronary artery blood flow, an increased posterior thoracic curve, and increased peripheral resistance.

A client with chronic undifferentiated schizophrenia is admitted to the psychiatric unit of a local hospital. During the next several days, the client is seen laughing, yelling, and talking to himself. This behavior is characteristic of: a) delusion. b) looseness of association. c) illusion. d) hallucination.

d) CORRECT ANSWER hallucination. Reason: Auditory hallucination, in which one hears voices when no external stimuli exist, is common in schizophrenic clients. Such behaviors as laughing, yelling, and talking to oneself suggest such a hallucination. Delusions, also common in schizophrenia, are false beliefs or ideas that arise without external stimuli. Clients with schizophrenia may exhibit looseness of association, a pattern of thinking and communicating in which ideas aren't clearly linked to one another. Illusion is a less severe perceptual disturbance in which the client misinterprets actual external stimuli. Illusions are rarely associated with schizophrenia.

"An adolescent client with type I diabetes mellitus is admitted to the emergency department for treatment of diabetic ketoacidosis. Which assessment findings should the nurse expect to note? "a) sweating and tremors b) hunger and hypertension c) cold, clammy skin and irritability d) fruity breath and decreasing level of consciousness

d) fruity breath and decreasing level of consciousness"Hyperglycemia occurs with diabetic ketoacidosis. Signs of hyperglycemia include fruity breath and a decreasing level of consciousness. Hunger can be a sign of hypoglycemia or hyperglycemia, but hypertension is not a sign of diabetic ketoacidosis. Instead, hypotension occurs because of a decrease in blood volume related to the dehydrated state that occurs during diabetic ketoacidosis. Cold, clammy skin, irritability, sweating, and tremors are all signs of hypoglycemia."

When taking a health history, the nurse screens for manifestations suggestive of diabetes type I. Which of the following manifestations are considered the primary manifestations of diabetes type I and would be most suggestive of diabetes type I and require follow-up investigation? "a. Excessive intake of calories, rapid weight gain, and difficulty losing weight b. Poor circulation, wound healing, and leg ulcers, c. Lack of energy, weight gain, and depression d. An increase in three areas: thirst, intake of fluids, and hunger

d. An increase in three areas: thirst, intake of fluids, and hunger "The primary manifestations of diabetes type I are polyuria (increased urine output), polydipsia (increased thirst), polyphagia (increased hunger). Excessive calorie intake, weight gain, and difficulty losing weight are common risk factors for type 2 diabetes. Poor circulation, wound healing and leg ulcers are signs of chronic diabetes. Lack of energy, weight gain and depression are not necessarily indicative of any type of diabetes."

Isoproterenol - NC

stimulates beta 1 & 2 adrenergic receptors used for heart block, ventricular arrhythmias, and bradycardia bronchodilator for asthma and bronchospasms don't give at night (disrupts sleep patterns) monitor BP and pulse

Dobutamine - NC

stimulates beta 1 receptors incompatible w/ alkaline solutions (sodium bicarb) central venous cath or large peripheral vein incompatible w/ many meds monitor EKG, BP, I&O, serum K+

"cycline" - demeclocycline, doxycycline, minocycline, tetracycline

tetracycline antibiotic


Kaugnay na mga set ng pag-aaral

Accounting, Finance & Valuation (Chapter 2)

View Set

Topic Review Guide: Gene Regulation

View Set

Chapter 1: What is multimedia? - Quiz 1

View Set

Chapter 21 Assignment Question Set Eco 252

View Set

Chapter 7 - Strict Liability and Product Liability

View Set

3000 most common words in spoken english - second 1000 words

View Set